SlideShare una empresa de Scribd logo
1 de 46
Descargar para leer sin conexión
1
Modelo 2014. Pregunta 5A.- Considere la reacción en equilibrio A (g) + 3B (g) 2C (g). Cuando se introduce 1
mol de A y 3 mol de B en un recipiente de 5 L y se alcanza el equilibrio a 350 K, se observa que se han formado 1,6 mol
de C.
a) Calcule la constante de equilibro Kp de la reacción a 350 K.
Dato. R = 0,082 atm·L·mol−1
·K−1
.
Puntuación máxima por apartado: 0,5 puntos.
Solución.
a. x ≡ moles de A que reaccionan. El cuadro de reacción es:
( ) ( ) ( )
( )
( ) x2x33x1molEquilibrio.C
31molIniciales.C
gC2gB3gA
−−
−
↔+
( )( ) x26,1gCn Eq ==
( )( )
( )( )



=⋅−=
=−=
=
mol6,08,033gBn
mol2,08,01gAn
:mol8,0x
eq
eq
Conocidos los moles de todos los componentes del equilibrio y el volumen del sistema, se calcula Kc, y conocido este
valor, mediante la relación entre las constantes, se calcula Kp ( ) ( )( )gn
cp RTKK ∆⋅= .
Por la ley de Acción de Masas:
[ ]
[ ] [ ]
( )
( ) ( )
5.1481
5
6,0
5
2,0
5
6,1
V
Bn
V
An
V
Cn
BA
C
K
3
2
3
2
3
2
c =




⋅









=





⋅










=
⋅
=
( ) ( ) ( ) ( ) 799.1350082,05,1481RTKK 312gn
cp =⋅⋅=⋅= +−∆
Modelo 2014. Pregunta 5B.- El producto de solubilidad del hidróxido de hierro (III) a 25 ºC es Ks = 2,8×10−39
.
a) Calcule la solubilidad de este hidróxido, en g·L−1
.
Datos. Masas atómicas: Fe = 55,8; O = 16,0; H = 1,0; Cl = 35,5.
Puntuación máxima por apartado: 0,5 puntos apartado a); 0,75 puntos apartados b) y c).
Solución.
a. El equilibrio heterogéneo de solubilidad del Fe(OH)3 es:
( ) ( ) ( ) −+
+↔ OH3aqFesOHFe 3
3 [ ] [ ]33
s OHFeK −+ ⋅=
Si definimos por s los moles por litro de hidróxido disueltos, el cuadro de reacción queda de la siguiente forma.
( ) ( ) ( )
s3s
OH3aqFesOHFe 3
3
−
+↔ −+
Sustituyendo en la expresión del producto de solubilidad, se obtiene la solubilidad del hidróxido en mol L‒1
.
[ ][ ] ( ) 3333
s s27s3sOHFeK =⋅=⋅= −+ 1104
39
4 s lmol1001,1
27
108,2
27
K
s −−
−
⋅×=
×
==
Para expresarla en g L‒1
se multiplica por la masa molecular del hidróxido.
18110 Lg1008,1
mol
g8,106
lmol1001,1s −−−− ⋅×=⋅⋅×=
Septiembre 2013. Pregunta A4.- Se introduce fosgeno (COCl2) en un recipiente vacío de 1 L a una presión de 0,92
atm y temperatura de 500 K, produciéndose su descomposición según la ecuación: COCl2 (g) ↔ CO (g) + Cl2 (g).
Sabiendo que en estas condiciones el valor de Kc es 4,63×10–3
; calcule:
a) La concentración inicial de fosgeno.
b) Las concentraciones de todas las especies en el equilibrio.
c) La presión parcial de cada uno de los componentes en el equilibrio.
Dato. R = 0,082 atm·L·mol−1·K−1
.
Puntuación máxima por apartado: 0,5 puntos apartado a); 0,75 puntos apartados b) y c).
Solución.
2
a. Aplicando la ecuación de gases ideales a las condiciones iniciales:
nRTVP =⋅
RT
P
V
n
=
( )
[ ]
RT
P
COCl
V
COCln o
o2
o2
==
[ ] L
mol1024,2
K500
Kmol
Latm
082,0
atm92,0
COCl 2
o2
−
×=
⋅
⋅
⋅
=
b. Si se denomina x al número de moles de fosgeno que se disocian, el cuadro de reacción queda de la siguiente
forma:
( ) ( ) ( )
( )
( ) xxx92,0
L
molequilibrio.Cond
0,92
L
moliniciales.Cond
gClgCOgCOCl 22
−
−−
+↔
Según la ley de acción de masas, la constante de equilibrio en función de las concentraciones es
[ ] [ ]
[ ]2
2
c
COCl
ClCO
K
⋅
=
Sustituyendo por los valores de cuadro de reacción y por el valor de la contante, y ordenando, se obtiene una
ecuación de segundo grado.
x1024,2
x
x1024,2
xx
1063,4 2
2
2
3
−×
=
−×
⋅
=× −−
−




−=
×=
=×−×+
−
−−
químicosentidotieneNo01,0x
101,8x
:01004,11063,4x
3
432
[ ] L
mol1043,1101,81024,2COCl 232
eq2
−−−
×=×−×=
[ ] [ ] L
mol101,8ClCO 3
eq2eq
−
×==
c. Aplicando la ecuación de gases ideales a cada componente de la mezcla gaseosa:
nRTVP =⋅ RT
V
n
P =
[ ] atm0,586K500
Kmol
Latm
082,0
L
mol
1043,1RTCOClRT
V
n
P 2
2
COCl
COCl
2
2
=⋅
⋅
⋅
⋅×=⋅== −
[ ] atm0,332K500
Kmol
Latm
082,0
L
mol
101,8RTClRT
V
n
PP 3
2
Cl
ClCO
2
2
=⋅
⋅
⋅
⋅×=⋅=== −
Septiembre 2013. Pregunta B2.- Se tiene una reacción en equilibrio del tipo:
aA (g) + bB (g) ↔ cC (l) + dD (s).
a) Escriba la expresión de Kp.
b) Justifique cómo se modifica el equilibrio cuando se duplica el volumen del recipiente.
c) Justifique cómo se modifica el equilibrio si se aumenta la presión parcial de la sustancia A.
d) Justifique qué le ocurre al valor de Kp si aumenta la temperatura del sistema.
Puntuación máxima por apartado: 0,5 puntos.
Solución.
a. Según la ley de acción de masas, las constantes de equilibrios heterogéneos solo dependen de las sustancias que
estén en el estado de agregación con mayor libertad, en el caso de un equilibrio sólido-líquido-gas, solo dependerá de las
sustancias que estén en estado gaseoso.
b
B
a
A
p
PP
1
K
⋅
=
b. Si en un sistema en equilibrio aumenta el volumen o disminuye la presión, el equilibrio se desplaza hacia el
sentido donde exista mayor número de moles gaseosos y de esa forma contrarrestar el aumento de volumen o la
disminución de presión.
3
c. Si en el sistema en equilibrio aumentamos la presión parcial de A, y teniendo en cuenta que el valor de Kp no
varia con la presión, deberá disminuir la presión parcial de B, lo cual se consigue desplazando el equilibrio hacia la
derecha.
d. Dependerá de que la reacción sea endotérmica o exotérmica. Si la reacción es endotérmica, al aumentar la
temperatura aumentará el valor de Kp, y si es exotérmica, al aumentar la temperatura disminuirá el valor de Kp.
Junio 2013. Pregunta 2A.- Justifique si son verdaderas o falsas las siguientes afirmaciones:
d. La constante de solubilidad de una sal poco soluble aumenta por efecto ión común.
Puntuación máxima por apartado: 0,5 puntos.
Solución.
d. Falso. El efecto ión común, influye en la solubilidad de la sal, pero no influye en la constante, la cual solo es
función de la temperatura.
Junio 2013. Pregunta 5A.- El valor de la constante de equilibrio Kc para la reacción H2 (g) + F2 (g) 2HF (g),
es 6,6×10–4
a 25 ºC. Si en un recipiente de 10 L se introduce 1 mol de H2 y 1 mol de F2, y se mantiene a 25 ºC hasta
alcanzar el equilibrio, calcule:
a) Los moles de H2 que quedan sin reaccionar una vez que se ha alcanzado el equilibrio.
b) La presión parcial de cada uno de los compuestos en el equilibrio.
c) El valor de Kp a 25 ºC.
Dato. R = 0,082 atm·L·mol−1
·K−1
.
Puntuación máxima por apartado: 0,75 puntos apartados a) y b); 0,5 puntos apartado c).
Solución.
a. Si se denomina por x al número de moles que reaccionan de hidrógeno y fluor, el cuadro de reacción en función
de x queda de la siguiente forma:
( ) ( ) ( )
( )
( ) x2x1x1mol.Eq.C
11mol.I.C
gHF2gFgH 22
−−
−
⇔+
El número de moles de cada componente en el equilibrio se obtiene calculando x a partir del valor de la
constante de equilibrio.
[ ]
[ ] [ ]
( )
( ) ( )
( )( )
( ) ( )
( )
( ) ( )
( )
( )2
22
22
2
22
2
22
2
c
x1
x2
x1x1
x2
FnHn
HFn
V
Fn
V
Hn
V
HFn
FH
HF
K
−
=
−⋅−
=
⋅
=
⋅






=
⋅
=
2
c
x1
x2
K 





−
= cK
x1
x2
=
−
mol1027,1
106,62
106,6
K2
K
x 2
4
4
c
c −
−
−
×=
×+
×
=
+
=
( ) mol987,01027,11x1Hn 2
Eq2 =×−=−= −
b. La presión parcial de un componente de una mezcla gaseosa es
V
RTn
P i
i
⋅
=
( ) atm4,2
10
298082,0987,0
V
RTHn
PP 2
FH 22
=
⋅⋅
=
⋅
==
( ) atm062,0
10
298082,01027,12
V
RTHFn
P
2
HF =
⋅⋅×⋅
=
⋅
=
−
c. ( ) ( ) ( ) c
0
c
gn
cp KRTKRTKK =⋅=⋅= ∆
Junio 2013. Pregunta 2B.- La siguiente reacción, no ajustada: CH3OH (l) + O2 (g) H2O (l) + CO2 (g) es
exotérmica a 25 ºC.
a) Escriba la expresión para la constante de equilibrio Kp de la reacción indicada.
b) Razone cómo afecta al equilibrio un aumento de la temperatura.
c) Razone cómo afecta a la cantidad de CO2 desprendido un aumento de la cantidad de CH3OH (l).
4
d) Justifique cómo se modifica el equilibrio si se elimina CO2 del reactor.
Puntuación máxima por apartado: 0,5 puntos.
Solución.
NOTA: Por lo general, las reacciones de combustión son irreversibles, por lo tanto no tiene sentido hablar de equilibrio
en una reacción de combustión. En cualquier caso, como se propone como equilibrio, lo trataremos como tal.
a. Por ser un equilibrio heterogéneo líquido/gas, las constantes de equilibrio solo serán función de las especies que
estén en el estado de agregación de mayor libertad (gas).
( ) ( ) ( ) ( )gCO2lOH4gO3lOHCH2 2223 +↔+
3
O
2
CO
p
2
2
P
P
K =
b. Según el principio de Le Châtelier, al producir una perturbación en un sistema en equilibrio, este evoluciona en
contra de la perturbación de forma que reestablezca el equilibrio. Si se aumenta la temperatura, el sistema tiende a
desplazarse en el sentido endotérmico (absorbiendo calor), y de esa forma restablecer el equilibrio. Teniendo en cuenta
que la reacción es exotérmica, tal y como dice el enunciado, el sentido endotérmico será hacia la izquierda, por lo tanto,
al aumentar la temperatura, el equilibrio se desplaza hacia los reactivos.
c. Por encontrarse el metanol en estado líquido y ser un equilibrio heterogéneo líquido/gas, la concentración de
etanol no influye en el equilibrio, y por tanto, no influye en la cantidad de CO2 desprendida.
d. Al eliminar CO2, el sistema se desplaza hacia la derecha, generando más CO2, y oponiéndose a la perturbación.
Modelo 2013. Pregunta 1A.- Cuando se trata agua líquida con exceso de azufre sólido en un recipiente cerrado, a
25 ºC, se obtienen los gases sulfuro de hidrógeno y dióxido de azufre.
a) Formule el equilibrio que se establece entre reactivos y productos.
b) Escriba las expresiones de Kc y Kp.
c) Indique cómo afecta al equilibrio un aumento de presión.
Puntuación máxima por apartado: 0,5 puntos.
Solución.
a. ( ) ( ) ( ) ( )gSOgSH2sS3lOH2 222 +↔+
b. En los equilibrios heterogéneos, las constantes solo dependen la las especies que se encuentren en el estado de
agregación con mayor grado de libertad. El equilibrio propuesto es sólido / líquido / gas, por lo tanto las constantes solo
depende de los componentes que estén en estado gas.
[ ] [ ]2
2
2c SOSHK ⋅= 22
SO
2
SHp PPK ⋅=
c. Según Le Chàtelier, si un sistema químico en equilibrio experimenta un cambio, entonces el equilibrio se
desplaza para contrarrestar el cambio impuesto y restablecer el equilibrio.
Si se aumenta la presión, el equilibrio se desplazará en el sentido en el que disminuya el volumen y de esa forma
contrarrestar el aumento de presión. Como el número de moles de especies gaseosas es mayor en los productos, el
equilibrio se desplaza hacia los reactivos.
Modelo 2013. Pregunta 2B.- El yoduro de bismuto (III) es una sal muy poco soluble en agua.
a) Escriba el equilibrio de solubilidad del yoduro de bismuto sólido en agua.
b) Escriba la expresión para la solubilidad del compuesto BiI3 en función de su producto de solubilidad.
c) Sabiendo que la sal presenta una solubilidad de 0,7761 mg en 100 mL de agua a 20 ºC, calcule la constante del
producto de solubilidad a esa temperatura.
Datos. Masas atómicas: Bi = 209,0; I = 126,9
Puntuación máxima por apartado: 0,5 puntos apartados a) y b); 1 punto apartado c).
Solución.
a. ( ) ( ) ( )aqI3aqBisBiI 3OH
3
2 −+
+ →←
b. Si se solubilizan s moles por litro de la sal:
( ) ( ) ( )
s3sexc
aqI3aqBisBiI 3OH
3
2 −+
+ →←
5
[ ] [ ] ( ) 44333
s
27
Ks
ss27s3sIBiK =⇒=⋅=⋅= −+
c.
( )
( ) ( ) L
mol1032,1
L10100
molg9,1263209
g107761,0
LV
M
m
LV
disueltaSaln
s 6
3
3
−
−
−
×=
×
⋅+
×
===
[ ] [ ] ( ) 1946433
s 102,81032,127s27IBiK −−−+
×=×⋅==⋅=
Modelo 2013. Pregunta 4B.- En un recipiente de 15 litros se introducen 3 mol de compuesto A y 2 mol del
compuesto B.
Cuando se calienta el recipiente a 400 K se establece el siguiente equilibrio: 2 A (g) + B (g) ⇔ 3 C (g).
Sabiendo que cuando se alcanza el equilibrio las presiones parciales de B y C son iguales, calcule:
a) Las concentraciones de A, B y C en el equilibrio.
b) La presión total en el equilibrio.
c) El valor de las constantes de equilibrio Kc y Kp a 400 K.
Dato. R = 0,082 atm·L·mol−1
·K−1
Puntuación máxima por apartado: 1 punto apartado a); 0,5 puntos apartados b) y c).
Solución.
a. Equilibrio homogéneo en fase gas. Si se denomina como x al número de moles que reaccionan de B, el
cuadrado de reacción queda de la siguiente forma:
( ) ( ) ( )
( )
( ) x3x2x23molEquilibrio.C
23molIniciales.C
gC3gBgA2
−−
−
↔+
Si en el equilibrio las presiones parciales de B y C son iguales y, teniendo en cuenta que
T
RTn
P i
i = , el número
de moles de B y C también sarán iguales 





=⇒=== CBC
CB
B nnP
T
RTn
T
RTn
P .
( ) ( ) mol5,0xx3x2CnBn ==−=
Conocidos los moles en el equilibrio de cada componente y el volumen del recipiente, se calculan las
concentraciones.
( ) [ ] ( )
( )
( ) [ ] ( )
( )
( ) [ ] ( )
( )
1
1
1
lmol1,0
15
5,1
lV
Cn
C5,15,03Cn
lmol1,0
15
5,1
lV
Bn
B5,15,02Bn
lmol133,0
15
2
lV
An
A25,023An
−
−
−
⋅====⋅=
⋅====−=
⋅====⋅−=
b. Conocidos los moles de cada componente del equilibrio se calcula el número total de moles, y con La ecuación
de gases ideales, se calcula la presión total.
( ) ( ) ( ) mol55,15,12CnBnAnnT =++=++=
atm9,10
L15
K400
Kmol
Latm
082,0mol5
V
RTn
P T =
⋅⋅
==
c. Conocidas las concentraciones de cada uno de los componentes del equilibrio, se calcula la constante de
equilibrio en función de las concentraciones aplicando la ley de Acción de Masas.
[ ]
[ ] [ ]
565,0
1,0133,0
1,0
CA
C
K 2
3
2
3
c
⋅
=
⋅
=
Conocida Kc se calcula Kp mediante la relación entre ellas.
( ) ( ) ( ) ( ) ( ) 565,0KRTKRTKRTKK c
0
c
123
c
gn
cp ==⋅=⋅=⋅= +−∆
6
Septiembre 2012. Pregunta A5.- En un recipiente cerrado de 1 L de capacidad se introducen 73,6 gramos de
tetraóxido de dinitrógeno. Se mantiene a 22 ºC hasta alcanzar el equilibrio N2O4 (g) ⇔ 2NO2 (g), siendo Kc = 4,66×10‒3
.
a) Calcule las concentraciones de ambos gases en el equilibrio.
b) Calcule el valor de Kp.
c) Cuando la temperatura aumenta al doble, aumenta Kc. Justifique el signo de ∆H para esta reacción.
Datos. R = 0,082 atm·L·mol‒1
·K‒1
. Masas atómicas: N = 14 y O = 16.
Puntuación máxima por apartado: 1 punto apartado a); 0,5 puntos apartados b) y c).
Solución.
a. Se plantea el cuadro de reacción para la disociación del tetraóxido de dinitrógeno denominando por x la
concentración de tetraóxido de dinitrógeno disociado.
( ) ( )
x2xCEquilibrio.Cond
CInicialesCond.
gNO2gON
o
o
242
−
−
↔
Llevando las condiciones del equilibrio a la definición de la constante Kc, se plantea una ecuación de segundo
grado que permite calcular el valor de x.
[ ]
[ ]
( )
xC
x4
xC
x2
ON
NO
K
o
2
o
2
42
2
2
c
−
=
−
==
Ordenando: 0CKxKx4 occ
2
=−+
[ ] ( ) ( ) ( )
L
mol8,0
L1
molg92
g6,73
V
ONMONm
V
ONn
ONC 424242
42o =====



−=
=
=⋅×−×+ −−
válidaNo03,0x
03,0x
:08,01066,41066,4x4 332
[ ] L
mol77,003,08,0xCON o42 =−=−=
[ ] L
mol06,003,02x2NO2 =⋅==
b.
[ ]
[ ]( )
[ ]
[ ]
[ ]
RTKRT
ON
NO
RTON
RTNO
RTCP
idealesgases.Ecc
P
P
K c
42
2
2
42
2
2
iiON
2
NO
P
42
2
⋅==
⋅
⋅
=






⋅=
==
113,0295082,01066,4RTKK 3
cP =⋅⋅×=⋅= −
c. Según Le Chatelier al producir una perturbación en un sistema en equilibrio, el sistema evoluciona en contra de
la perturbación, si aumenta la temperatura el sistema se desplaza en el sentido endotérmico, consumiendo calor y de esta
forma oponiéndose al aumento de temperatura. Si al aumentar la temperatura, aumenta la constante (aumentan los
productos y disminuyen los reactivos), el equilibrio se esta desplazando hacia la derecha, por consiguiente el sentido
endotérmico es hacia la derecha, la reacción es ENDOTÉRMICA.
Septiembre 2012. Pregunta B2.- Para las sales cloruro de plata y yoduro de plata, cuyas constantes de producto de
solubilidad, a 25 ºC, son 1,6×10‒10
y 8×10‒17
, respectivamente:
a) Formule los equilibrios heterogéneos de disociación y escriba las expresiones para las constantes del producto
de solubilidad de cada una de las sales indicadas, en función de sus solubilidades.
b) Calcule la solubilidad de cada una de estas sales en g·L‒1
.
c) ¿Que efecto produce la adición de cloruro de sodio sobre una disolución saturada de cloruro de plata?
d) ¿Como varia la solubilidad de la mayoría de las sales al aumentar la temperatura? Justifique la respuesta.
Datos. Masas atómicas: Cl = 35,5; Ag = 108,0; I = 127,0.
Puntuación máxima por apartado: 0,5 puntos.
Solución.
a. Equilibrios heterogéneos sólido/líquido.
( ) ( ) ( )
ssexc
aqClaqAgsAgCl −+
+↔
[ ] [ ] 2
s sClAgK =⋅= −+
7
( ) ( ) ( )
ssexc
aqIaqAgsAgI −+
+↔
[ ] [ ] 2
s sIAgK =⋅= −+
b. AgCl: ( ) 2
s sAgClK = ⇒ ( ) L
mol1026,1106,1KAgCls 510
s
−−
×=×==
( ) 135
Lg1082,1
mol
g
5,143
L
mol
1026,1AgCls −−−
⋅×=⋅×=
AgI: ( ) 2
s sAgIK = ⇒ ( ) L
mol1094,8108KAgIs 917
s
−−
×=×==
( ) 169
Lg101,2
mol
g
235
L
mol
1094,8AgIs −−−
⋅×=⋅×=
c. Según Le Chatelier al producir una perturbación en un sistema en equilibrio, el sistema evoluciona en contra de
la perturbación, si se aumenta la concentración de cloruros, el sistema evoluciona en el sentido en el que se consuma
cloruros, hacia la izquierda, disminuyendo la solubilidad de la sal, y produciendo la aparición de un precipitado de AgCl.
d. Para la mayoría de la sales el proceso de disolución es endotérmico
( ) ( ) ( )aqBaqAQsAB −+
+↔+
Al aumentar la temperatura, el equilibrio se desplaza en el sentido endotérmico, consumiendo calor y
oponiéndose de esa forma al aumento de temperatura, por lo tanto se desplaza hacia la derecha aumentando la
solubilidad de la sal.
Junio 2012. Pregunta 5A.- Se introducen 0,5 moles de pentacloruro de antimonio en un recipiente de 2 litros. Se
calienta a 200 ºC y una vez alcanzado el equilibrio, hay presentes 0,436 moles del compuesto. Todas las especies son
gases a esa temperatura.
a) Escriba la reacción de descomposición del pentacloruro de antimonio en cloro molecular y en tricloruro de
antimonio.
b) Calcula Kc para la reacción anterior.
c) Calcule la presión total de la mezcla en el equilibrio.
Datos. R = 0,082 atm·L·K‒1
·mol‒1
.
Puntuación máxima por apartado: 0,5 puntos apartado a); 0,75 puntos apartado b) y c).
Solución.
a. ( ) ( ) ( )gClgSbClgSbCl 235 +↔
b. Por la Ley de Acción de Masas, la constante de equilibrio es:
[ ] [ ]
[ ]5
23
c
SbCl
ClSbCl
K
⋅
=
Para calcular las concentraciones en el equilibrio, se plantea el siguiente cuadro de reacción (mol), donde x
representa los moles de pentacloruro de antimonio que reaccionan.
( ) ( )
xxx5,0Equilibrio.C
0,5Iniciales.C
ClgSbClgSbCl 235
−
−−
+↔
Por los datos del enunciado, se sabe que el número de moles de pentacloruro de antimonio en el equilibrio son
0,436.
436,0x5,0 =− ; mol064,0x =
Conocidos los moles de todas la especies en el equilibrio, se calculan susu concentraciones, y con estas, el valor
de la constante de equilibrio.
[ ] ( ) [ ] [ ] L
mol032,0
2
064,0
V
n
ClSbCl;
L
mol218,0
2
436,0
V
SbCln
SbCl
Eq
23
5
5 =======
8
3
c 107,4
436,0
032,0032,0
K −
×=
⋅
=
c. La presión en el equilibrio se calcula mediante la ecuación de gases ideales con el número total de moles de la
mezcla gaseosa.
( ) ( ) ( ) mol564,0064,0064,0436,0ClnSbClnSbClnn eq2eq3eq5T =++=++=
atm9,10
2
473082,0564,0
V
nRT
P =
⋅⋅
==
Modelo 2012. Pregunta 2B.- Para la reacción en fase gaseosa A + B ↔ C los valores de entalpía de reacción y
energía de activación de la reacción directa son: ∆H = −150 kJ·mol‒1
y Ea = 85 kJ·mol‒1
.
a. Justifique el efecto de un aumento de temperatura en la constante de equilibrio y en la composición en
equilibrio.
c. Justifique el efecto de un aumento de volumen en la constante de equilibrio y en la composición en equilibrio.
Puntuación máxima por apartado: 0,5 puntos.
Solución.
En los apartados referentes a perturbaciones sobre el equilibrio, hay que tener en cuenta la Ley de Le Chatelier.
“Siempre que se modifique las condiciones de un sistema en equilibrio se produce un desplazamiento del mismo en el
sentido que restablezca las condiciones iniciales”.
a. Al aumentar la temperatura en un sistema en equilibrio, este se desplaza en el sentido endotérmico, absorbiendo
calor y oponiéndose a la perturbación. Por tratarse de una reacción exotérmica, el sentido endotérmico es hacia la
izquierda, por lo tanto, al aumentar la temperatura el sistema se desplaza hacia la izquierda, aumentando las
concentraciones de los reactivos y disminuyendo la de los productos, lo cual produce una disminución en el valor de la
constante de equilibrio.
c. Las variaciones de volumen no afectan al valor de la constante de equilibrio, la cual solo es función de la
temperatura.
Para equilibrios gaseosos con variación del número de moles gaseosos entre reactivos y productos ( )( )0gn ≠∆ ,
las variaciones de volumen desplazan el equilibrio. Si se aumenta el volumen, el equilibrio se desplaza en el mismo
sentido que lo haría con una disminución de presión, se desplazará en el sentido en el que aumente el volumen del
sistema, para la reacción propuesta hacia la izquierda (reactivos).
Septiembre 2011. Pregunta 5A.- Cuando se ponen 0,7 moles de N2O4 en un reactor de 10 L a 359 K se establece el
equilibrio
N2O4 (g) ↔ 2 NO2 (g) y la presión es de 3,3 atm. Calcule:
a) La concentración molar de todas las especies en el equilibrio.
b) El valor de Kc.
c) Si el sistema se comprime hasta reducir el volumen a 8 L ¿cual seria la presión total en el equilibrio?
Dato. R = 0,082 atm·L·mol‒1
·K‒1
.
Puntuación máxima por apartado: a) y c) 0,75 puntos; b) 0,5 puntos.
Solución.
a. Para resolver el problema es conveniente plantear el siguiente cuadro de reacción, donde x representa los moles
de N2O4 que se disocian.
El número total de moles en el equilibrio es la suma de los moles de N2O4 y de NO2.
( ) ( ) ( ) x7,0x2x7,0NOnONneqn Eq2Eq42T +=+−=+=
El número de moles en el equilibrio, se puede calcular mediante la ecuación de gases ideales conocidas las
condiciones del equilibrio (P, V y T).
mol12,1
K359
Kmol
Latm
082,0
L10atm3,3
RT
PV
nT =
⋅
⋅
⋅
⋅
==
9
Conocidos los moles totales en el equilibrio se calculan los moles disociados de N2O4.
( ) 12,1x7,0eqnT =+= ; mol42,0x =
Conocidos los moles de N2O4 disociados se calculan los moles de cada compuesto en el equilibrio, y dividiendo
estos por el volumen las concentraciones en el equilibrio.
( ) [ ] ( )
( ) [ ] ( ) M084,0
10
84,0
V
NOn
NOmol84,042,02x2NOn
M028,0
10
28,0
V
ONn
ONmol28,042,07,0x7,0ONn
2
2Eq2
42
42Eq42
====⋅==
====−=−=
b.
[ ]
[ ]
252,0
028,0
084,0
ON
NO
K
2
42
2
2
c ===
c. Para calcular la nueva presión una vez alcanzado el equilibrio después de la perturbación, es necesario calcular
el número de moles de cada componente en el nuevo equilibrio.
Para calcular las nuevas condiciones del equilibrio se tiene en cuenta que el valor de la constante no depende del
volumen.
[ ]
[ ] 42
2
42
2
ON
2
NO
ON
2
NO
42
2
2
c
n
n
V
1
V
n
V
n
ON
NO
K ⋅=








==
Al disminuir el volumen del sistema, y siendo diferente el número de moles gaseosos en reactivos y productos,
el equilibrio se desplaza hacia donde menor volumen ocupe, hacia la izquierda (reactivos). Si se denomina como x ahora
al número de moles de NO2 que se dimerizan para volver a formar N2O4, el cuadro de reacción queda de la siguiente
forma:
Llevando los valores del equilibrio a la expresión de la constante, se llega a una ecuación de segundo grado.
42
2
ON
2
NO
c
n
n
V
1
K ⋅= ;
( )
2
x
28,0
x84,0
8
1
252,0
2
+
−
⋅= ; ( )2
x84,0
2
x
28,08252,0 −=





+⋅⋅



=
=
=+−
Válida0,054x
válidaNo63,2x
:0142,0x688,2x2
Conocido x se calculan los moles de cada componente en el nuevo equilibrio.
( ) mol307,0
2
054,0
28,0ONn Eq42 =+= ; ( ) mol786,0054,084,0NOn Eq2 =−=
( ) ( ) ( ) mol093,1786,0307,0NOnONnTn Eq2Eq42Eq =+=+=
Conocidos los moles en el equilibrio, con la ecuación de gases ideales se calcula la nueva presión.
atm4
L8
K359
Kmol
Latm
0,082mol093,1
V
nRT
P ≈
⋅
⋅
⋅
⋅
==
Septiembre 2011. Pregunta 2B.- El hidróxido de magnesio es poco soluble en agua (Ks = 1,8·10‒11
).
a) Formule el equilibrio de disolución del hidróxido de magnesio y escriba la expresión para Ks.
b) Calcule la solubilidad en mol·L‒1
.
c) Como afectaría a la solubilidad la adición de acido clorhídrico?
d) Como afectaría a la solubilidad la adición de cloruro de magnesio?
Puntuación máxima por apartado: 0,5 puntos.
Solución.
a. ( ) ( ) ( ) ( )aqOH2aqMgsOHMg 2
2
−+
+↔
10
[ ] [ ]22
s OHMgK −+
⋅=
b. Si se disuelven s moles por litro de la sal, el cuadro de reacción será:
Sustituyendo en la expresión de la constante se obtiene una relación entre la constante de solubilidad y la
solubilidad de la sal.
[ ] [ ] ( ) 3222
s s4s2sOHMgK =⋅=⋅= −+
; 143
11
3 s Lmol1065,1
4
108,1
4
K
s −−
−
⋅⋅=
⋅
==
c. Al añadir un ácido fuerte, los protones del ácido reaccionan con OH‒
del hidróxido formando agua, al disminuir
la concentración de OH‒
, el sistema se desplaza hacia la derecha para volver a recuperar el equilibrio, disolviéndose mas
cantidad de Mg(OH)2, y aumentando su solubilidad.
d. Al añadir MgCl2 se produce el efecto de ión común, aumentando la concentración de Mg2+
lo que desplaza el
equilibrio hacia la izquierda y disminuye la solubilidad.
Junio 2011. Pregunta 5A.- En un recipiente de 5 L se introducen 3,2 g de COCl2 a 300 K. Cuando se alcanza el
equilibrio COCl2 ↔ CO + Cl2, la presión final es de 180 mm de Hg. Calcule:
d) Las presiones parciales de COCl2, CO y Cl2 en el equilibrio.
e) Las constantes de equilibrio Kp y Kc.
Datos. R = 0,082 atm·L·mol‒1
· K‒1
; Masas atómicas: C = 12; 0= 16; Cl = 35,5.
Puntuación máxima por apartado: a) 1 punto.
Solución.
a. Las presiones parciales de los componentes del equilibrio se pueden calcular mediante la ecuación de gases
ideales conocido el volumen, la temperatura y los moles de cada componente.
V
RTn
P i
i =
Para calcular los moles en el equilibrio hay que tener en cuenta que el fosgeno (COCl2) se disocia según el
siguiente cuadro de reacción.
( ) ( ) ( )
( )
( ) xxxnmolEquilibrioC.
nmolinicialesC.
gClgCOgCOCl
o
o
22
−
−−
+⇔
Siendo no el número de moles iniciales y x los moles de fosgeno (COCl2) que se disocian.
El número total de moles en el equilibrio se puede calcular con los datos del equilibrio y como suma de los
moles de cada uno de los componentes.
( ) ( ) ( ) xnClnCOnCOClnn oEq2EqEq2Eq +=++=
mol048,0
K300
Kmol
Latm
082,0
L5atm
760
180
RT
VP
n
Eq
Eq =
⋅
⋅
⋅
⋅
=
⋅
= :
( )
( )
mol032,0
mol
g99
g2,3
COClM
COClm
n
2
2
o ===
Sustituyendo en la igualdad anterior:
x032,0048,0 += : 016,0x =
Número de moles y presiones parciales en el equilibrio
( )
( ) ( ) atm08,0
L5
K300
Kmol
Latm
082,0mol016,0
PP:016,0xClnCOn
atm08,0
L5
K300
Kmol
Latm
082,0mol016,0
P:016,0016.0032,0xnCOCln
2
2
ClCOEq2Eq
COCloEq2
=
⋅
⋅
⋅
⋅
=====
=
⋅
⋅
⋅
⋅
==−=−=
11
El problema también se puede resolver en función de las presiones planteando el cuadro de reacción en función
de la presión.
( ) ( ) ( )
( )
( ) xxxPatmEquilibrioC.
PatminicialesC.
gClgCOgCOCl
o
o
22
−
−−
+⇔
Siendo x la presión de equilibrio del monóxido de carbono y del cloro molecular.
xPPPPPP oClCOCOCliT 22
+=++== ∑
atm24,0
760
180
PT == : atm16,0
L5
K300
Kmol
Latm
082,0
mol
g99
g2,3
V
RTn
P o
o =
⋅
⋅
⋅
⋅
==
16,0x24,0 += : atm08,0x =
atm08,016,024,0xPP oCOCl2
=−=−= : atm08,0xPP 2ClCO ===
b. Conocidas las presiones parciales se calcula el valor de la constante KP, y mediante la relación entre las
constantes el valor de KC.
08,0
08.0
08.008,0
P
PP
K
2
2
COCl
ClCO
P =
⋅
=
⋅
=
( ) ( )
3
12n
P
c 1025,3
300082,0
08,0
RT
K
K −
−∆
×=
⋅
==
Modelo 2011. Cuestión 2A.- Diga si son ciertas o falsas las siguientes afirmaciones, razonando sus respuestas:
d. La solubilidad del fluoruro de magnesio en agua es 8,2510−5
M. Dato. Ks = 6,8·10−9
.
Puntuación máxima por apartado: 0,5 puntos.
Solución.
d. Falso. El fluoruro de magnesio se disocia según:
( ) ( ) ( )aqF2aqMgsMgF 2OH
2
2 −+
+ →
La solubilidad de la sal se puede obtener del producto de solubilidad a partir de su definición en función de las
concentraciones y estas últimas, en función de la solubilidad (s) de la sal (por cada s moles por litro que se disuelven de
la sal se forman s moles por litro de catión magnesio y 2 s moles litro de fluoruro).
( ) 322
s s4s2sFMgK =⋅=⋅= −+
M1019,1
4
108,6
4
K
s 33
9
3 s −
−
⋅=
⋅
==
Modelo 2011. Problema 2A.- A 532 K se introducen 0,1 moles de PCl5 en un recipiente X de 1,2 L y 0,1 moles en
otro recipiente Y. Se establece el equilibrio PCl5 ↔ PCl3 + Cl2, y la cantidad de PCl5 se reduce un 50% en el recipiente
X y un 90% en el recipiente Y. Todas las especies se encuentran en fase gaseosa. Calcule:
a) La presión en el equilibrio en el recipiente X.
b) La constante de equilibrio Kc.
c) El volumen del recipiente Y.
d) La presión en el equilibrio en el recipiente Y.
Datos. R = 0,082 atm・L・mol−1
・K−1
.
Puntuación máxima por apartado: 0,5 puntos.
Solución.
a. Independientemente del reactor donde se lleve a cabo la reacción y, teniendo en cuenta que se conoce el número
de moles iniciales y el grado de disociación ( )9,0;5,0 YX =α=α , el cuadros de reacción es el que se muestra a
continuación
12
La presión en el interior del reactor X se calcula mediante la ecuación de gases ideales, siendo necesario
calcular el número de moles totales que hay en el equilibrio.
( ) ( ) ( ) ( ) ( ) ( ) ( )=α+α+α−=++= XoXoXoo235T nnnnClnPClnPClnXn
( ) ( ) mol15,05,011,01nnn XoXoo =+⋅=α+=α+=
RTnVP XXX =⋅ : atm45,5
L1,2
K532
Kmol
Latm
0,082mol15,0
V
RTn
P
X
X
X =
⋅
⋅
⋅
⋅
==
b. La constante Kc, se obtiene por la ley de acción de masas.
( ) ( )
( )
( ) ( )
( ) ( )Xo
XoXo
5
23
5
23
5
23
c
1n
nn
V
1
PCln
ClnPCln
V
1
V
PCln
V
Cln
V
PCln
PCl
ClPCl
K
α−
α⋅α
⋅=
⋅
⋅=
⋅
=
⋅
=
2
2
X
2
Xo
X
c 1017,4
5,01
5,01,0
2,1
1
1
n
V
1
K −
×=
−
⋅
⋅=
α−
α
⋅=
c. Teniendo en cuenta que el valor de constante solo depende de la temperatura, y que los dos reactores están a la
misma temperatura, la constante de equilibrio aplicada al reactor Y permite calcular su volumen.
Y
2
Yo
Y
c
1
n
V
1
K
α−
α
⋅= : L4,19
9,01
9,01,0
1017,4
1
1
n
K
1
V
2
2
Y
2
Yo
c
Y =
−
⋅
⋅
⋅
=
α−
α
⋅= −
d. Al igual que en el apartado a, la presión en el interior del reactor X se calcula mediante la ecuación de gases
ideales.
( ) ( ) ( ) mol19,09,011,01nYn YoT =+⋅=α+=
atm43,0
L19,4
K532
Kmol
Latm
0,082mol19,0
V
RTn
P
Y
Y
Y =
⋅
⋅
⋅
⋅
==
Modelo 2011. Cuestión 2B.- En sendos recipientes R1 y R2, de 1 L cada uno, se introduce 1 mol de los compuestos
A y B, respectivamente. Se producen las reacciones cuya información se resume en la tabla:
Reacción Concentración
inicial
Ecuación cinética
reacción directa
Constante
cinética
Constante de
equilibrio
R1 A ↔ C + D [A]o = 1 M v1 = k1 [A] k1 = 1 s−1
K1 = 50
R2 B ↔ E + F [B]o = 1 M v2 = k2 [B] k2 = 100 s−1
K2 = 2×10−3
Justifique las siguientes afirmaciones, todas ellas verdaderas.
b. Cuando se alcance el equilibrio, la concentración de A será menor que la de B.
c. Para las reacciones inversas en R1 y R2 se cumple k−1 < k−2.
Puntuación máxima por apartado: 0,5 puntos.
Solución.
b. En este apartado debemos fijarnos en el valor de la constante de equilibrio. Teniendo en cuenta que las
concentraciones iniciales son iguales y que K1 > K2, el equilibrio en la reacción 1 (R1) esta mas desplazado hacia la
derecha (productos) que el de la reacción 2 (R2), por tanto, en el equilibrio [A] < [B].
c. El equilibrio químico es dinámico, las reacciones directa e inversa no se detienen al llegar al equilibrio sino que
sus velocidades se igualan y por tanto las concentraciones de todas las especies presentes en el mismo permanecen
constante. En el equilibrio, las concentraciones de A y B ([A], [B]) son menores a las concentraciones iniciales, teniendo
en cuenta que la velocidad de reacción es directamente proporcional a la concentración de reactivo como pone de
manifiesto la ecuación cinética (v1 = k1 [A]; v2 = k2 [B]), al disminuir las concentraciones, disminuye la velocidad.
Septiembre 2010. FM. Problema 2A.- En un recipiente de 14 L de volumen se introducen 3,2 moles de nitrógeno y
3 moles de hidrógeno. Cuando se alcanza el equilibrio a 200°C se obtienen 1,6 moles de amoniaco.
a) Formule y ajuste la reacción.
b) Calcule el número de moles de H2 y de N2 en el equilibrio.
13
c) Calcule los valores de las presiones parciales en el equilibrio de H2, N2 y NH3
d) Calcule Kc y Kp a 200°C.
Dato. R = 0,082 atm·L·mol−1
·K−1
Puntuación máxima por apartado: 0,5 puntos
Solución.
a. ( ) ( ) ( )gNH2gH3gN 322 ↔+
b. Para calcular el número de moles en el equilibrio se plantea el cuadro de reacción definiendo x como el número
de moles de nitrógeno que reaccionan.
Sabiendo que el número de moles de NH3 en el equilibrio (2x) es 1,6, se calcula x.
6,1x2 = : mol8,0x =
Conocidos los moles de nitrógeno que han reaccionado, se pueden calcular los moles de cada componente en el
equilibrio.
( ) mol4,28,02,3Nn eq2 =−=
( ) mol6,08,033Hn eq2 =⋅−=
( ) mol6,1NHn eq3 =
c. Aplicando la ecuación de gases ideales a cada componentes de la mezcla gaseosa, se calculan las presiones
parciales.
RTnVP ii =⋅
Donde Pi es la presión parcial de componente i y ni los moles de dicho componente.
( )
atm65,6
14
473082,04,2
V
RTNn
P 2
N2
=
⋅⋅
=
⋅
=
( )
atm66,1
14
473082,06,0
V
RTHn
P 2
H2
=
⋅⋅
=
⋅
=
( )
atm43,4
14
473082,06,1
V
RTNHn
P 3
NH3
=
⋅⋅
=
⋅
=
d. Con los datos obtenidos en el apartado c se calcula el valor de KP,
65,0
66,165,6
43,4
PP
P
K 3
2
3
HN
2
NH
P
22
3
=
⋅
=
⋅
=
Conocido KP se calcula KC mediante la relación entre ellas.
( ) ( )gn
CP RTKK ∆
⋅=
( ) ( )
( )
5,970
473082,0
65,0
RT
K
K 42gn
P
c =
⋅
== −∆
Septiembre 2010. FM. Cuestión 2B.- La síntesis del amoniaco según la reacción en fase
gaseosa, 322 NH2H3N ↔+ , es un buen ejemplo para diferenciar factores cinéticos y termodinámicos.
d. Escriba la expresión para KP en función de la presión total.
Dato. ( ) 0NHH 3f
o
<∆
Puntuación máxima por apartado: 0.5 puntos.
Solución.
d. Para la reacción ( ) ( ) ( )gNH2gH3gN 322 ↔+ , la constante de equilibrio en función de las presiones parciales
tiene la expresión:
14
3
HN
2
NH
P
22
3
PP
P
K
⋅
=
Teniendo en cuenta que la presión parcial de un componente de una mezcla gaseosa es igual a la presión total
por la fracción molar ( )ii PP χ⋅= , se puede expresar Kp en función de la presión total y de la fracción molar de todos
los componentes de la mezcla gaseosa.
( )
( ) 3
HN
2
NH
23
HN
2
NH
HH
NN
NHNH
3
HN
2
NH
P
22
3
22
3
22
22
33
22
3
P
1
PP
P
PP
PP
PP
PP
P
K
χ⋅χ
χ
⋅=
χ⋅⋅χ⋅
χ⋅
=










χ⋅=
χ⋅=
χ⋅=
=
⋅
=
Septiembre 2010. Problema lA.- A 330 K y l atm, 368 g de una mezcla al 50% en masa de N02 y N2O4 se
encuentran en equilibrio. Calcule:
a) La fracción molar de cada componente en dicha mezcla.
b) La constante de equilibrio Kp para la reacción 2 N02 ↔ N204
c) La presión necesaria para que la cantidad de N02 en el equilibrio se reduzca a la mitad.
d) El volumen que ocupa la mezcla del apartado c) en el equilibrio.
Datos. R = 0,082 atm·L·K−1
·mol−1
masas atómicas: N = 14; O = 16
Puntuación máxima por apartado: 0.5 puntos.
Solución.
a. La fracción molar de un componente de una mezcla gaseosa es:
T
i
i
n
n
=χ ; siendo ∑= iT nn
( ) ( )
( )
( )
( )
( )
( )
( )






===
===
===
mol2
mol
g29
g184
ONM
ONm
ONn
mol4
mol
g46
g184
NOM
NOm
NOn
:g184
2
368
ONmNOm
42
42
42
2
2
2
422
67,0
6
4
24
4
2NO ==
+
=χ ; 33,0
6
2
24
2
42ON ==
+
=χ
b. El equilibrio de la reacción de dimerización del dióxido de nitrógeno, viene representado por:
{ }
( )
75,0
67,0
33,0
1
1
P
1
P
P
PP
P
P
K
22
NO
ON
2
NO
ON
ii2
NO
ON
p
2
42
2
42
2
42
=⋅=
χ
χ
⋅=
χ⋅
χ⋅
=χ⋅===
c. Si la temperatura permanece constante, Kp permanece constante. Para que la cantidad de NO2 se
reduzca a la mitad, deben reaccionar dos moles, según la estequiometria del proceso, la nueva composición de
equilibrio será:
Las nuevas fracciones molares son:





==
+
=χ
==
+
=χ
60,0
5
3
32
3
40,0
5
2
32
2
42
2
ON
NO
;
Teniendo en cuenta que la constante de equilibrio no varia, se calcula la nueva presión de equilibrio.
2
NO
ON
2
NO
ON
p
2
42
2
42
P
1
P
P
K
χ
χ
⋅== : atm5
40,0
60,0
75,0
1
K
1
P
22
NO
ON
p
2
42
=⋅=
χ
χ
⋅=
15
El aumento de presión que experimenta el sistema para desplazarse a la derecha y, de esta forma
disminuir el número de moles de NO2, esta de acuerdo con leyes de Lechatelier. Al aumentar la presión el
sistema evoluciona hacia donde menor volumen ocupa, contrarrestando de esta forma el aumento de presión.
d. Conocida la presión, el número de moles y la temperatura, la ecuación de gases ideales permite
calcular el volumen de la mezcla de equilibrio.
nRTPV = :
( ) L27
5
330082,032
P
nRT
V =
⋅⋅+
==
Septiembre 2010. FG. Cuestión 2B.- La siguiente descomposición:
2 NaHCO3 (s) → Na2CO3 (s) + H2O (g) + CO2 (g), es un proceso endotérmico.
a) Escriba la expresión para la constante de equilibrio Kp de la reacción indicada.
b) Razone cómo afecta al equilibrio un aumento de la temperatura.
c) Razone cómo afecta a la cantidad de CO2 desprendido un aumento de la cantidad de NaHC03
d) Justifique cómo afecta al equilibrio la eliminación del CO2 del medio.
Puntuación máxima por apartado: 0.5 puntos.
Solución.
a. Equilibrio heterogéneo. Las sustancias sólidas se encuentran en exceso y por tanto sus concentraciones se
mantienen constantes y se introducen en el valor de la constante, quedando la constante únicamente en función de
las especies en estado gaseoso.
22 COOHp PPK ⋅=
b. Según Le Chatelier, al aumentar la temperatura en un sistema, el equilibrio se restablece desplazándose en
el sentido en el que se consuma calor (sentido endotérmico), contrarrestando de esta forma el aumento de
temperatura. Teniendo en cuenta que la reacción es endotérmica, el sistema se desplaza hacia la derecha
(productos), consumiendo calor.
c. Por estar en fase sólida, y por tanto no formar parte de la constante, el cociente de reacción no se ve
afectado por aumentar la concentración de bicarbonato sódico (NaHCO3), por lo que no desplaza el equilibrio y
por tanto no modifica la cantidad de CO2.
d. Según Le Chatelier, al introducir una perturbación en un sistema en equilibrio, el sistema reaccionará de
forma que se oponga a la perturbación y de esa forma restablecer el equilibrio.
Si se elimina dióxido de carbono, el equilibrio se desplazará en el sentido de formación de CO2, es decir,
hacia la derecha (productos).
Junio 2010. FM. Problema 2A.- Se parte de 150 gramos de ácido etanoico, y se quieren obtener 176 gramos de
etanoato de etilo por reacción con etanol.
a) Escriba la reacción de obtención del etanoato de etilo indicando de qué tipo es.
b) Sabiendo que Kc vale 5, calcule los gramos de alcohol que hay que utilizar.
c) Calcule las fracciones molares de cada uno de los 4 compuestos presentes en el equilibrio.
Datos. Masas atómicas: C = 12; O = 16; H = 1
Puntuación máxima por apartado: 1 punto.
Solución.
a. Reacción de esterificación. Adición con eliminación
OHCHCHCOOCHOHCHCHCOOHCH 2323233 +−−−↔−+−
Ac. Etanoico Etanol Etanoato de etilo Agua
b. Se trata de un equilibrio de esterificación en el que se conocen los moles iniciales del ácido, los moles en el
equilibrio del ester y la constante de equilibrio. Si denominamos por x a los moles de alcohol iniciales y por y, a los
moles de ácido y alcohol que reaccionan, el cuadro de reacción queda de la siguiente forma:
( ) mol5,2
molg60
g150
Ácidon
1o =
⋅
=
−
( ) mol2
molg88
g176
Estern
1o =
⋅
=
−
16
Aplicando la ley de acción de masas al equilibrio:
( ) ( )
( ) ( )
( ) ( )
( ) ( ) ( ) ( )yxy5,2
yy
OHCnOHCn
OHnOHCn
V
OHCn
V
OHCn
V
OHn
V
OHCn
OHCOHC
OHOHC
K
62242
2284
62242
2284
62242
2284
c
−⋅−
⋅
=
⋅
⋅
=
⋅
⋅
=
⋅
⋅
=
Teniendo en cuenta que los moles en el equilibrio de C4H8O2 son 2 (y = 2), y que Kc = 5:
( )2x5,0
4
5
−⋅
=
Despejando se obtiene x = 3,6 mol de C2H6O. Conocidos los moles iniciales de etanol se calcula la masa.
( ) ( ) ( ) g6,165
mol
g
46mol6,3OHCMOHCnOHCm 626262 =⋅=⋅=
c. Conocidos los moles de todos los componentes en el equilibrio, se calculan las fracciones molares.
T
i
i
n
n
=χ ; 1,6226,15,0nT =+++=
( )
( )
0820,0
1,6
5,0
n
OHCn
OHC
T
242
242 ===χ ( )
( )
2623,0
1,6
6,1
n
OHCn
OHC
T
62
62 ===χ
( )
( )
3279,0
1,6
2
n
OHCn
OHC
T
284
284 ===χ ( )
( )
3279,0
1,6
2
n
OHn
OH
T
2
2 ===χ
Junio 2010. FG. Problema 2A. En un reactor se introducen 5 moles de tetraóxido de di nitrógeno gaseoso, que tiene
en el recipiente una densidad de 2,3 1
Lg −
⋅ . Este compuesto se descompone según la reacción NZO4 (g) ↔ 2 NO2 (g), y
en el equilibrio a 325 K la presión es 1 atm. Determine en estas condiciones:
a) El volumen del reactor.
b) El número de moles de cada componente en el equilibrio.
c) El valor de la constante de equilibrio Kp
d) El valor de la constante de equilibrio Kc
Datos. R = 0,082 11
KmolLatm −−
⋅⋅⋅ ; Masas atómicas: N = 14; O = 16
Puntuación máxima por 'apartado: 0,5 puntos.
Solución.
a. Aplicando la definición de densidad a las condiciones iniciales del reactor (solo hay N2O4), se obtiene el
volumen del reactor.
V
m
d = ;
( ) ( ) L200
Lg3,2
mol
g
92mol5
d
ONMONn
d
m
V 1
4242 =
⋅
⋅
=
⋅
== −
b. Si se define x como el número de moles de N2O4 que se disocian, el cuadro de reacción queda:
Las condiciones del sistema en el punto de equilibrio (V = 200 L; P = 1 atm; T = 298 K), permiten calcular el
número de moles totales.
mol5,7
K298
Kmol
Latm
082,0
L200atm1
TR
VP
nT =
⋅
⋅
=
⋅
⋅
=
( ) ( ) 5,7x5x2x5NOnONnn 242T =+=+−=+=
mol5,2x =
17
( ) mol5,25,25ONn 42 =−=
( ) mol55,22NOn 2 =⋅=
c.
42
2
ON
2
NO
p
P
p
K =
( ) ( )
( ) ( ) 33,1
333,0
666,0
K:
atm333,0
200
325082,05,2
V
RTNOn
ONP
atm666,0
200
325082,05
V
RTNOn
NOP 2
p
2
42
2
2
==





=
⋅⋅
=
⋅
=
=
⋅⋅
=
⋅
=
d.
( ) ( )
05,0
325082,0
33,1
RT
K
K 12n
p
c −∆
⋅
==
Junio 2010. FG. Cuestión 2B. Considerando el equilibrio existente entre el oxígeno molecular y el ozono, de
acuerdo a la reacción 3O2 (g) ↔ 2O3 (g), cuya entalpía de reacción ∆Hr = 284 kJ, justifique:
a) El efecto que tendría sobre el equilibrio un aumento de la presión del sistema.
b) El efecto que tendría sobre la cantidad de ozono en el equilibrio una disminución de la temperatura.
c) El efecto que tendría sobre el equilibrio la adición de un catalizador.
d) El efecto que tendría sobre la constante de equilibrio Kp añadir más ozono al sistema.
Puntuación máxima por apartado: 0,5 puntos.
Solución.
Según Le Chàtelier, si un sistema químico en equilibrio experimenta un cambio en la concentración,
temperatura, volumen, o la presión parcial, entonces el equilibrio se desplaza para contrarrestar el cambio impuesto y
restablecer el equilibrio.
a. Un aumento de presión tenderá a llevar al sistema hacia donde menos volumen ocupe y de esta forma
contrarrestar el aumento de presión, aplicado al equilibrio oxígeno/ozono, el sistema se desplazará hacia la derecha,
aumentando la concentración de ozono y disminuyendo la de oxígeno.
b. Al disminuir la temperatura el sistema evoluciona en el sentido en el que produzca calor (sentido exotérmico),
teniendo en cuenta que la reacción es endotérmica (∆H = 284 kJ > 0), el sentido exotérmico es hacia la izquierda,
aumentando la concentración de oxígeno y disminuyendo la de ozono.
c. Los Catalizadores son sustancias ajenas a una reacción cuya presencia modifica la velocidad de la misma sin
que la reacción experimenten alteración permanente alguna manteniendo las condiciones iniciales y finales, por lo tanto
su adición no perturba el equilibrio.
d. Las constantes de equilibrio solo son función de la temperatura, un aumento de la concentración de ozono
modificará el cociente de reacción obligando al sistema a evolucionar en contra de dicho aumento pero no modificara el
valor de Kp.
Modelo 2010. Cuestión 3A.- Dado el equilibrio C (s) + H2O (g) ↔ CO (g) + H2 (g), justifique si son verdaderas o
falsas las siguientes afirmaciones:
a) La expresión de la constante de equilibrio Kp es: Kp = p(CO) . p(H2) / { p(C) . p(H20) }
b) Al añadir más carbono, el equilibrio se desplaza hacia la derecha.
c) En esta reacción, el agua actúa como oxidante.
d) El equilibrio se desplaza hacia la izquierda cuando aumenta la presión total del sistema.
Puntuación máxima por apartado: 0,5 puntos.
Solución.
a. FALSA. En los equilibrios heterogéneos, la constante solo depende las sustancias que se encuentren en el
estado de agregación de mayor libertad, en el caso de equilibrio heterogéneos sólido-gas, las constantes de equilibrio
solo depende de las sustancias que estén en estado gaseoso.
OH
HCO
p
2
2
P
PP
K
⋅
=
18
b. FALSA. En un equilibrio heterogéneo sólido-gas, el equilibrio solo depende de las sustancias que se encuentren
en estado gaseoso.
c. VERDADERA. El hidrógeno el agua actúa con estado de oxidación +1, capta protones y se reduce a hidrógeno
molecular con estado de oxidación 0, actuando el agua como oxidante, mientras que el carbono sólido pierde electrones
oxidándose a monóxido de carbono y actuando como reductor.
d. VERDADERA. Según Le Chatelier, “Siempre que se modifiquen las condiciones de un sistema en equilibrio
se produce un desplazamiento del mismo en el sentido que restablezca la condiciones iniciales”. Al aumentar la presión
en un sistema gaseoso, el equilibrio se desplaza hacia donde menor volumen ocupe, oponiéndose de esta forma al
aumento de presión e intentar reestablecer el equilibrio.
Modelo 2010. Problema 1B.- Una mezcla de 2 moles de N2 y 6 moles de H2 se calienta hasta 700 ºC en un reactor de
100 L, estableciéndose el equilibrio N2 (g) + 3 H2 (g) ↔ 2NH3 (g). En estas condiciones se forman 48,28 g de amoniaco
en el reactor. Calcule:
a) La cantidad en gramos de N2 y de H2 en el equilibrio.
b) La constante de equilibrio Kc.
c) La presión total en el reactor cuando se ha alcanzado el equilibrio.
Datos. Masas atómicas: N = 14, H = 1; R = 0,082 atm·L·mol−1
·K−1
.
Puntuación máxima por apartado: a) y b) 0,75 puntos; c) 0,5 puntos.
Solución.
a. Para resolver el problema es conveniente plantear el cuadro de reacción. Si definimos x como el número de
moles de N2 que reaccionan, el cuadro queda de la siguiente forma:
Conocida la masa de amoniaco en el equilibrio (48,28 gr), se calcula x.
( ) ( )
( )
( )
84,2x2:
x2NHn
mol84,2
mol
gr17
gr28,48
NHM
NHm
NHn
eq3
3
3
eq3
=





=
===
: 42,1
2
84,2
x ==
Conocido el valor de x se calcula el número de moles en el equilibrio de N2 y de H2, y a continuación la masa en
gramos de cada uno.
( ) 58,042,12x2Nn 2 =−=−= : ( ) gr24,16
mol
gr28mol58,0MnNm 2 =⋅=⋅=
( ) 74,142,136x36Hn 2 =⋅−=−= : ( ) gr48,3
mol
gr2mol74,1MnHm 2 =⋅=⋅=
b. Conocidos los moles de todos los compuestos presentes en el equilibrio y el volumen del reactor, se calcula la
constante Kc mediante su definición.
( )
( ) ( )
4
3
2
3
22
2
3
3
22
2
3
c 1064,2
100
74,1
100
58,0
100
84,2
V
Hn
V
Nn
V
NHn
HN
NH
K ×=






⋅











=






⋅











=
⋅
=
c. Conocidos los moles en el equilibrio de todos los componentes de la mezcla gaseosa el volumen y la
temperatura, con la ecuación de gases ideales se calcula la presión en el interior del reactor.
( ) ( ) ( ) mol16,584,274,158,0NHnHnNnn 322T =++=++=
K973273700T =+=
TRnVP ⋅⋅=⋅ : atm12,4
L100
K973
Kmol
Latm
0,082mol16,5
V
TRn
P =
⋅
⋅
⋅
⋅
=
⋅⋅
=
19
Septiembre 2009. Cuestión 3.- En las siguientes comparaciones entre magnitudes termodinámicas y cinéticas
indique qué parte de la afirmación es falsa y qué parte es cierta:
a) En una reacción exotérmica tanto la entalpía de reacción como la energía de activación son negativas.
b) Las constantes de velocidad y de equilibrio son adimensionales.
c) Un aumento de temperatura siempre aumenta los valores de las constantes de velocidad y de equilibrio.
d) La presencia de catalizadores aumenta tanto la velocidad de reacción como la constante de equilibrio.
Puntuación máxima por apartado: 0,5 puntos.
Solución.
a. Verdadero en lo referente a la entalpía de reacción, falso en lo referente a la energía de activación. La energía de
activación de una reacción química siempre es positiva, no dependiendo de si la reacción es endotérmica o exotérmica,
mientras que la entalpía de una reacción exotérmica es negativa.
b. Verdadero en lo referente a la constante de equilibrio, falso en lo referente a la constante de velocidad. La
constante de equilibrio por definición es adimensional (cociente de concentraciones o de presiones), mientras que la
constante de velocidad tiene dimensiones que dependen del orden total de reacción (n).
( ) 11nn1
slmolvelocidadk −−−
⋅⋅=
c. Verdadero en lo referente a la constante de velocidad, falso en lo referente a la constante de equilibrio. Según
pone de manifiesto la ecuación de Arrhenius, la constante de velocidad es exponencialmente directa a la temperatura
RT
Ea
eAk
−
⋅= , si aumenta la temperatura, aumenta la constante. En el caso de la constante de equilibrio, la influencia de
la temperatura sobre la constante depende el signo de la entalpía de la reacción:
Si 0HR >∆ (Reacción endotérmica), si T aumenta, la constante aumenta
Si 0HR <∆ (Reacción exotérmica), si T aumenta, la constante disminuye
d. Verdadero en lo referente a la velocidad de reacción, falso en lo referente a la constante de equilibrio. El uso de
catalizadores positivos disminuye la energía de activación aumentando la velocidad de reacción, pero no afecta al
equilibrio, siendo la constante de equilibrio únicamente función de la temperatura.
Septiembre 2009. Problema 2A.- En el proceso Haber-Bosch se sintetiza amoniaco haciendo pasar corrientes de
nitrógeno e hidrógeno en proporciones 1:3 (estequiométricas) sobre un catalizador. Cuando dicho proceso se realiza a
500°C y 400 atm. se consume el 43 % de los reactivos, siendo el valor de la constante de equilibrio Kp=1,55×10−5
.
Determine, en las condiciones anteriores:
a) El volumen de hidrógeno necesario para la obtención de 1 tonelada de amoniaco puro.
b) La fracción molar de amoniaco obtenido.
c) La presión total necesaria para que se consuma el 60 % de los reactivos.
Datos. R = 0,082 atm·L·K−1
·mol−1
; Masas atómicas: N = 14, H = 1.
Puntuación máxima por apartado: a) y c) 0,75 puntos, b) 0,5 puntos.
Solución.
a. Si no es el número de moles iniciales N2 que se introducen en el reactor, y el nitrógeno e hidrógeno se
introducen en proporciones estequiométricas, 3no serán los moles de hidrógeno iniciales.
Si se denomina por α al tanto por uno de nitrógeno que reacciona, el cuadro de reacción queda de la siguiente
forma:
Conocida la cantidad de amoniaco que se quiere obtener y el porcentaje de reactivos que se consumen, se puede
obtener el número de moles iniciales de nitrógeno, y de este, los moles de hidrógeno iniciales.
( ) mol108,58
mol
g17
g101000
NHn 3
3
Eq3 ×=
×
=
( ) α= oEq3 n2NHn :
( )
2
3
3
Eq3
o Nmol104,68
43,02
108,58
2
NHn
n ×=
⋅
×
=
α
=
( ) ( ) mol102,205104,683Nn3Hn 33
o2o2 ×=×⋅=
20
Aplicando la ecuación de gases ideales, se calcula el volumen de hidrógeno necesario para obtener 1 tonelada de
amoniaco.
TRnVP ⋅⋅=⋅ : 33
3
m32,5L105,32
atm004
K773
Kmol
Latm
0,082mol102,205
P
TRn
V =×=
⋅
⋅
⋅
⋅×
=
⋅⋅
=
b.
( )
T
3
NH
n
NHn
3
=χ
( ) ( ) ( ) ( )α−=α−=α+α−+α−=++= 24nn2n4n2n3n3nnNHnHnNnn oooooooo322T
( )
274,0
43,02
43,0
224
2
24n
n2
o
o
NH3 −
=
α−
α
=
α−
α
=
α−
α
=χ
c. La presión se puede obtener a partir de Kp teniendo en cuenta que su valor no varía con la P, y que se puede
expresar en función de las fracciones molares y de la presión total del sistema.
3
HN
2
NH
23
H
3
N
2
NH
2
3
HN
2
NH
p
22
3
22
3
22
3
P
1
PP
P
PP
P
K
χ⋅χ
χ
=
χ⋅⋅χ⋅
χ⋅
=
⋅
=
3
HN
2
NH
p
22
3
K
1
P
χ⋅χ
χ
=
( )
( )
( )
( )
1428,0
6,024
6,01
24
1
24n
1n
24n
nn
n
Nn
o
o
o
oo
T
2
N2
=
⋅−
−
=
α−
α−
=
α−
α−
=
α−
α−
==χ
( )
( )
( )
( )
4286,0
6,024
6,033
24
33
24n
33n
24n
n3n3
n
Hn
o
o
o
oo
T
2
H2
=
⋅−
⋅−
=
α−
α−
=
α−
α−
=
α−
α−
==χ
( )
4286,0
6,02
6,0
224
2
24n
n2
o
o
NH3 −
=
α−
α
=
α−
α
=
α−
α
=χ
atm7,1026
4286,01428,0
4286,0
1055,1
1
P 3
2
5
=
⋅×
= −
Valor que confirma la Ley de Le Chatelier “Si la presión aumenta, el equilibrio se desplaza hacia la derecha,
debido a que como producto ocupa menor volumen”
Junio 2009. Cuestión 2.- Para la reacción: a A (g) ↔ B (g) + C (g), el coeficiente estequiométrico a podría tener los
valores 1, 2 ó 3. Indique de manera razonada el valor de a, los signos de las magnitudes termodinámicas ∆H0
, ∆S0
y
∆G0
, y el intervalo de temperatura en el que la reacción sería espontánea, para cada uno de los siguientes casos
particulares:
i) Caso A: La concentración de A en el equilibrio disminuye si aumenta la temperatura o la presión.
ii) Caso B: La concentración de A en el equilibrio aumenta si aumenta la temperatura o la presión.
Puntuación máxima por apartado: 1,0 punto.
Solución.
a A (g) ↔ B (g) + C (g)
i) Que la concentración de A disminuye respecto de la de equilibrio, indica que el sistema se desplaza a la
derecha (hacia productos).
Si se aumenta la temperatura, según Le Chatelier, los sistemas evolucionan consumiendo calor,
sentido endotérmico, por lo tanto en este caso el sentido endotérmico es hacia la derecha, la reacción es
endotérmica (∆∆∆∆H > 0).
Al aumentar la presión, el sistema se desplaza hacia donde menor volumen gaseoso ocupe, por lo
tanto, la reacción tendrá mayor volumen gaseoso en reactivos que en productos a = 3. 3A(g) ↔ B(g) +
C(g)
Si a = 3, el sistema estará pasando de 3 componentes gaseosos a dos, se ordena y la entropía
disminuye (∆∆∆∆S < 0).
21
Si ∆H > 0, ∆S < 0 y T > 0 (Temperatura absoluta), por la definición de energía libre ∆G = ∆H −
T∆S > 0 a cualquier temperatura. La reacción es no espontánea a cualquier temperatura.
ii) Si la concentración de A aumenta respecto a la de equilibrio, indica que el sistema se desplaza a la
izquierda (hacia reactivos).
Si al aumentar la temperatura el sistema se desplaza hacia la izquierda, el sentido endotérmico de
la reacción es hacia la izquierda, la reacción es exotérmica (∆∆∆∆H < 0).
Si al aumentar la presión es sistema se desplaza a la izquierda será porque en reactivos habrá
menor número de moles gaseosos que en productos, a = 1.
A(g) ↔ B(g) + C(g).
Si a = 1, el sistema está pasando de un componente gaseoso a dos, se desordena, aumenta la
entropía (∆∆∆∆S > 0).
Si ∆H < 0, ∆S > 0 y T > 0 (Temperatura absoluta), por la definición de energía libre ∆G = ∆H −
T∆S < 0 a cualquier temperatura. La reacción es espontánea a cualquier temperatura.
Junio 2009. Problema 2A.- El pentacloruro de fósforo se descompone con la temperatura dando tricloruro de fósforo
y cloro. Se introducen 20,85 g de pentacloruro de fósforo en un recipiente cerrado de 1 L, y se calientan a 250 °C hasta
alcanzar el equilibrio. A esa temperatura todas las especies están en estado gaseoso y la constante de equilibrio Kc vale
0,044.
a) Formule y ajuste la reacción química que tiene lugar
b) Obtenga la concentración en mol·L−1
de cada una de las especies de la mezcla gaseosa a esa temperatura
c) ¿Cuál será la presión en el interior del recipiente?
d) Obtenga la presión parcial de Cl2-
Datos. R = 0,082 atm-L·K−1
·mol−l
. Masas atómicas: P = 31,0; Cl = 35,5
Puntuación máxima por apartado: 0.5 puntos
Solución.
a. ( ) ( ) ( )gClgPClgPCl 235 +↔
b. Para resolver el ejercicio es conveniente plantear el siguiente cuadro de reacción, donde Co representa la
concentración inicial del pentacloruro de fósforo y x la concentración del mismo que se disocia
Aplicando la ley de acción de masas al equilibrio
xC
x
xC
xx
PCl
ClPCl
K
o
2
o5
23
c
−
=
−
⋅
=
⋅
=
ordenando se obtiene una ecuación de segundo grado.
0KCxKx ooc
2
=−+
La concentración inicial del pentacloruro (Co) se calcula a partir del número de moles iniciales y del volumen.
( )
( )
( )
M1,0
1
5,208
85,20
V
PClM
PClm
V
PCln
C 5
5
5o
o ===
Sustituyendo en la ecuación:



−=
=
=⋅−+
químicosentidoSin09,0x
048,0x
:0044,01,0x044,0x2
Concentraciones en el equilibrio:




==
=−=
M048,0ClPCl
M052,0048,01,0PCl
23
5
22
c. La presión en el interior del recipiente se calcula mediante la ecuación de estado de gases ideales.
V
TRn
P
⋅⋅
=
( ) ( ) ( ) mol148,01048,01048,01052,0VPClVPClVPClClnPClnPClnn 335235 =⋅+⋅+⋅=⋅+⋅+⋅=++=
atm3,6
1
523082,0148,0
P =
⋅⋅
=
d.
( )
atm1,2523082,0048,0TRCl
V
TRCln
P 2
2
=⋅⋅=⋅⋅=
⋅⋅
=
Modelo 2009. Cuestión 3.- Dada la reacción endotérmica para la obtención de hidrógeno
CH4 (g) ↔ C (s) + 2 H2 (g)
a) Escriba la expresión de la constante de equilibrio Kp.
b) Justifique cómo afecta un aumento de presión al valor de Kp.
c) Justifique cómo afecta una disminución de volumen a la cantidad de H2 obtenida.
d) Justifique cómo afecta un aumento de temperatura a la cantidad de H2 obtenida.
Puntuación máxima por apartado: 1,0 punto.
Solución.
a. Equilibrio heterogéneo sólido-gas. La definición de las constantes solo es función de los componentes del
equilibrio que estén en fase gaseosa..
4
2
CH
2
H
p
P
P
K =
b. Las constantes de equilibrio solo son función de la temperatura, por lo tanto un aumento de presión no afecta al
valor de Kp.
c. Una disminución de volumen equivale a un aumento de presión. Según Le Chatelier, al aumentar la presión el
equilibrio se desplaza en el sentido en el que ocupe menos volumen, en este caso hacia la izquierda (reactivos),
disminuyendo la producción de H2.
d. Reacción endotérmica (∆H>0), consume calor. Según Le Chatelier, al aumentar la temperatura en un sistema en
equilibrio, este se desplaza en el sentido en que consuma calor (sentido endotérmico), en este caso se desplaza
hacia la derecha (productos), aumentando la producción de H2.
Modelo 2009. Problema 1B.- Un recipiente de 37,5 L, que se encuentra a 343 K y 6 atm, contiene una mezcla en
equilibrio con el mismo número de moles de NO2 y N2O4, según la reacción
2 NO2 (g) ↔ N2O4 (g).
Determine:
a) El número de moles de cada componente en el equilibrio.
b) El valor de la constante de equilibrio Kp.
c) La fracción molar de cada uno de los componentes de la mezcla si la presión se reduce a la mitad.
Dato. R = 0,082 atm-L·K−1
·mol−l
Puntuación máxima por apartado: a) 0,5 puntos; b) y c) 0,75 puntos.
Solución.
a. Equilibrio homogéneo en fase gaseosa, mezcla binaria. El número de moles del sistema en equilibrio se
obtienen mediante la ecuación de gases ideales a partir de las de las condiciones de equilibrio (P, V y T).
TRnVP ⋅⋅=⋅ mol8
K343
Kmol
Latm
0,082
L37,5atm6
TR
VP
n ≈
⋅
⋅
⋅
⋅
=
⋅
⋅
=
Teniendo en cuenta que la mezcla es binaria y que hay igual número de moles de cada uno de los componentes:
23




=
=




=
=+
mol4n
mol4n
:
nn
8nn
2
2
422
422
NO
NO
ONNO
ONNO
b. La constante de equilibrio en función de las presiones viene dado por la expresión:
2
NO
ON
p
2
42
P
P
K =
Usando la ley de Raoult (Pi = P · χi), las presiones parciales se pueden expresar en función de la presión total y
de las fracciones molares.
( ) 2
NO
ON
2
NO
ON
2
NO
ON
p
2
42
2
42
2
42
PP
P
P
P
K
χ⋅
χ
=
χ⋅
χ⋅
==
Si el numero de moles de cada componente de la mezcla binaria es el mismo, las fracciones molares de cada
componente serán 0,5.
5,0
8
4
n
n
χχ
T
i
ONNO 422
====
33,0
5,06
5,0
P
K 22
NO
ON
p
2
42
=
⋅
=
χ⋅
χ
=
c. Teniendo en cuenta que Kp no varia con la presión y que las suma de las fracciones molares es la unidad, se
puede plantear un sistema para calcular las nuevas fracciones molares.
2
NOON
ONNO
2
NO
ON
ONNO
2
NO
ON
ONNO
2
NO
ON
p
242
422
2
42
422
2
42
422
2
42
:
1
1
1
3
33,0
1
P
K
χ=χ





=χ+χ
=
χ
χ
=





=χ+χ
χ⋅
χ
=
=





=χ+χ
χ⋅
χ
=



−=χ
=χ
=−χ+χ=χ+χ
62,1
62,0
:01:1
2
2
2222
NO
NO
NO
2
NO
2
NONO
La solución negativa no tiene sentido químico.
0,380,62110,62 2422 NOONNO =−=χ−=χ⇒=χ
Los resultado son concordantes con las leyes de Lavoisier, las cuales predicen que al disminuir la presión el
sistema se desplaza en el sentido en que aumente su volumen, en este caso a la izquierda (reactivos), aumentando la
fracción molar el dióxido de nitrógeno i disminuyendo las del tetraóxido de dinitrógeno.
Septiembre 2008. Problema 1B.-
El valor de la constante de equilibrio a 700 K para la reacción 2HI (g) ↔ H2 (g) + I2 (g) es 0,0183. Si se introducen 3,0
moles de HI en un recipiente de 5 L que estaba vacío y se deja alcanzar el equilibrio:
a) ¿Cuántos moles de I2 se forman?
b) ¿Cuál es la presión total?
c) ¿Cuál será la concentración de HI en el equilibrio si a la misma temperatura se aumenta el volumen al doble?
Datos. R = 0,082 atm·L·K−l
·mol−1
.
Puntuación máxima por apartado: a) 1 punto, b) y c) 0,5 puntos.
Solución.
a. Equilibrio homogéneo en fase gaseosa del que se conoce los moles iniciales de ioduro de hidrogeno y la
constante de equilibrio. Teniendo en cuenta la estequiometria de reacción, si 2x son los moles de ioduro de hidrógeno
disociado, x serán los moles de yodo e hidrógeno formados, el cuadro de reacción queda de la siguiente forma:
24
Según la ley de acción de masas: 2
22
HI
IH
K
⋅
=
Debido a que en el equilibrio no hay variación en el número de moles gaseosos entre reactivos y productos, la
constante se puede expresar en función del número de moles.
( ) ( )
( )
( ) ( )
( )( ) ( ) ( )
2
2
2
22
22
2
22
x23
x
x23
x
x23
xx
HIn
InHn
V
HIn
V
In
V
Hn
K 





−
=
−
=
−
⋅
=
⋅
=






⋅
=
Operando se despeja x
2
2
Idemoles32'0
0183,021
0183.03
K21
K3
xK
x23
x
x23
x
K =
+
=
+
=⇒=
−
⇒





−
=
b. La presión de equilibrio se calcula mediante la ecuación de gases ideales teniendo en cuenta que, en el proceso
de disociación no hay variación del número de moles entre productos y reactivos, y por tanto, el número de moles
gaseosos en el equilibrio coinciden con el número de moles iniciales.
atm44,34
L5
K700
Kmol
Latm
0,082mol3
V
TRn
V
TRn
P oEq
Eq =
⋅⋅
=
⋅⋅
=
⋅⋅
=
c. El equilibrio no se modifica por variaciones de presión o de volumen ya que ∆n(g) = 0, pero al aumentar el
volumen, disminuye la concentración.
( ) 1
o
Lmol236,0
52
32,023
V2
x23
V
HIn
HI −
=
⋅
⋅−
=
−
==
Junio 2008. Cuestión 3.- Considerando la reacción 2 SO2 (g) + O2 (g) ↔ 2 SO3 (g) razone si las siguientes
afirmaciones son verdaderas o falsas.
a) Un aumento de la presión conduce a una mayor producción de SO3.
b) Una vez alcanzado el equilibrio, dejan de reaccionar las moléculas de SO2 y O2 entre sí.
c) El valor de Kp es superior al de Kc, a temperatura ambiente.
d) La expresión de la constante de equilibrio en función de las presiones parciales es:
Kp = P2
(SO2)·P(O2)/P2
(SO3)
Dato. R = 0,082 atm·L·K−1
·mo1
Puntuación máxima por apartado: 0,5 puntos.
Solución.
a. VERDADERO. Según el principio de Le Chatelier, siempre que se modifiquen las condiciones de un sistema
en equilibrio se produce un desplazamiento del mismo en el sentido en el que se restablezcan las condiciones
iniciales. Sí se aumenta la presión, la reacción se desplaza en el sentido de originar aquellas sustancias que
ocupen menor volumen, es decir, el sistema se desplaza hacia donde el número de moles gaseosos sea menor,
en nuestra reacción hacia la derecha, aumentando la producción de SO3.
b. FALSO. El equilibrio químico es un equilibrio dinámico. Un sistema llega al equilibrio cuando las velocidades
de reacción directa e inversa se igual, ambas reacciones (directa e inversa) se siguen produciendo, pero las
concentraciones de la especies presente en el sistema no varían en el tiempo.
c. FALSO. Según la relación entre Kp y Kc:
( ) ( )gn
cp RTKK ∆
⋅=
∆n(g) = n(g)R − n(g)P = 2 − (2 + 1) = −1
( )
RT
K
K:RTKK c
p
1
cp =⋅= −
Teniendo en cuenta que RT > 0
cp KK <
25
d. FALSO. Teniendo en cuenta la ley de acción de masas aplicada a equilibrios gaseosos en función de las
presiones:
22
3
O
2
SO
2
SO
p
PP
P
K =
Septiembre 2008. Cuestión 3.- La reacción 2H2O (1) ↔ 2H2 (g) + O2 (g) no es espontánea a 25°C. Justifique si las
siguientes afirmaciones son verdaderas o falsas.
b. Se cumple que Kp/Kc = RT.
c. Si se duplica la presión de H2, a temperatura constante, el valor de Kp aumenta.
Puntuación máxima por apartado: 0’5 puntos.
Solución.
b. Falso. Las constantes de equilibrio Kc y Kp están relacionadas por la expresión:
( ) ( )gn
cp RTKK ∆
⋅=
Donde ∆n (g) es la diferencia del número de moles gaseosos entre productos y reactivos
( ) ( ) 3012)reactivos(n)productos(ngn gg =−+=−=∆ ∑∑
Sustituyendo en la expresión y ordenando:
( )3
cp RTKK ⋅= ⇒ ( )3
c
p
RT
K
K
=
c. Falso. La constante de equilibrio Kp no depende de las presiones parciales de los componentes, para cada
equilibrio depende únicamente de la temperatura.
Septiembre 2007. Problema 1B. En un recipiente de 25 L se introducen dos moles de hidrógeno, un mol de
nitrógeno y 3,2 moles de amoniaco. Cuando se alcanza el equilibrio a 400°C, el número de moles de amoniaco se ha
reducido a 1,8. Para la reacción 3H2 (g) + N2 (g) ↔ 2NH3 (g) calcule:
a) El número de moles de H2 y de N2 en el equilibrio.
b) Los valores de las constantes de equilibrio Kc y Kp a 400°C. Datos. R = 0,082 atm·L·mo1−1
·K−1
.
Puntuación máxima por apartado: 1’0 punto.
Solución.
a. Los datos del enunciado indican que la reacción se desplaza hacia la izquierda para alcanzar el equilibrio, cosa
que ocurre cuando el cociente de reacción Q > KEq.
Si se supone que desaparecen 2x moles de amoniaco, y teniendo en cuenta la estequiometria de la reacción, el
cuadro de reacción queda de la siguiente forma:
El número de moles de amoniaco en el equilibrio es un dato:
( ) 8'1x22'3NHn Eq3 =−=
Expresión de la que se puede despejar x, y de esta forma calcular los moles en el equilibrio de N2 e H2.
moles7'0
2
8'12'3
x =
−
=
( ) moles7'17'01Nn 2 =+= ( ) moles1'47'032Hn 2 =⋅+=
b. Según la ley de acción de masas la constante de equilibrio en función de las concentraciones para la síntesis del
amoniaco es:
26
( )
( ) ( )
28'17
25
1'4
25
7'1
25
8'1
V
Hn
V
Nn
V
NHn
HN
NH
K
3
2
3
eq2eq2
2
eq3
3
22
2
3
c =






⋅











=








⋅
















=
⋅
=
Kp se calcula a partir de la relación entre Kc y Kp.
( ) ( )gn
cp RTKK ∆
⋅=
Donde ∆n (g) es la diferencia del número de moles gaseosos entre productos y reactivos
( ) ( ) 2312)reactivos(n)productos(ngn gg −=+−=−=∆ ∑∑
( ) ( ) 322
cp 1067'5673082'028'17RTKK −−−
×=⋅⋅=⋅=
Junio 2007. Problema 1B.- A temperatura elevada, un mol de etano se mezcla con un mol de vapor de ácido nítrico,
que reaccionan para formar nitroetano (CH3CH2N02) gas y vapor de agua. A esa temperatura, la constante de equilibrio
de dicha reacción es Kc = 0,050.
a) Formule la reacción que tiene lugar.
b) Calcule la masa de nitroetano que se forma.
c) Calcule la entalpía molar estándar de la reacción.
d) Determine el calor que se desprende o absorbe hasta alcanzar el equilibrio.
Datos. Masas atómicas: H = 1, C = 12, N = 14, 0=16.
Etano (g) Ác. nítrico (g) Nitroetano (g) Agua (g)
∆Hºf (kJmol−l
) −124,6 −164,5 −236,2 −285,8
Puntuación máxima por apartado: 0.5 puntos.
Solución.
a) ( ) ( ) ( ) ( )gOHgNOCHCHgHNOgCHCH 2223333 +−↔+−
b) Para calcular la masa de nitroetano en el equilibrio se plantea el cuadro de reacción y se aplican las
condiciones de equilibrio a la constante Kc.
Sea x los moles de etano que reaccionan:
333
2223
333
2223
HNOCHCH
OHNOCHCH
HNOCHCH
OHNOCHCH
333
2223
c
nn
nn
V
n
V
n
V
n
V
n
HNOCHCH
OHNOCHCH
K
⋅
⋅
=
⋅
⋅
=
⋅
⋅
=
El equilibrio no depende del volumen. Sustituyendo los valores en el equilibrio, se despeja x (número de moles
de etano que han reaccionado que coincide con los moles de nitroetano formados)
( ) ( ) ( )2
2
x1
x
x1x1
xx
050'0
−
=
−⋅−
⋅
=
Operando se despeja x:
18'0
05'01
05'0
x;05'0
x1
x
;050'0
x1
x
2
=
+
==
−
=





−
NOTA: Solo tiene sentido químico el valor positivo de la raíz.
Conocidos los moles en el equilibrio, se calcula la masa.
( ) ( ) ( ) gr5'13
mol
gr75mol18'0NOCHCHMNOCHCHnNOCHCHm 223223223 =⋅=⋅=
27
c) Por tratarse la entalpía de una función de estado, sus variaciones solo depende de las condiciones iniciales y
finales, por lo tanto ∆HR se calcula mediante la ley de Hess.
( ) ( )∑∑ ∆−∆=∆ ReactivosHProductosHH o
f
o
f
o
R
( ) ( ) ( ) ( )( )3
o
f33
o
f2
o
f223
o
f
o
R HNOHCHCHHOHHNOCHCHHH ∆+∆−∆+∆=∆
( ) ( )( ) 1o
R molkJ9'2325'1646'1248'2852'236H −
−=−−−−−+−=∆
Reacción EXOTÉRMICA.
d) o
RHnQ ∆⋅=∆ Donde n es el número de moles de etano que han reaccionado
( ) kJ9'41molkJ9'232mol18'0Q 1
−=−⋅=∆ −
Hasta alcanzar el equilibrio, se desprende 41’9 kJ.
Modelo 2007. Cuestión 3.- El cloruro de plata (I) es una sal muy insoluble en agua.
a) Formule el equilibrio heterogéneo de disociación.
b) Escriba la expresión de la constante del equilibrio de solubilidad (Ks) y su relación con la solubilidad molar (s).
c) Dado que la solubilidad aumenta con la temperatura, justifique si el proceso de disolución es endotérmico o
exotérmico.
d) Razone si el cloruro de plata (I) se disuelve más o menos cuando en el agua hay cloruro de sodio en disolución.
Puntuación máxima por apartado: 0,5 puntos.
Solución.
a) ( ) ( ) ( )aqClaqAgsAgCl −+
+↔
b) La constante de equilibrio solo depende de las concentraciones de los componentes que estén en el estado de
agregación de mayor libertad. En el equilibrio sólido/líquido, la constante (Ks, constante ó producto de
solubilidad) solo depende de las concentraciones de los que estén en fase líquida o disuelta (acuosa).
−+
⋅= ClAgKs
Si se define s como la cantidad de moles por litro que se disuelven de la sal AgCl, por estequiometria,
se formaran s moles por litro de Ag+
y Cl−
.
Sustituyendo las concentraciones en la expresión de la constante:
2
s sssK =⋅=
c) El aumento de la solubilidad indica que el equilibrio se desplaza hacia la derecha (productos), Si al aumentar la
temperatura el equilibrio se desplaza hacia la derecha es porque la reacción es endotérmica (∆H>0, absorbe
calor).
d) Se puede explicar de dos formas: Por leyes de equilibrio, al aumentar la concentración de un producto (Cl−
). El
equilibrio se desplaza hacia la izquierda, disminuyendo la solubilidad de la sal. Efecto ión común
Modelo 2007. Problema 2B.- A 400 ºC y 1 atmósfera de presión el amoniaco se encuentra disociado en un 40%, en
nitrógeno e hidrógeno gaseosos, según la reacción ( ) ( ) ( )gN1/2gH3/2gNH 223 +↔ . Calcule:
a) La presión parcial de cada uno de los gases en el equilibrio.
b) El volumen de la mezcla si se parte de 170 g de amoniaco.
c) El valor de la constante Kp.
d) El valor de la constante Kc.
Datos.- -1-1
KmolLatm0,082R ⋅⋅⋅= ; masas atómicas: N = 14, H = 1
Puntuación máxima por apartado: 0,5 puntos.
Solución.
a) Según la ley Raoult, la presión parcial de un componente de una mezcla gaseosa viene dada por la expresión:
ii PP χ⋅=
Donde P es la presión total, y iχ es la fracción molar del componente i.
28
T
i
i
n
n
=χ
Para conocer el número de moles de cada especie en el equilibrio, se hace el siguiente cuadro, teniendo
en cuenta las relaciones estequiométricas y, definiendo no como el número de moles iniciales y α como el grado
de disociación.
o
Disociados
n
n
=α
Según el cuadro, el número de moles totales es:
( ) ( ) ( ) ( )α+=α+=α+α+α−=++= 1nnnn
2
3
n
2
1
nnHnNnNHnn ooooooo223T
Conocido el número de moles totales y los moles de cada componente, se calculan las fracciones
molares.
( )
( )
( )
( )
( )
( )
( ) ( )
( )
( ) ( )α+
α
=
α+
α
=χ
α+
α
=
α+
α
=χ
α+
α−
=
α+
α−
=χ
12
3
1n
n
2
3
H:
121n
n
2
1
N:
1
1
1n
1n
NH
o
o
2
o
o
2
o
o
3
Para α = 0’40:
( ) ( )
( )
( )
( )
( )
( )
43'0
4'012
4'03
H:0'14
4'012
4'0
N:0'43
4'01
4'01
NH 223 =
+
⋅
==χ=
+
=χ=
+
−
=χ
Con las fracciones molares y la presión total del sistema, se calculan las presiones parciales de cada
componente en la mezcla.
atm43'043'0atm1PP
atm14'014'0atm1PP
atm43'043'0atm1PP
22
22
33
HH
NN
NHNH
=⋅=χ⋅=
=⋅=χ⋅=
=⋅=χ⋅=
b) El volumen de la mezcla si se parte de 170 g de amoniaco.A partir de la ecuación de gases ideales:
P
nRT
V =
Donde n es el número total de moles de la mezcla en equilibrio, que se calcula a partir del número de
moles iniciales de amoniaco según la expresión:.
( )
( )
( )
( ) ( ) mol1440'01
mol
gr17
gr170
1
NHM
NHm
1nn
3
3o
oT =+=α+=α+=
Aplicando la ecuación de gases ideales:
( )
L6'772
atm1
K673
Kmol
Latm
082'0mol14
V =
⋅
⋅
⋅
⋅
=
c) Para la reacción ( ) ( ) ( )gN1/2gH3/2gNH 223 +↔ , se define la constante de equilibrio en función de las
presiones (Kp) como: 245'0
43'0
14'043'0
P
PP
K
2
1
2
3
NH
2
1
N
2
3
H
p
3
22
=
⋅
=
⋅
=
d) Kc y Kp se relacionan por la siguiente expresión:
29
( ) ( )gn
pc RTKK ∆−
⋅=
Donde ∆n(g) es la diferencia de numero de moles gaseosos entre productos y reactivos.
( ) 11
2
1
2
3
gn =−+=∆
( ) 31
c 104'4673082'0245'0K −−
×=⋅⋅=
Septiembre 2006. Cuestión 3.- El amoniaco reacciona a 298 K con oxígeno molecular y se oxida a monóxido de
nitrógeno y agua, siendo su entalpía de reacción negativa.
a) Formule la ecuación química correspondiente con coeficientes estequiométricos enteros.
b) Escriba la expresión de la constante de equilibrio Kc.
c) Razone cómo se modificará el equilibrio al aumentar la presión total a 298 K si son todos los compuestos
gaseosos a excepción del H2O que se encuentra en estado líquido.
d) Explique razonadamente cómo se podría aumentar el valor de la constante de equilibrio.
Puntuación máxima por apartado: 0’5 puntos.
Solución.
a.
( ) ( ) ( ) ( )lOH6gNO4gO5gNH4 223 +↔+
b. Por tratarse de un equilibrio heterogéneo (líquido-gas), la constante de equilibrio solo es función de los
componentes en estado gaseoso.
5
2
4
3
4
ONH
NO
Kc =
c. Según Le Chatelier al aumentar la presión, la reacción se desplaza en el sentido de originar aquellas sustancias
que ocupen menos volumen. En la reacción propuesta, al aumentar la presión el equilibrio se desplaza hacia
PRODUCTOS (derecha), por haber nueve moles gaseosos en reactivos y cuatro moles gaseosos en productos.
d. Por ser una reacción exotérmica (∆H<0, desprende calor), al disminuir la temperatura el equilibrio se desplaza
en el sentido en el que se desprenda calor, hacia productos (derecha),
( ) ( ) ( ) ( ) QlOH6gNO4gO5gNH4 223 ++↔+
aumentando las concentraciones de productos y disminuyendo la de reactivos, y por lo tanto aumentado el valor de la
constante de equilibrio.
Junio 2006. Problema 1B.- En un recipiente de 0,4 L se introduce 1 mol de N2 y 3 mol de H2 a la temperatura de
780 K. Cuando se establece el equilibrio para la reacción N2 + 3 H2 ↔ 2 NH3, se tiene una mezcla con un 28 % en mol
de NH3. Datos.- R = 0,082 atm·L·K−1
·mol−1
.
Determine:
a) El número de moles de cada componente en el equilibrio.
b) La presión final del sistema.
c) El valor de la constante de equilibrio, Kp.
Datos.- R = 0,082 atm·L·K−1
·mol−1
.
Puntuación máxima por apartado: a) y c) 0,75 puntos y b) 0,5 puntos.
Solución.
a. Para el equilibrio de formación del amoniaco, conocidos los moles iniciales de hidrógeno y nitrógeno y la
fracción molar del amoniaco en la mezcla resultante, se pide calcular la composición en moles de la mezcla en equilibrio.
Si definimos x como los moles de hidrógeno que reaccionan, el cuadro de reacción es el siguiente:
30
( ) ( )
( ) ( ) ( ) ( ) ( ) ( )
28'0
x24
x2
x2x33x1
x2
NHnHnNn
NHn
n
NHn
322
3
T
3
NH3
=
−
=
+−+−
=
++
==χ
Despejando: x = 0’44
( )
( )
( ) 88'044'02x2NHn
68'144'033x33Hn
56'044'01x1Nn
eq3
eq2
eq2
=⋅==
=⋅−=−=
=−=−=
b. Con el número total de moles el volumen y la temperatura, mediante la ecuación de gases ideales se calcula la
presión en el equilibrio.
moles12'344'024x24nT =⋅−=−=
atm499
L4'0
K780
Kmol
Latm
082'0mol12'3
V
nRT
P ≈
⋅
⋅
⋅
⋅
==
c. Para el equilibrio de formación el NH3 se define Kp como:
3
HN
2
NH
p
22
3
PP
P
K
⋅
=
Teniendo en cuenta la ley de Raoult ( )ii PP χ⋅= , se puede expresar Kp en función de la presión total y de las
fracciones molares de los componentes gaseosos del equilibrio.
3
HN
2
NH
23
H
3
N
2
NH
2
p
22
3
22
3
P
1
PP
P
K
χ⋅χ
χ
=
χ⋅⋅χ⋅
χ⋅
=
( ) ( )
54'0
12'3
68'1
n
Hn
18'0
12'3
56'0
n
Nn
28'0
T
2
H
T
2
NNH 223
===χ===χ=χ
5
3
2
23
HN
2
NH
2p 101'1
54'018'0
28'0
500
1
P
1
K
22
3 −
×=
⋅
⋅=
χ⋅χ
χ
=
Modelo 2006. Cuestión 3.- Al calentar, el dióxido de nitrógeno se disocia en fase gaseosa en monóxido de nitrógeno
y oxígeno:
a) Formule la reacción química que tiene lugar.
b) Escriba Kp para esta reacción.
c) Explique el efecto que produce un aumento de presión total sobre el equilibrio.
d) Explique como se verá afectada la constante de equilibrio al aumentar la temperatura.
Puntuación máxima por apartado: 0,5 puntos.
Solución.
a. ( ) ( ) ( )gO
2
1
gNOgNO 22 +⇔
b.
2
2
1
ONO
NO1
PP
Kp 2
⋅
=
c. Al aumentar la presión en un sistema en equilibrio, este se desplaza hacia donde menor volumen ocupe, y de esa
forma se contrarresta el aumento de presión, en este caso, el sistema se desplaza hacia los reactivos(izquierda).
d. Al aumentar la temperatura se ve favorecido el proceso endotérmico y el equilibrio se desplaza hacia la derecha
ya que la reacción es endotérmica ( ( )0H >∆ , aumentando el valor de Kp.
31
Modelo 2006. Problema 2B.- Se introduce en un recipiente de 3 L, en el que previamente se ha hecho el vacío, 0’04
moles de SO3 a 900 K. Una vez alcanzado el equilibrio, se encuentra que hay presentes 0,028 moles de SO3.
a) Calcule el valor de Kc para la reacción: 2SO3 (g) ↔2SO2 (g) + O2 (g) a dicha temperatura.
b) Calcule el valor de Kp para dicha disociación.
Dato.- R = 0,082 atm • L • mol-1
K-1
Puntuación máxima por apartado: 1,0 punto.
Solución.
a.
mol0'006xmol012'0028'004'0x2 =⇒=−=
conocido el valor de x se conocen el número de moles de cada especie presente en el equilibrio.
n(SO3) = 0’028
n(SO2) = 0’012
n(O2) = 0’006
La expresión de Kc se puede poner en función del número de moles y del volumen
( ) ( )
( ) 2
3
2
2
2
2
2
3
2
2
2
c
V
SOn
V
On
V
SOn
SO
OSO
K






⋅





=
⋅
=
Simplificando volúmenes se obtiene la siguiente expresión:
( ) ( )
( ) ( )2
2
3
2
22
2
c
028'0
006'0012'0
3
1
SOn
OnSOn
V
1
K
⋅
⋅=
⋅
⋅=
4
c 107'3K −
⋅=
b.
( ) ( )
( ) ( )
( )RTKcKp
1212gn
RTKcKp gn
⋅=




=−+=∆
⋅= ∆
( ) 027'0900082'01067'3Kp 4
=⋅⋅= −
Septiembre 2005. Problema 2A. Para la reacción N2 (g) + O2 (g) → 2 NO (g) el valor de la constante de equilibrio,
Kc, es 8,8×10−4
a 1930 °C. Si se introducen 2 moles de N2 y 1 mol de O2 en un recipiente vacío de 2 L y se calienta hasta
1930 °C, calcule:
a) La concentración de cada una de las especies en equilibrio.
b) La presión parcial de cada especie y el valor de la constante de equilibrio Kp.
Datos.- R = 0’082 atm·L·mo1−1
·K−1
Puntuación máxima por apartado: 1,0 punto.
Solución.
a. Para la reacción de formación del monóxido de nitrógeno se plantea el cuadro de reacción.
A partir de la ley de Acción de Masas se obtiene la expresión de la constante que rige el equilibrio en función
del número de moles.
32
( )
( ) ( )
( )
( ) ( )
( )
( ) ( )
4
2
22
22
2
22
2
22
2
c 108'8
2x3x
x4
x1x2
x2
OnNn
NOn
v
On
v
Nn
v
NOn
ON
NO
K −
×=
+−
=
−⋅−
=
⋅
=
⋅






=
⋅
=
Expresión que se puede transformar en una ecuación de segundo grado.
( ) 01076'1x1064'2x108'84 3324
=×−×+×− −−−
despreciando 8’8×10−4
frente a 4, la ecuación queda de la forma:
01076'1x1064'2x4 332
=×−×+ −−
cuyas soluciones son:



−=
=
02'0x
02'0x
despreciando la solución negativa por carecer de sentido químico, el número de moles en el equilibrio queda de la
siguiente forma
Conocidos los moles en el equilibrio y el volumen se calculan las concentraciones en el equilibrio.
( )
l
mol99'0
2
98'1
v
Nn
N 2
2 ===
( )
l
mol49'0
2
98'0
v
On
O 2
2 ===
( )
l
mol02'0
2
04'0
v
On
NO 2
===
b. Partiendo de la ecuación de gases ideales aplicada a cada componente del equilibrio, se obtienen sus presiones
parciales.
RTnVP ii =⋅ : RT
V
n
P i
i = : RTCP ii =
( ) atm9'178K2203
Kmol
latm
082'0
l
mol
99'0RTNP 2N2
=⋅





⋅
⋅
⋅





=⋅=
( ) atm5'88K2203
Kmol
latm
082'0
l
mol
49'0RTOP 2O2
=⋅





⋅
⋅
⋅





=⋅=
( ) atm6'3K2203
Kmol
latm
082'0
l
mol
02'0RTNOPNO =⋅





⋅
⋅
⋅





=⋅=
Para calcular el valor de Kp, se tiene en cuenta la relación entre las constantes Kp y Kc
( ) ( ) 4
c
0
c
n
cp 108'8KRTKRTKK −∆
×==⋅=⋅=
Junio 2005. Cuestión 3.- El dióxido de nitrógeno es un gas que se presenta en la forma monómera a 100 ºC. Cuando
se disminuye la temperatura del reactor hasta 0 ºC se dimeriza para dar tetraóxido de dinitrógeno gaseoso.
a) Formule el equilibrio químico correspondiente a la reacción de dimerización.
b) ¿Es exotérmica o endotérmica la reacción de dimerización?
c) Explique el efecto que produce sobre el equilibrio una disminución del volumen del reactor a temperatura
constante.
d) Explique cómo se verá afectado el equilibrio si disminuye la presión total, a temperatura constante.
Puntuación máxima por apartado: 0,5 puntos.
33
Solución.
a. ( ) ( )gONgNO2 422 ↔
b. Al disminuir la temperatura el equilibrio se desplaza hacia la derecha, lo cual, indica que se absorbe calor en
sentido contrario. La reacción es EXOTÉRMICA. 0HR <∆
( ) ( ) QgONgNO2 422 +↔
c. Al disminuir el volumen del reactor a temperatura constante aumenta la presión del sistema. y el equilibrio se
desplaza hacia donde menor volumen ocupa, hacia la derecha, favoreciendo la dimerización.
d. Si disminuye la presión, el equilibrio se desplaza hacia donde más volumen ocupa, hacia la izquierda. No
favorece la dimerización
Junio 2005. Problema 2B.- Se introducen 2 moles de COBr2 en un recipiente de 2 L y se calienta hasta 73 ºC. El
valor de la constante Kc, a esa temperatura, para el equilibrio COBr2(g) ⇔ CO(g) + Br2(g) es 0,09. Calcule en dichas
condiciones:
a) El número de moles de las tres sustancias en el equilibrio.
b) La presión total del sistema.
c) El valor de la constante Kp
Dato.- R = 0,082 atm · L · mol-1· K-1
Puntuación máxima por apartado: a) 1,0 punto; b) y c) 0,5 puntos.
Solución.
a.
A partir de la expresión de la constante de equilibrio en función de las concentraciones, se puede encontrar una
relación entre la constante y el número de moles de cada especie en equilibrio.
( ) ( )
( )
( ) ( )
( )2
2
2
2
2
2
c
COBrn
BrnCOn
V
1
V
COBrn
V
Brn
V
COn
COBr
BrCO
K
⋅
⋅=
⋅
=
⋅
=
sustituyendo por los datos del equilibrio
18'0
x2
x
x2
xx
2
1
09'0K
2
c =
−−
⋅
⋅==
ordenando



−=
=
=−+
7'0x
52'0x
:036'0x18'0x2
Se desprecia la solución negativa por no tener sentido químico, por lo que en el equilibrio queda:
( ) ( )
( ) moles48'152'02COBrn
moles52'0BrnCOn
2
2
=−=
==
b. Se obtiene a partir de la ecuación de gases ideales.
V
TRn
PTRnVP T ⋅⋅
=⋅⋅=⋅
( ) atm7'35
2
346082'052'0248'1
P =
⋅⋅⋅+
=
c. Se obtiene a partir de la relación entre Kp y Kc.
( ) ( )
( )( )
55'2346082'009'0RTKK 111gn
cp =⋅⋅=⋅= −+∆
Modelo 2005. Cuestión 3.- Para la reacción de síntesis del amoníaco, N2{g) + 3H2{g) ↔ 2NH3{g), se conocen los
34
valores, a temperatura ambiente, de las siguientes magnitudes: o
rH∆ (valor negativo), o
rG∆ (valor negativo), Kp (valor
muy alto) y Ea (valor muy alto). Conteste a las siguientes preguntas, indicando cuál o cuáles de dichas magnitudes están
directamente relacionadas con los conceptos que se enumeran a continuación:
Puntuación máxima por apartado:0,5 puntos.
d. Efecto de la presión. ¿Qué efecto tiene para esta reacción un aumento de presión?
Solución.
d. Relacionada con la constante del equilibrio a través del cociente de reacción y por tanto de la estequiometria. Si
aumenta la presión el cociente de reacción disminuye y por tanto el sistema se desplaza hacia la derecha para restablecer
el valor de la constante de equilibrio.
3
HN
2
NH
23
HN
2
NH
p
22
3
22
3
χχ
χ
P
1
PP
P
K
⋅
⋅=
⋅
=
Modelo 2005. Problema 1B.- En un recipiente cerrado, a la temperatura de 490 K, se introduce 1 mol de PCl5(g) que
se descompone parcialmente según la reacción PCl5(g) ↔ PCl3 (g) + Cl2 (g). Cuando se alcanza el equilibrio, la presión
es de 1 atm y la mezcla es equimolecular (igual número de moles de PCl5, PCl3 y Cl2).
a) Determine el valor de la constante de equilibrio, Kp, a dicha temperatura.
b) Si la mezcla se comprime hasta 10 atm, calcule la nueva composición de equilibrio.
Puntuación máxima por apartado: 1,0 punto.
Solución.
a.
Teniendo en cuenta que la mezcla es equimolecular:
( ) ( ) ( )235 ClnPClnPCln ==
moles0'5xxx1 ==−
Si el número de moles de los tres componentes del equilibrio son iguales, sus fracciones molares también lo
serán. Teniendo en cuenta además que la suma de todas las fracciones molares de una mezcla gaseosa es 1, se puede
concluir que en este equilibrio la fracción molar de cada componente es de un tercio
3
1
235 ClPClPCl =χ=χ=χ
033'
3
1
P
P
PP
P
PP
K
5
23
5
23
5
23
PCl
ClPCl
PCl
ClPCl
PCl
ClPCl
p ==
χ
χ⋅χ
=
χ⋅
χ⋅⋅χ⋅
=
⋅
=
Solución.
b.
La expresión de la constante Kp para la reacción de disociación del pentacloruro de fósforo es:
5
23
5
23
5
23
PCl
ClPCl
PCl
ClPCl
PCl
ClPCl
p P
P
PP
P
PP
K
χ
χχ
⋅=
χ⋅
χ⋅⋅χ⋅
=
⋅
=
El número total de moles en el equilibrio es:
( )α−⋅=α+=α+α+α−= 1nn:nnnnnnn oTooooooT
Conocido el número de moles totales se calculan las fracciones molares en función de α.
35
( ) ( )
( )
( ) ( )
( )





α+
α
=
α+
α
===χ=χ
α+
α−
=
α+
α−
==χ
=χ
11n
n
n
Cln
n
PCln
1
1
1n
1n
n
PCln
:
n
n
o
o
T
2
T
3
ClPCl
o
o
T
5
PCl
T
i
i
23
5
Sustituyendo las fracciones molares en Kp, se obtiene la expresión buscada:
2
2
PCl
ClPCl
p
1
P
1
1
11PPK
5
23
α−
α
⋅=
α+
α−
α+
α
⋅
α+
α
⋅=
χ
χχ
⋅=
expresión que relaciona Kp α y P. Teniendo en cuenta que Kp no depende de la presión y que por tanto permanece
invariable si se aumenta la presión, se puede calcular el nuevo α.
18'0:
1
1033'0 2
2
=α
α−
α
⋅=
Conocido α se calculan las fracciones molares en el nuevo equilibrio
15'0
1
7'0
18'01
18'01
1
1
23
5
ClPCl
PCl
=
α+
α
=χ=χ
=
+
−
=
α+
α−
=χ
Septiembre 2004. Cuestión 1.- La reacción de obtención del polietileno a partir de eteno,
[ ] )s(CHCH)g(CHCHn n2222 −−−⇔=
es exotérmica:
a) Escriba la expresión de la constante de equilibrio KP.
b) ¿Qué tipo de polimerización se produce?
c) ¿Cómo afecta un aumento de la temperatura a la obtención de polietileno?
d) ¿Cómo afecta un aumento de la presión total del sistema a la obtención de polietileno?
Puntuación máxima por apartado: 0’5 puntos.
Solución.
a. Teniendo en cuenta que es un equilibrio heterogéneo (sólido /gas) la constante es función únicamente de los
componentes de equilibrio que estén en fase gas, y por tanto su expresión es:
n
22 CHCH
1
Kp
=
=
b. Reacción de polimerización por adición.
c. Teniendo en cuenta que la reacción es exotérmica, desprende calor, al aumentar la temperatura el equilibrio se
desplazará en el sentido endotérmico, consumiendo calor y contrarrestando el aumento de temperatura, como la reacción
es exotérmica, se desplazará hacia la izquierda (reactivos) disminuyendo la obtención de polietileno.
d. Favorece la obtención de acetileno ya que al aumentar la presión el sistema se desplaza hacia donde menor
volumen ocupa, contrarrestando el aumento de presión, en este caso hacia la derecha (productos).
Septiembre 2004. Problema 2A. En un reactor de 1 L, a temperatura constante, se establece el equilibrio
NO2 + SO2 ⇔ NO + SO3
siendo las concentraciones molares en l equilibrio: 2'0NO2 = , 6'0SO2 = , 0'4NO = y 2'1SO3 = .
a) Calcular el valor de KC a esa temperatura.
b) Si se añaden 0’4 moles de NO2 ¿Cuál será la nueva concentración de reactivos y productos cuando sé
reestablezca de nuevo el equilibrio?
Puntuación máxima por apartado: 1,0 punto.
Solución.
a. Según la ley de acción de masas, la constante del equilibrio tiene la siguiente expresión.
36
22
3
c
SO·NO
SO·NO
K =
Sustituyendo los valores de las concentraciones en equilibrio:
40
6'02'0
2'14
Kc =
⋅
⋅
=
b.
Sustituyendo en la expresión de la constante:
( )( )
( )( )
40
x6'0·x6'0
x2'1·x4
Ke =
−−
++
= ; 40
36'0x2'1x
8'4x2'5x
2
2
=
+−
++
ordenando como una ecuación de segundo grado y resolviendo:



=
=
=++
21'0x
15'1x
06'9x2'53x39 2
despreciando la solución x = 1’15 por ser mayor que las concentraciones de partida, las concentraciones en equilibrio en
(ml/l) son:
Junio 2004. Problema 1B. El yoduro de hidrógeno se descompone a 400 ºC de acuerdo con la ecuación
( ) )g(IgH)g(HI2 22 +↔ , siendo el valor de Kc = 0’0156. Una muestra de 0’6 moles de HI se introduce en un matraz de
1 L y parte del HI se descompone hasta que el sistema alcanza el equilibrio.
a) ¿Cuál es la concentración de cada especie en el equilibrio?
b) Calcule KP.
c) Calcule la presión total en el equilibrio.
Puntuación máxima por apartado: a) y c) 0’75 puntos; b) 0’5 puntos.
Solución.
a. Se pide estudiar el equilibrio de disociación del yoduro de hidrógeno conocida la constante de equilibrio, las
variables de estado(P, V, T) y la concentración inicial.
La constante de equilibrio, según la Ley de Acción de Masas tiene la expresión:
2
22
c
HI
IH
K
⋅
=
por no haber variación en el numero de moles, la constante se puede expresar en función del número de moles en el
equilibrio.
2
HI
IH
2
HI
IH
2
22
c
n
nn
V
n
V
n
V
n
HI
IH
K 22
22
⋅
=






⋅
=
⋅
=
sustituyendo los datos de la tabla
( ) ( )
2
2
2
22
HI
IH
c
x26'0
x
x26'0
x
x26'0
xx
n
nn
K 22






−
=
−
=
−
⋅
=
⋅
=
expresión de la que se puede despejar x en función de Kc
37
06'0
0156'021
0156'06'0
K21
K6'0
x
x26'0
x
K
c
c
c =
+
⋅
=
+
⋅
=⇒
−
=
conocido el valor de x, se puede calcular el número de moles en el equilibrio, y conocido el volumen, la concentración de
cada especie en el equilibrio.
l
mol06'0
1
06'0
V
n
IH06'0nn
l
mol48'0
1
48'0
V
n
HI48'006'026'0n
22IH
HI
HI
22
====⇒==
===⇒=⋅−=
b. Teniendo en cuenta que no hay variación en el número de moles de reactivos a productos(∆n=0), y que:
( ) 0156'0KKRTKK cp
n
cp ==⇒⋅= ∆
c. La presión total se calcula mediante la ecuación de estado de gases ideales utilizando el número total de moles
presentes en el equilibrio
6'006'006'048'0nnnn 22 IHHIT =++=++=
TRnVP ⋅⋅=⋅
( )atm1'33
1
673082'06'0
V
TRn
P =
⋅⋅
=
⋅⋅
=
Septiembre 2003. Problema 2B. En un recipiente cerrado de volumen constante igual a 22 L y a la temperatura de
305 K se introduce 1 mol de N2O4(g). Este gas se descompone parcialmente según la reacción
N2O4 (g) ↔ 2NO2 (g)
cuya constante de equilibrio KP vale 0,249 a dicha temperatura.
a) Calcule el valor de la constante de equilibrio, Kc.
b) Determine las fracciones molares de los componentes de la mezcla en equilibrio.
c) ¿Cuál es la presión total cuando se ha alcanzado el equilibrio?
Dato: R = 0,082 atm·mol-1
·K-1
Puntuación máxima por apartado: b)1,0 punto; a) y c) 0,5 puntos.
Solución.
a. Según la ley de acción de masas
42
2
2
c
ON
NO
K =
Aplicando la ecuación de gases ideales a cada componente de la mezcla
RT
P
V
n
:RTnVP ii
ii ==⋅
siendo
V
ni
la concentración de componente i.
RT
P
ON:
RT
P
NO 422 ON
42
NO
2 ==
sustituyendo estas expresiones en Kc y ordenando
RT
1
K
RT
1
P
P
RT
P
RT
P
K p
K
ON
2
NO
ON
2
NO
c
P
42
2
42
2
⋅=⋅=












=
321
expresión que permite calcular Kc conocida KP. Sustituyendo los datos
2
c 10
305082'0
1
249'0K −
=
⋅
⋅=
b. Partiendo de la estequiometria de la reacción se plantean las condiciones de equilibrio en función del número
inicial de moles de tetraóxido de nitrógeno y del grado de disociación
38
sustituyendo en Kc y ordenando
V
1
1
n4
V
nn
V
n2
K
2
o
oo
2
o
c ⋅
α−
α
=
α−





 α
=
Sustituyendo por los datos
α−
α
=⋅
α−
α⋅⋅
=−
1
4
22'0:
22
1
1
14
10
22
2
ordenando se obtiene una ecuación de segundo grado. Al resolver la ecuación se obtienen dos valores de los que uno de
ellos no tendrá sentido químico



−=α
=α
=−α+α
químicosentidotieneNo26'0
21'0
:022'022'04 2
Las fracciones molares se pueden expresar en función del grado de disociación teniendo en cuenta que, el
número total de moles presentes en el equilibrio es la suma de los moles de NO2 y de N2O4.
( ) ( )α+=α+=α−+α=+= 1nnnnnn2nnn ooooooONNOT 422
( )
( )
( )






=
+
−
=
α+
α−
=
α+
α−
==χ
=
+
⋅
=
α+
α
=
α+
α
==χ
65'0
21'01
21'01
1
1
1n
1n
n
n
35'0
21'01
21'02
1
2
1n
n2
n
n
o
o
T
ON
NO
o
o
T
NO
NO
42
2
2
2
c. La presión de equilibrio se puede calcular de dos formas, mediante KP ó mediante la ecuación de gases ideales.
Empleando esta segunda forma:
atm38'1
22
305082'021'1
V
RTn
P:RTnVP T
T =
⋅⋅
=
⋅
=⋅=⋅
Junio 2003. Cuestión 3. Justifique si las siguientes afirmaciones son ciertas o falsas:
a) Un valor negativo de una constante de equilibrio significa que la reacción inversa es espontánea.
b) Para una reacción exotérica, se produce un desplazamiento hacia la formación de productos al aumentar la
temperatura.
c) Para una reacción a temperatura constante con igual número de moles gaseosos de reactivos y productos, no se
produce desplazamiento del equilibrio si se modifica la presión.
d) Para una reacción a temperatura constante donde únicamente son gases los productos, el valor de la constante de
equilibrio disminuye cuando disminuimos el volumen del recipiente.
Puntuación máxima por apartado: 0,5 puntos.
Solución.
a) Falso. La constante de equilibrio es la relación entre el producto de las concentraciones de los productos y el
producto de las concentraciones de los relativos. Al ser las concentraciones nº reales positivos, las constantes de
equilibrio son positivas.
b) Falso. Una reacción exotérmica desprende calor, si se aumenta la temperatura, se introduce calor en el sistema y
el sistema se desplaza hacia los reactivos para volver a recuperar el equilibrio.
QBA +⇔
c) Verdadero. Las variaciones de presión solo modifican el equilibrio en sistemas gaseosos en los que el número
de moles gaseosos de reactivos es distinto al número de moles gaseosos de productos.
d) Falso. Las constantes de equilibrio solo son funciones de la temperatura y de la naturaleza de los reactivos, no
siendo función de la concentración, volumen ó presión.
39
Junio 2003. Problema 2B. El equilibrio PCl5 (g) ↔ PCl3 (g) + Cl2 (g) se alcanza calentando 3 g de pentacloruro de
fósforo hasta 300ºC en un recipiente de medio litro, siendo la presión final de 2 atm. Calcule:
a) El grado de disociación del pentacloruro de fósforo.
b) El valor de Kp a dicha temperatura.
Datos.- .0,31P;5,355CI:atómicasMasas;K·atm·L·mol082,0R 1-1
=== −
Puntuación máxima por apartados: 1 punto.
Solución.
a. Se pide estudiar el equilibrio de disociación del pentacloruro de fósforo en tricloruro de fósforo y cloro,
conocida la cantidad inicial de pentacloruro de fósforo y las variables presión, volumen y temperatura de equilibrio.
Las condiciones iniciales y de equilibrio se plantean en el siguiente cuadro
donde α representa el grado de disociación del pentacloruro de fósforo
El número total de moles en el equilibrio se puede expresar en función del número de moles iniciales y del
grado de disociación:
nT = n (PCl5) + n (PCl3) + n (Cl2) = no − noα + noα + noα = no + noα = no· (1 + α)
siendo
( )
( )
moles0144'0
mol
gr208'5
gr3
PClM
PClm
n
5
5
o ===
El número total de moles se puede calcular a partir de las variables de estado que definen el equilibrio mediante
la ecuación de gases ideales:
TR
VP
nT
⋅
⋅
=
igualando las dos expresiones de nT
( )
TR
VP
1no
⋅
⋅
=α+⋅
se puede despejar el grado de disociación del pentacloruro de fósforo
1
TRn
VP
o
−
⋅⋅
⋅
=α
sustituyendo por los datos del problema
( ) ( )
( ) ( )( )
478'01
K273300
Kmol
Latm
082'0moles0'0144
L5'0atm2
=−
+⋅





⋅
⋅
⋅
⋅
=α
El grado de disociación del pentacloruro de fósforo es del 47’8 %.
b. La constante de equilibrio en función de las presiones viene dada por la expresión:
5
23
PCl
ClPCl
P
P
PP
K
⋅
=
que mediante la ley de Raoult “Pi = P · χi” se puede expresar en función de la fracciones molares de los componentes
gaseosos de la mezcla
5
23
5
23
PCl
ClPCl
PCl
ClPCl
P P
·P
·P·P
K
χ
χ⋅χ
⋅=
χ
χ⋅χ
=
Las fracciones molares se pueden expresar en función del grado de disociación
( ) ( )
( ) α+
α−
=
α+
α−
==χ
1
1
1n
1n
n
PCln
o
o
T
5
PCl5
( )
( ) α+
α
=
α+
α
==χ=χ
11n
n
n
PCln
o
o
T
3
ClPCl 23
Equilibrio soluciones selectividad
Equilibrio soluciones selectividad
Equilibrio soluciones selectividad
Equilibrio soluciones selectividad
Equilibrio soluciones selectividad
Equilibrio soluciones selectividad
Equilibrio soluciones selectividad

Más contenido relacionado

La actualidad más candente

Compuestos de cordinadinacion 2
Compuestos de cordinadinacion 2Compuestos de cordinadinacion 2
Compuestos de cordinadinacion 2
mnilco
 
6reaccionesprecipitacion
6reaccionesprecipitacion6reaccionesprecipitacion
6reaccionesprecipitacion
Fernanda Bremer
 
Manual Permanganometria
Manual PermanganometriaManual Permanganometria
Manual Permanganometria
analiticauls
 
Ejercicios libro soluciones
Ejercicios libro solucionesEjercicios libro soluciones
Ejercicios libro soluciones
mariavarey
 

La actualidad más candente (20)

Ejercicios propuestos sobre métodos gravimétricos
Ejercicios propuestos sobre métodos gravimétricos Ejercicios propuestos sobre métodos gravimétricos
Ejercicios propuestos sobre métodos gravimétricos
 
Tabla de potenciales redox
Tabla de potenciales redoxTabla de potenciales redox
Tabla de potenciales redox
 
Reporte de Práctica-Síntesis del Benzoato de Metilo.
Reporte de Práctica-Síntesis del Benzoato de Metilo.Reporte de Práctica-Síntesis del Benzoato de Metilo.
Reporte de Práctica-Síntesis del Benzoato de Metilo.
 
Kps (Calculo de la solubilidad)
Kps (Calculo de la solubilidad)Kps (Calculo de la solubilidad)
Kps (Calculo de la solubilidad)
 
Kps clase 2015
Kps clase 2015Kps clase 2015
Kps clase 2015
 
Relación Kp y temperatura - Ecuacion Van't Hoff
Relación Kp y temperatura - Ecuacion Van't HoffRelación Kp y temperatura - Ecuacion Van't Hoff
Relación Kp y temperatura - Ecuacion Van't Hoff
 
Compuestos de cordinadinacion 2
Compuestos de cordinadinacion 2Compuestos de cordinadinacion 2
Compuestos de cordinadinacion 2
 
6reaccionesprecipitacion
6reaccionesprecipitacion6reaccionesprecipitacion
6reaccionesprecipitacion
 
Reporte de Práctica-Síntesis y Propiedades del Cloruro de Terc-butilo.
Reporte de Práctica-Síntesis y Propiedades del Cloruro de Terc-butilo.Reporte de Práctica-Síntesis y Propiedades del Cloruro de Terc-butilo.
Reporte de Práctica-Síntesis y Propiedades del Cloruro de Terc-butilo.
 
Manual Permanganometria
Manual PermanganometriaManual Permanganometria
Manual Permanganometria
 
Solucionario de balance Materia RODRIGO LONDOÑO GARCÍA
Solucionario de balance Materia RODRIGO LONDOÑO GARCÍASolucionario de balance Materia RODRIGO LONDOÑO GARCÍA
Solucionario de balance Materia RODRIGO LONDOÑO GARCÍA
 
Electroquímica
ElectroquímicaElectroquímica
Electroquímica
 
Problemas resueltos de Cinetica Quimica
Problemas resueltos de Cinetica QuimicaProblemas resueltos de Cinetica Quimica
Problemas resueltos de Cinetica Quimica
 
Síntesis de Ciclohexeno (practica)
Síntesis de Ciclohexeno (practica)Síntesis de Ciclohexeno (practica)
Síntesis de Ciclohexeno (practica)
 
Electrolisis ejercicios resueltos
Electrolisis ejercicios resueltosElectrolisis ejercicios resueltos
Electrolisis ejercicios resueltos
 
Práctica 3 Lab. Orgánica Cristalización por par de disolventes
Práctica 3 Lab. Orgánica Cristalización por par de disolventesPráctica 3 Lab. Orgánica Cristalización por par de disolventes
Práctica 3 Lab. Orgánica Cristalización por par de disolventes
 
Ejercicios resueltos de balance de energía sin reacción química
Ejercicios resueltos de balance de energía sin reacción químicaEjercicios resueltos de balance de energía sin reacción química
Ejercicios resueltos de balance de energía sin reacción química
 
Determinacion Gravimetrica de calcio
Determinacion Gravimetrica de calcioDeterminacion Gravimetrica de calcio
Determinacion Gravimetrica de calcio
 
Ejercicios libro soluciones
Ejercicios libro solucionesEjercicios libro soluciones
Ejercicios libro soluciones
 
Problemario de balance_de_materia_y_ener
Problemario de balance_de_materia_y_enerProblemario de balance_de_materia_y_ener
Problemario de balance_de_materia_y_ener
 

Similar a Equilibrio soluciones selectividad

Q5 pau-equilibrio
Q5 pau-equilibrioQ5 pau-equilibrio
Q5 pau-equilibrio
mariavarey
 
02 equilibrioqu e_dmico
02 equilibrioqu e_dmico02 equilibrioqu e_dmico
02 equilibrioqu e_dmico
Home
 
Equilibrio quimico general_1
Equilibrio quimico general_1Equilibrio quimico general_1
Equilibrio quimico general_1
Natalia Tello
 
2ª eval. química 2º bac 2012 2013
2ª eval. química 2º bac 2012 20132ª eval. química 2º bac 2012 2013
2ª eval. química 2º bac 2012 2013
quimbioalmazan
 
Q5 pau-equilibrio-soluc
Q5 pau-equilibrio-solucQ5 pau-equilibrio-soluc
Q5 pau-equilibrio-soluc
mariavarey
 

Similar a Equilibrio soluciones selectividad (20)

Q5 pau-equilibrio
Q5 pau-equilibrioQ5 pau-equilibrio
Q5 pau-equilibrio
 
Equilibrio químico
Equilibrio químicoEquilibrio químico
Equilibrio químico
 
equilibrioquímico
equilibrioquímicoequilibrioquímico
equilibrioquímico
 
02 Equilibrio Qu%E Dmico
02 Equilibrio Qu%E Dmico02 Equilibrio Qu%E Dmico
02 Equilibrio Qu%E Dmico
 
equilibrioquímico
equilibrioquímicoequilibrioquímico
equilibrioquímico
 
02 equilibrioqu e_dmico
02 equilibrioqu e_dmico02 equilibrioqu e_dmico
02 equilibrioqu e_dmico
 
Equilibrio quimico general_1
Equilibrio quimico general_1Equilibrio quimico general_1
Equilibrio quimico general_1
 
Equilibrio químico
Equilibrio químicoEquilibrio químico
Equilibrio químico
 
1º control 2ª eval. química 2º bac 2015 2016
1º control 2ª eval. química 2º bac 2015 20161º control 2ª eval. química 2º bac 2015 2016
1º control 2ª eval. química 2º bac 2015 2016
 
Equilibrio quimico yaipen
Equilibrio quimico yaipenEquilibrio quimico yaipen
Equilibrio quimico yaipen
 
equilibrioquímico
equilibrioquímicoequilibrioquímico
equilibrioquímico
 
02 equilibrioquimico
02 equilibrioquimico02 equilibrioquimico
02 equilibrioquimico
 
Equilibrio qumicos
Equilibrio qumicosEquilibrio qumicos
Equilibrio qumicos
 
02 equilibrioqumico
02 equilibrioqumico02 equilibrioqumico
02 equilibrioqumico
 
Ies luis cobiella_equilibrioquímico
Ies luis cobiella_equilibrioquímicoIes luis cobiella_equilibrioquímico
Ies luis cobiella_equilibrioquímico
 
2ª eval. química 2º bac 2012 2013
2ª eval. química 2º bac 2012 20132ª eval. química 2º bac 2012 2013
2ª eval. química 2º bac 2012 2013
 
Examen Termoquímica y Cinética
Examen Termoquímica y CinéticaExamen Termoquímica y Cinética
Examen Termoquímica y Cinética
 
Q5 pau-equilibrio-soluc
Q5 pau-equilibrio-solucQ5 pau-equilibrio-soluc
Q5 pau-equilibrio-soluc
 
Equilibrioquimicopresentacion 141120171731-conversion-gate02
Equilibrioquimicopresentacion 141120171731-conversion-gate02Equilibrioquimicopresentacion 141120171731-conversion-gate02
Equilibrioquimicopresentacion 141120171731-conversion-gate02
 
02 equilibrioquímico
02 equilibrioquímico02 equilibrioquímico
02 equilibrioquímico
 

Más de Gracia Gomez Lara

Más de Gracia Gomez Lara (20)

Resumen condicionales ingles
Resumen condicionales inglesResumen condicionales ingles
Resumen condicionales ingles
 
Puntos notables de un triangulo
Puntos notables de un trianguloPuntos notables de un triangulo
Puntos notables de un triangulo
 
Ficha 5 problemas concentraciones (parte ii)
Ficha 5 problemas concentraciones (parte ii)Ficha 5 problemas concentraciones (parte ii)
Ficha 5 problemas concentraciones (parte ii)
 
Ejercicios naturaleza de la materia i. ud2
Ejercicios naturaleza de la materia i. ud2Ejercicios naturaleza de la materia i. ud2
Ejercicios naturaleza de la materia i. ud2
 
Ejercicios leyes ponderales
Ejercicios leyes ponderalesEjercicios leyes ponderales
Ejercicios leyes ponderales
 
Matbtema1ejerciciosresueltos
Matbtema1ejerciciosresueltosMatbtema1ejerciciosresueltos
Matbtema1ejerciciosresueltos
 
T2 gravitatorio ejercicios
T2 gravitatorio ejerciciosT2 gravitatorio ejercicios
T2 gravitatorio ejercicios
 
Matematicas 4c2ba-eso-formulario
Matematicas 4c2ba-eso-formularioMatematicas 4c2ba-eso-formulario
Matematicas 4c2ba-eso-formulario
 
Enlace quimico
Enlace quimicoEnlace quimico
Enlace quimico
 
Naturaleza y estructura de la materia
Naturaleza y estructura de la materiaNaturaleza y estructura de la materia
Naturaleza y estructura de la materia
 
Ejercicios selectividad termoquimica
Ejercicios selectividad termoquimicaEjercicios selectividad termoquimica
Ejercicios selectividad termoquimica
 
Formulario 01-cinematica-1-bach
Formulario 01-cinematica-1-bachFormulario 01-cinematica-1-bach
Formulario 01-cinematica-1-bach
 
Formulario 06-geometria-analitica-1-bach-cc
Formulario 06-geometria-analitica-1-bach-ccFormulario 06-geometria-analitica-1-bach-cc
Formulario 06-geometria-analitica-1-bach-cc
 
Formulario 05-vectores-1-bach-cc
Formulario 05-vectores-1-bach-ccFormulario 05-vectores-1-bach-cc
Formulario 05-vectores-1-bach-cc
 
Formulario 01-numeros-reales-1-bach-cc
Formulario 01-numeros-reales-1-bach-ccFormulario 01-numeros-reales-1-bach-cc
Formulario 01-numeros-reales-1-bach-cc
 
Ejercicios disolucionesestequiometria
Ejercicios disolucionesestequiometriaEjercicios disolucionesestequiometria
Ejercicios disolucionesestequiometria
 
B3 magn2 resueltos
B3 magn2 resueltosB3 magn2 resueltos
B3 magn2 resueltos
 
Cuaderno 5 ejercicios campo magnetico
Cuaderno 5 ejercicios campo magneticoCuaderno 5 ejercicios campo magnetico
Cuaderno 5 ejercicios campo magnetico
 
Ejercicios fq3eso septiembre-2009
Ejercicios fq3eso septiembre-2009Ejercicios fq3eso septiembre-2009
Ejercicios fq3eso septiembre-2009
 
Tomo 3
Tomo 3Tomo 3
Tomo 3
 

Último

🦄💫4° SEM32 WORD PLANEACIÓN PROYECTOS DARUKEL 23-24.docx
🦄💫4° SEM32 WORD PLANEACIÓN PROYECTOS DARUKEL 23-24.docx🦄💫4° SEM32 WORD PLANEACIÓN PROYECTOS DARUKEL 23-24.docx
🦄💫4° SEM32 WORD PLANEACIÓN PROYECTOS DARUKEL 23-24.docx
EliaHernndez7
 
Concepto y definición de tipos de Datos Abstractos en c++.pptx
Concepto y definición de tipos de Datos Abstractos en c++.pptxConcepto y definición de tipos de Datos Abstractos en c++.pptx
Concepto y definición de tipos de Datos Abstractos en c++.pptx
Fernando Solis
 

Último (20)

Posición astronómica y geográfica de Europa.pptx
Posición astronómica y geográfica de Europa.pptxPosición astronómica y geográfica de Europa.pptx
Posición astronómica y geográfica de Europa.pptx
 
Biografía de Charles Coulomb física .pdf
Biografía de Charles Coulomb física .pdfBiografía de Charles Coulomb física .pdf
Biografía de Charles Coulomb física .pdf
 
ACERTIJO LA RUTA DEL MARATÓN OLÍMPICO DEL NÚMERO PI EN PARÍS. Por JAVIER SOL...
ACERTIJO LA RUTA DEL MARATÓN OLÍMPICO DEL NÚMERO PI EN  PARÍS. Por JAVIER SOL...ACERTIJO LA RUTA DEL MARATÓN OLÍMPICO DEL NÚMERO PI EN  PARÍS. Por JAVIER SOL...
ACERTIJO LA RUTA DEL MARATÓN OLÍMPICO DEL NÚMERO PI EN PARÍS. Por JAVIER SOL...
 
La Evaluacion Formativa SM6 Ccesa007.pdf
La Evaluacion Formativa SM6  Ccesa007.pdfLa Evaluacion Formativa SM6  Ccesa007.pdf
La Evaluacion Formativa SM6 Ccesa007.pdf
 
Prueba libre de Geografía para obtención título Bachillerato - 2024
Prueba libre de Geografía para obtención título Bachillerato - 2024Prueba libre de Geografía para obtención título Bachillerato - 2024
Prueba libre de Geografía para obtención título Bachillerato - 2024
 
FICHA PROYECTO COIL- GLOBAL CLASSROOM.docx.pdf
FICHA PROYECTO COIL- GLOBAL CLASSROOM.docx.pdfFICHA PROYECTO COIL- GLOBAL CLASSROOM.docx.pdf
FICHA PROYECTO COIL- GLOBAL CLASSROOM.docx.pdf
 
Desarrollo y Aplicación de la Administración por Valores
Desarrollo y Aplicación de la Administración por ValoresDesarrollo y Aplicación de la Administración por Valores
Desarrollo y Aplicación de la Administración por Valores
 
Usos y desusos de la inteligencia artificial en revistas científicas
Usos y desusos de la inteligencia artificial en revistas científicasUsos y desusos de la inteligencia artificial en revistas científicas
Usos y desusos de la inteligencia artificial en revistas científicas
 
🦄💫4° SEM32 WORD PLANEACIÓN PROYECTOS DARUKEL 23-24.docx
🦄💫4° SEM32 WORD PLANEACIÓN PROYECTOS DARUKEL 23-24.docx🦄💫4° SEM32 WORD PLANEACIÓN PROYECTOS DARUKEL 23-24.docx
🦄💫4° SEM32 WORD PLANEACIÓN PROYECTOS DARUKEL 23-24.docx
 
Plan-de-la-Patria-2019-2025- TERCER PLAN SOCIALISTA DE LA NACIÓN.pdf
Plan-de-la-Patria-2019-2025- TERCER PLAN SOCIALISTA DE LA NACIÓN.pdfPlan-de-la-Patria-2019-2025- TERCER PLAN SOCIALISTA DE LA NACIÓN.pdf
Plan-de-la-Patria-2019-2025- TERCER PLAN SOCIALISTA DE LA NACIÓN.pdf
 
Lecciones 06 Esc. Sabática. Los dos testigos
Lecciones 06 Esc. Sabática. Los dos testigosLecciones 06 Esc. Sabática. Los dos testigos
Lecciones 06 Esc. Sabática. Los dos testigos
 
Prueba de evaluación Geografía e Historia Comunidad de Madrid 4ºESO
Prueba de evaluación Geografía e Historia Comunidad de Madrid 4ºESOPrueba de evaluación Geografía e Historia Comunidad de Madrid 4ºESO
Prueba de evaluación Geografía e Historia Comunidad de Madrid 4ºESO
 
Feliz Día de la Madre - 5 de Mayo, 2024.pdf
Feliz Día de la Madre - 5 de Mayo, 2024.pdfFeliz Día de la Madre - 5 de Mayo, 2024.pdf
Feliz Día de la Madre - 5 de Mayo, 2024.pdf
 
Supuestos_prácticos_funciones.docx
Supuestos_prácticos_funciones.docxSupuestos_prácticos_funciones.docx
Supuestos_prácticos_funciones.docx
 
Tema 17. Biología de los microorganismos 2024
Tema 17. Biología de los microorganismos 2024Tema 17. Biología de los microorganismos 2024
Tema 17. Biología de los microorganismos 2024
 
Novena de Pentecostés con textos de san Juan Eudes
Novena de Pentecostés con textos de san Juan EudesNovena de Pentecostés con textos de san Juan Eudes
Novena de Pentecostés con textos de san Juan Eudes
 
Revista Apuntes de Historia. Mayo 2024.pdf
Revista Apuntes de Historia. Mayo 2024.pdfRevista Apuntes de Historia. Mayo 2024.pdf
Revista Apuntes de Historia. Mayo 2024.pdf
 
Concepto y definición de tipos de Datos Abstractos en c++.pptx
Concepto y definición de tipos de Datos Abstractos en c++.pptxConcepto y definición de tipos de Datos Abstractos en c++.pptx
Concepto y definición de tipos de Datos Abstractos en c++.pptx
 
Sesión de clase APC: Los dos testigos.pdf
Sesión de clase APC: Los dos testigos.pdfSesión de clase APC: Los dos testigos.pdf
Sesión de clase APC: Los dos testigos.pdf
 
Tema 10. Dinámica y funciones de la Atmosfera 2024
Tema 10. Dinámica y funciones de la Atmosfera 2024Tema 10. Dinámica y funciones de la Atmosfera 2024
Tema 10. Dinámica y funciones de la Atmosfera 2024
 

Equilibrio soluciones selectividad

  • 1. 1 Modelo 2014. Pregunta 5A.- Considere la reacción en equilibrio A (g) + 3B (g) 2C (g). Cuando se introduce 1 mol de A y 3 mol de B en un recipiente de 5 L y se alcanza el equilibrio a 350 K, se observa que se han formado 1,6 mol de C. a) Calcule la constante de equilibro Kp de la reacción a 350 K. Dato. R = 0,082 atm·L·mol−1 ·K−1 . Puntuación máxima por apartado: 0,5 puntos. Solución. a. x ≡ moles de A que reaccionan. El cuadro de reacción es: ( ) ( ) ( ) ( ) ( ) x2x33x1molEquilibrio.C 31molIniciales.C gC2gB3gA −− − ↔+ ( )( ) x26,1gCn Eq == ( )( ) ( )( )    =⋅−= =−= = mol6,08,033gBn mol2,08,01gAn :mol8,0x eq eq Conocidos los moles de todos los componentes del equilibrio y el volumen del sistema, se calcula Kc, y conocido este valor, mediante la relación entre las constantes, se calcula Kp ( ) ( )( )gn cp RTKK ∆⋅= . Por la ley de Acción de Masas: [ ] [ ] [ ] ( ) ( ) ( ) 5.1481 5 6,0 5 2,0 5 6,1 V Bn V An V Cn BA C K 3 2 3 2 3 2 c =     ⋅          =      ⋅           = ⋅ = ( ) ( ) ( ) ( ) 799.1350082,05,1481RTKK 312gn cp =⋅⋅=⋅= +−∆ Modelo 2014. Pregunta 5B.- El producto de solubilidad del hidróxido de hierro (III) a 25 ºC es Ks = 2,8×10−39 . a) Calcule la solubilidad de este hidróxido, en g·L−1 . Datos. Masas atómicas: Fe = 55,8; O = 16,0; H = 1,0; Cl = 35,5. Puntuación máxima por apartado: 0,5 puntos apartado a); 0,75 puntos apartados b) y c). Solución. a. El equilibrio heterogéneo de solubilidad del Fe(OH)3 es: ( ) ( ) ( ) −+ +↔ OH3aqFesOHFe 3 3 [ ] [ ]33 s OHFeK −+ ⋅= Si definimos por s los moles por litro de hidróxido disueltos, el cuadro de reacción queda de la siguiente forma. ( ) ( ) ( ) s3s OH3aqFesOHFe 3 3 − +↔ −+ Sustituyendo en la expresión del producto de solubilidad, se obtiene la solubilidad del hidróxido en mol L‒1 . [ ][ ] ( ) 3333 s s27s3sOHFeK =⋅=⋅= −+ 1104 39 4 s lmol1001,1 27 108,2 27 K s −− − ⋅×= × == Para expresarla en g L‒1 se multiplica por la masa molecular del hidróxido. 18110 Lg1008,1 mol g8,106 lmol1001,1s −−−− ⋅×=⋅⋅×= Septiembre 2013. Pregunta A4.- Se introduce fosgeno (COCl2) en un recipiente vacío de 1 L a una presión de 0,92 atm y temperatura de 500 K, produciéndose su descomposición según la ecuación: COCl2 (g) ↔ CO (g) + Cl2 (g). Sabiendo que en estas condiciones el valor de Kc es 4,63×10–3 ; calcule: a) La concentración inicial de fosgeno. b) Las concentraciones de todas las especies en el equilibrio. c) La presión parcial de cada uno de los componentes en el equilibrio. Dato. R = 0,082 atm·L·mol−1·K−1 . Puntuación máxima por apartado: 0,5 puntos apartado a); 0,75 puntos apartados b) y c). Solución.
  • 2. 2 a. Aplicando la ecuación de gases ideales a las condiciones iniciales: nRTVP =⋅ RT P V n = ( ) [ ] RT P COCl V COCln o o2 o2 == [ ] L mol1024,2 K500 Kmol Latm 082,0 atm92,0 COCl 2 o2 − ×= ⋅ ⋅ ⋅ = b. Si se denomina x al número de moles de fosgeno que se disocian, el cuadro de reacción queda de la siguiente forma: ( ) ( ) ( ) ( ) ( ) xxx92,0 L molequilibrio.Cond 0,92 L moliniciales.Cond gClgCOgCOCl 22 − −− +↔ Según la ley de acción de masas, la constante de equilibrio en función de las concentraciones es [ ] [ ] [ ]2 2 c COCl ClCO K ⋅ = Sustituyendo por los valores de cuadro de reacción y por el valor de la contante, y ordenando, se obtiene una ecuación de segundo grado. x1024,2 x x1024,2 xx 1063,4 2 2 2 3 −× = −× ⋅ =× −− −     −= ×= =×−×+ − −− químicosentidotieneNo01,0x 101,8x :01004,11063,4x 3 432 [ ] L mol1043,1101,81024,2COCl 232 eq2 −−− ×=×−×= [ ] [ ] L mol101,8ClCO 3 eq2eq − ×== c. Aplicando la ecuación de gases ideales a cada componente de la mezcla gaseosa: nRTVP =⋅ RT V n P = [ ] atm0,586K500 Kmol Latm 082,0 L mol 1043,1RTCOClRT V n P 2 2 COCl COCl 2 2 =⋅ ⋅ ⋅ ⋅×=⋅== − [ ] atm0,332K500 Kmol Latm 082,0 L mol 101,8RTClRT V n PP 3 2 Cl ClCO 2 2 =⋅ ⋅ ⋅ ⋅×=⋅=== − Septiembre 2013. Pregunta B2.- Se tiene una reacción en equilibrio del tipo: aA (g) + bB (g) ↔ cC (l) + dD (s). a) Escriba la expresión de Kp. b) Justifique cómo se modifica el equilibrio cuando se duplica el volumen del recipiente. c) Justifique cómo se modifica el equilibrio si se aumenta la presión parcial de la sustancia A. d) Justifique qué le ocurre al valor de Kp si aumenta la temperatura del sistema. Puntuación máxima por apartado: 0,5 puntos. Solución. a. Según la ley de acción de masas, las constantes de equilibrios heterogéneos solo dependen de las sustancias que estén en el estado de agregación con mayor libertad, en el caso de un equilibrio sólido-líquido-gas, solo dependerá de las sustancias que estén en estado gaseoso. b B a A p PP 1 K ⋅ = b. Si en un sistema en equilibrio aumenta el volumen o disminuye la presión, el equilibrio se desplaza hacia el sentido donde exista mayor número de moles gaseosos y de esa forma contrarrestar el aumento de volumen o la disminución de presión.
  • 3. 3 c. Si en el sistema en equilibrio aumentamos la presión parcial de A, y teniendo en cuenta que el valor de Kp no varia con la presión, deberá disminuir la presión parcial de B, lo cual se consigue desplazando el equilibrio hacia la derecha. d. Dependerá de que la reacción sea endotérmica o exotérmica. Si la reacción es endotérmica, al aumentar la temperatura aumentará el valor de Kp, y si es exotérmica, al aumentar la temperatura disminuirá el valor de Kp. Junio 2013. Pregunta 2A.- Justifique si son verdaderas o falsas las siguientes afirmaciones: d. La constante de solubilidad de una sal poco soluble aumenta por efecto ión común. Puntuación máxima por apartado: 0,5 puntos. Solución. d. Falso. El efecto ión común, influye en la solubilidad de la sal, pero no influye en la constante, la cual solo es función de la temperatura. Junio 2013. Pregunta 5A.- El valor de la constante de equilibrio Kc para la reacción H2 (g) + F2 (g) 2HF (g), es 6,6×10–4 a 25 ºC. Si en un recipiente de 10 L se introduce 1 mol de H2 y 1 mol de F2, y se mantiene a 25 ºC hasta alcanzar el equilibrio, calcule: a) Los moles de H2 que quedan sin reaccionar una vez que se ha alcanzado el equilibrio. b) La presión parcial de cada uno de los compuestos en el equilibrio. c) El valor de Kp a 25 ºC. Dato. R = 0,082 atm·L·mol−1 ·K−1 . Puntuación máxima por apartado: 0,75 puntos apartados a) y b); 0,5 puntos apartado c). Solución. a. Si se denomina por x al número de moles que reaccionan de hidrógeno y fluor, el cuadro de reacción en función de x queda de la siguiente forma: ( ) ( ) ( ) ( ) ( ) x2x1x1mol.Eq.C 11mol.I.C gHF2gFgH 22 −− − ⇔+ El número de moles de cada componente en el equilibrio se obtiene calculando x a partir del valor de la constante de equilibrio. [ ] [ ] [ ] ( ) ( ) ( ) ( )( ) ( ) ( ) ( ) ( ) ( ) ( ) ( )2 22 22 2 22 2 22 2 c x1 x2 x1x1 x2 FnHn HFn V Fn V Hn V HFn FH HF K − = −⋅− = ⋅ = ⋅       = ⋅ = 2 c x1 x2 K       − = cK x1 x2 = − mol1027,1 106,62 106,6 K2 K x 2 4 4 c c − − − ×= ×+ × = + = ( ) mol987,01027,11x1Hn 2 Eq2 =×−=−= − b. La presión parcial de un componente de una mezcla gaseosa es V RTn P i i ⋅ = ( ) atm4,2 10 298082,0987,0 V RTHn PP 2 FH 22 = ⋅⋅ = ⋅ == ( ) atm062,0 10 298082,01027,12 V RTHFn P 2 HF = ⋅⋅×⋅ = ⋅ = − c. ( ) ( ) ( ) c 0 c gn cp KRTKRTKK =⋅=⋅= ∆ Junio 2013. Pregunta 2B.- La siguiente reacción, no ajustada: CH3OH (l) + O2 (g) H2O (l) + CO2 (g) es exotérmica a 25 ºC. a) Escriba la expresión para la constante de equilibrio Kp de la reacción indicada. b) Razone cómo afecta al equilibrio un aumento de la temperatura. c) Razone cómo afecta a la cantidad de CO2 desprendido un aumento de la cantidad de CH3OH (l).
  • 4. 4 d) Justifique cómo se modifica el equilibrio si se elimina CO2 del reactor. Puntuación máxima por apartado: 0,5 puntos. Solución. NOTA: Por lo general, las reacciones de combustión son irreversibles, por lo tanto no tiene sentido hablar de equilibrio en una reacción de combustión. En cualquier caso, como se propone como equilibrio, lo trataremos como tal. a. Por ser un equilibrio heterogéneo líquido/gas, las constantes de equilibrio solo serán función de las especies que estén en el estado de agregación de mayor libertad (gas). ( ) ( ) ( ) ( )gCO2lOH4gO3lOHCH2 2223 +↔+ 3 O 2 CO p 2 2 P P K = b. Según el principio de Le Châtelier, al producir una perturbación en un sistema en equilibrio, este evoluciona en contra de la perturbación de forma que reestablezca el equilibrio. Si se aumenta la temperatura, el sistema tiende a desplazarse en el sentido endotérmico (absorbiendo calor), y de esa forma restablecer el equilibrio. Teniendo en cuenta que la reacción es exotérmica, tal y como dice el enunciado, el sentido endotérmico será hacia la izquierda, por lo tanto, al aumentar la temperatura, el equilibrio se desplaza hacia los reactivos. c. Por encontrarse el metanol en estado líquido y ser un equilibrio heterogéneo líquido/gas, la concentración de etanol no influye en el equilibrio, y por tanto, no influye en la cantidad de CO2 desprendida. d. Al eliminar CO2, el sistema se desplaza hacia la derecha, generando más CO2, y oponiéndose a la perturbación. Modelo 2013. Pregunta 1A.- Cuando se trata agua líquida con exceso de azufre sólido en un recipiente cerrado, a 25 ºC, se obtienen los gases sulfuro de hidrógeno y dióxido de azufre. a) Formule el equilibrio que se establece entre reactivos y productos. b) Escriba las expresiones de Kc y Kp. c) Indique cómo afecta al equilibrio un aumento de presión. Puntuación máxima por apartado: 0,5 puntos. Solución. a. ( ) ( ) ( ) ( )gSOgSH2sS3lOH2 222 +↔+ b. En los equilibrios heterogéneos, las constantes solo dependen la las especies que se encuentren en el estado de agregación con mayor grado de libertad. El equilibrio propuesto es sólido / líquido / gas, por lo tanto las constantes solo depende de los componentes que estén en estado gas. [ ] [ ]2 2 2c SOSHK ⋅= 22 SO 2 SHp PPK ⋅= c. Según Le Chàtelier, si un sistema químico en equilibrio experimenta un cambio, entonces el equilibrio se desplaza para contrarrestar el cambio impuesto y restablecer el equilibrio. Si se aumenta la presión, el equilibrio se desplazará en el sentido en el que disminuya el volumen y de esa forma contrarrestar el aumento de presión. Como el número de moles de especies gaseosas es mayor en los productos, el equilibrio se desplaza hacia los reactivos. Modelo 2013. Pregunta 2B.- El yoduro de bismuto (III) es una sal muy poco soluble en agua. a) Escriba el equilibrio de solubilidad del yoduro de bismuto sólido en agua. b) Escriba la expresión para la solubilidad del compuesto BiI3 en función de su producto de solubilidad. c) Sabiendo que la sal presenta una solubilidad de 0,7761 mg en 100 mL de agua a 20 ºC, calcule la constante del producto de solubilidad a esa temperatura. Datos. Masas atómicas: Bi = 209,0; I = 126,9 Puntuación máxima por apartado: 0,5 puntos apartados a) y b); 1 punto apartado c). Solución. a. ( ) ( ) ( )aqI3aqBisBiI 3OH 3 2 −+ + →← b. Si se solubilizan s moles por litro de la sal: ( ) ( ) ( ) s3sexc aqI3aqBisBiI 3OH 3 2 −+ + →←
  • 5. 5 [ ] [ ] ( ) 44333 s 27 Ks ss27s3sIBiK =⇒=⋅=⋅= −+ c. ( ) ( ) ( ) L mol1032,1 L10100 molg9,1263209 g107761,0 LV M m LV disueltaSaln s 6 3 3 − − − ×= × ⋅+ × === [ ] [ ] ( ) 1946433 s 102,81032,127s27IBiK −−−+ ×=×⋅==⋅= Modelo 2013. Pregunta 4B.- En un recipiente de 15 litros se introducen 3 mol de compuesto A y 2 mol del compuesto B. Cuando se calienta el recipiente a 400 K se establece el siguiente equilibrio: 2 A (g) + B (g) ⇔ 3 C (g). Sabiendo que cuando se alcanza el equilibrio las presiones parciales de B y C son iguales, calcule: a) Las concentraciones de A, B y C en el equilibrio. b) La presión total en el equilibrio. c) El valor de las constantes de equilibrio Kc y Kp a 400 K. Dato. R = 0,082 atm·L·mol−1 ·K−1 Puntuación máxima por apartado: 1 punto apartado a); 0,5 puntos apartados b) y c). Solución. a. Equilibrio homogéneo en fase gas. Si se denomina como x al número de moles que reaccionan de B, el cuadrado de reacción queda de la siguiente forma: ( ) ( ) ( ) ( ) ( ) x3x2x23molEquilibrio.C 23molIniciales.C gC3gBgA2 −− − ↔+ Si en el equilibrio las presiones parciales de B y C son iguales y, teniendo en cuenta que T RTn P i i = , el número de moles de B y C también sarán iguales       =⇒=== CBC CB B nnP T RTn T RTn P . ( ) ( ) mol5,0xx3x2CnBn ==−= Conocidos los moles en el equilibrio de cada componente y el volumen del recipiente, se calculan las concentraciones. ( ) [ ] ( ) ( ) ( ) [ ] ( ) ( ) ( ) [ ] ( ) ( ) 1 1 1 lmol1,0 15 5,1 lV Cn C5,15,03Cn lmol1,0 15 5,1 lV Bn B5,15,02Bn lmol133,0 15 2 lV An A25,023An − − − ⋅====⋅= ⋅====−= ⋅====⋅−= b. Conocidos los moles de cada componente del equilibrio se calcula el número total de moles, y con La ecuación de gases ideales, se calcula la presión total. ( ) ( ) ( ) mol55,15,12CnBnAnnT =++=++= atm9,10 L15 K400 Kmol Latm 082,0mol5 V RTn P T = ⋅⋅ == c. Conocidas las concentraciones de cada uno de los componentes del equilibrio, se calcula la constante de equilibrio en función de las concentraciones aplicando la ley de Acción de Masas. [ ] [ ] [ ] 565,0 1,0133,0 1,0 CA C K 2 3 2 3 c ⋅ = ⋅ = Conocida Kc se calcula Kp mediante la relación entre ellas. ( ) ( ) ( ) ( ) ( ) 565,0KRTKRTKRTKK c 0 c 123 c gn cp ==⋅=⋅=⋅= +−∆
  • 6. 6 Septiembre 2012. Pregunta A5.- En un recipiente cerrado de 1 L de capacidad se introducen 73,6 gramos de tetraóxido de dinitrógeno. Se mantiene a 22 ºC hasta alcanzar el equilibrio N2O4 (g) ⇔ 2NO2 (g), siendo Kc = 4,66×10‒3 . a) Calcule las concentraciones de ambos gases en el equilibrio. b) Calcule el valor de Kp. c) Cuando la temperatura aumenta al doble, aumenta Kc. Justifique el signo de ∆H para esta reacción. Datos. R = 0,082 atm·L·mol‒1 ·K‒1 . Masas atómicas: N = 14 y O = 16. Puntuación máxima por apartado: 1 punto apartado a); 0,5 puntos apartados b) y c). Solución. a. Se plantea el cuadro de reacción para la disociación del tetraóxido de dinitrógeno denominando por x la concentración de tetraóxido de dinitrógeno disociado. ( ) ( ) x2xCEquilibrio.Cond CInicialesCond. gNO2gON o o 242 − − ↔ Llevando las condiciones del equilibrio a la definición de la constante Kc, se plantea una ecuación de segundo grado que permite calcular el valor de x. [ ] [ ] ( ) xC x4 xC x2 ON NO K o 2 o 2 42 2 2 c − = − == Ordenando: 0CKxKx4 occ 2 =−+ [ ] ( ) ( ) ( ) L mol8,0 L1 molg92 g6,73 V ONMONm V ONn ONC 424242 42o =====    −= = =⋅×−×+ −− válidaNo03,0x 03,0x :08,01066,41066,4x4 332 [ ] L mol77,003,08,0xCON o42 =−=−= [ ] L mol06,003,02x2NO2 =⋅== b. [ ] [ ]( ) [ ] [ ] [ ] RTKRT ON NO RTON RTNO RTCP idealesgases.Ecc P P K c 42 2 2 42 2 2 iiON 2 NO P 42 2 ⋅== ⋅ ⋅ =       ⋅= == 113,0295082,01066,4RTKK 3 cP =⋅⋅×=⋅= − c. Según Le Chatelier al producir una perturbación en un sistema en equilibrio, el sistema evoluciona en contra de la perturbación, si aumenta la temperatura el sistema se desplaza en el sentido endotérmico, consumiendo calor y de esta forma oponiéndose al aumento de temperatura. Si al aumentar la temperatura, aumenta la constante (aumentan los productos y disminuyen los reactivos), el equilibrio se esta desplazando hacia la derecha, por consiguiente el sentido endotérmico es hacia la derecha, la reacción es ENDOTÉRMICA. Septiembre 2012. Pregunta B2.- Para las sales cloruro de plata y yoduro de plata, cuyas constantes de producto de solubilidad, a 25 ºC, son 1,6×10‒10 y 8×10‒17 , respectivamente: a) Formule los equilibrios heterogéneos de disociación y escriba las expresiones para las constantes del producto de solubilidad de cada una de las sales indicadas, en función de sus solubilidades. b) Calcule la solubilidad de cada una de estas sales en g·L‒1 . c) ¿Que efecto produce la adición de cloruro de sodio sobre una disolución saturada de cloruro de plata? d) ¿Como varia la solubilidad de la mayoría de las sales al aumentar la temperatura? Justifique la respuesta. Datos. Masas atómicas: Cl = 35,5; Ag = 108,0; I = 127,0. Puntuación máxima por apartado: 0,5 puntos. Solución. a. Equilibrios heterogéneos sólido/líquido. ( ) ( ) ( ) ssexc aqClaqAgsAgCl −+ +↔ [ ] [ ] 2 s sClAgK =⋅= −+
  • 7. 7 ( ) ( ) ( ) ssexc aqIaqAgsAgI −+ +↔ [ ] [ ] 2 s sIAgK =⋅= −+ b. AgCl: ( ) 2 s sAgClK = ⇒ ( ) L mol1026,1106,1KAgCls 510 s −− ×=×== ( ) 135 Lg1082,1 mol g 5,143 L mol 1026,1AgCls −−− ⋅×=⋅×= AgI: ( ) 2 s sAgIK = ⇒ ( ) L mol1094,8108KAgIs 917 s −− ×=×== ( ) 169 Lg101,2 mol g 235 L mol 1094,8AgIs −−− ⋅×=⋅×= c. Según Le Chatelier al producir una perturbación en un sistema en equilibrio, el sistema evoluciona en contra de la perturbación, si se aumenta la concentración de cloruros, el sistema evoluciona en el sentido en el que se consuma cloruros, hacia la izquierda, disminuyendo la solubilidad de la sal, y produciendo la aparición de un precipitado de AgCl. d. Para la mayoría de la sales el proceso de disolución es endotérmico ( ) ( ) ( )aqBaqAQsAB −+ +↔+ Al aumentar la temperatura, el equilibrio se desplaza en el sentido endotérmico, consumiendo calor y oponiéndose de esa forma al aumento de temperatura, por lo tanto se desplaza hacia la derecha aumentando la solubilidad de la sal. Junio 2012. Pregunta 5A.- Se introducen 0,5 moles de pentacloruro de antimonio en un recipiente de 2 litros. Se calienta a 200 ºC y una vez alcanzado el equilibrio, hay presentes 0,436 moles del compuesto. Todas las especies son gases a esa temperatura. a) Escriba la reacción de descomposición del pentacloruro de antimonio en cloro molecular y en tricloruro de antimonio. b) Calcula Kc para la reacción anterior. c) Calcule la presión total de la mezcla en el equilibrio. Datos. R = 0,082 atm·L·K‒1 ·mol‒1 . Puntuación máxima por apartado: 0,5 puntos apartado a); 0,75 puntos apartado b) y c). Solución. a. ( ) ( ) ( )gClgSbClgSbCl 235 +↔ b. Por la Ley de Acción de Masas, la constante de equilibrio es: [ ] [ ] [ ]5 23 c SbCl ClSbCl K ⋅ = Para calcular las concentraciones en el equilibrio, se plantea el siguiente cuadro de reacción (mol), donde x representa los moles de pentacloruro de antimonio que reaccionan. ( ) ( ) xxx5,0Equilibrio.C 0,5Iniciales.C ClgSbClgSbCl 235 − −− +↔ Por los datos del enunciado, se sabe que el número de moles de pentacloruro de antimonio en el equilibrio son 0,436. 436,0x5,0 =− ; mol064,0x = Conocidos los moles de todas la especies en el equilibrio, se calculan susu concentraciones, y con estas, el valor de la constante de equilibrio. [ ] ( ) [ ] [ ] L mol032,0 2 064,0 V n ClSbCl; L mol218,0 2 436,0 V SbCln SbCl Eq 23 5 5 =======
  • 8. 8 3 c 107,4 436,0 032,0032,0 K − ×= ⋅ = c. La presión en el equilibrio se calcula mediante la ecuación de gases ideales con el número total de moles de la mezcla gaseosa. ( ) ( ) ( ) mol564,0064,0064,0436,0ClnSbClnSbClnn eq2eq3eq5T =++=++= atm9,10 2 473082,0564,0 V nRT P = ⋅⋅ == Modelo 2012. Pregunta 2B.- Para la reacción en fase gaseosa A + B ↔ C los valores de entalpía de reacción y energía de activación de la reacción directa son: ∆H = −150 kJ·mol‒1 y Ea = 85 kJ·mol‒1 . a. Justifique el efecto de un aumento de temperatura en la constante de equilibrio y en la composición en equilibrio. c. Justifique el efecto de un aumento de volumen en la constante de equilibrio y en la composición en equilibrio. Puntuación máxima por apartado: 0,5 puntos. Solución. En los apartados referentes a perturbaciones sobre el equilibrio, hay que tener en cuenta la Ley de Le Chatelier. “Siempre que se modifique las condiciones de un sistema en equilibrio se produce un desplazamiento del mismo en el sentido que restablezca las condiciones iniciales”. a. Al aumentar la temperatura en un sistema en equilibrio, este se desplaza en el sentido endotérmico, absorbiendo calor y oponiéndose a la perturbación. Por tratarse de una reacción exotérmica, el sentido endotérmico es hacia la izquierda, por lo tanto, al aumentar la temperatura el sistema se desplaza hacia la izquierda, aumentando las concentraciones de los reactivos y disminuyendo la de los productos, lo cual produce una disminución en el valor de la constante de equilibrio. c. Las variaciones de volumen no afectan al valor de la constante de equilibrio, la cual solo es función de la temperatura. Para equilibrios gaseosos con variación del número de moles gaseosos entre reactivos y productos ( )( )0gn ≠∆ , las variaciones de volumen desplazan el equilibrio. Si se aumenta el volumen, el equilibrio se desplaza en el mismo sentido que lo haría con una disminución de presión, se desplazará en el sentido en el que aumente el volumen del sistema, para la reacción propuesta hacia la izquierda (reactivos). Septiembre 2011. Pregunta 5A.- Cuando se ponen 0,7 moles de N2O4 en un reactor de 10 L a 359 K se establece el equilibrio N2O4 (g) ↔ 2 NO2 (g) y la presión es de 3,3 atm. Calcule: a) La concentración molar de todas las especies en el equilibrio. b) El valor de Kc. c) Si el sistema se comprime hasta reducir el volumen a 8 L ¿cual seria la presión total en el equilibrio? Dato. R = 0,082 atm·L·mol‒1 ·K‒1 . Puntuación máxima por apartado: a) y c) 0,75 puntos; b) 0,5 puntos. Solución. a. Para resolver el problema es conveniente plantear el siguiente cuadro de reacción, donde x representa los moles de N2O4 que se disocian. El número total de moles en el equilibrio es la suma de los moles de N2O4 y de NO2. ( ) ( ) ( ) x7,0x2x7,0NOnONneqn Eq2Eq42T +=+−=+= El número de moles en el equilibrio, se puede calcular mediante la ecuación de gases ideales conocidas las condiciones del equilibrio (P, V y T). mol12,1 K359 Kmol Latm 082,0 L10atm3,3 RT PV nT = ⋅ ⋅ ⋅ ⋅ ==
  • 9. 9 Conocidos los moles totales en el equilibrio se calculan los moles disociados de N2O4. ( ) 12,1x7,0eqnT =+= ; mol42,0x = Conocidos los moles de N2O4 disociados se calculan los moles de cada compuesto en el equilibrio, y dividiendo estos por el volumen las concentraciones en el equilibrio. ( ) [ ] ( ) ( ) [ ] ( ) M084,0 10 84,0 V NOn NOmol84,042,02x2NOn M028,0 10 28,0 V ONn ONmol28,042,07,0x7,0ONn 2 2Eq2 42 42Eq42 ====⋅== ====−=−= b. [ ] [ ] 252,0 028,0 084,0 ON NO K 2 42 2 2 c === c. Para calcular la nueva presión una vez alcanzado el equilibrio después de la perturbación, es necesario calcular el número de moles de cada componente en el nuevo equilibrio. Para calcular las nuevas condiciones del equilibrio se tiene en cuenta que el valor de la constante no depende del volumen. [ ] [ ] 42 2 42 2 ON 2 NO ON 2 NO 42 2 2 c n n V 1 V n V n ON NO K ⋅=         == Al disminuir el volumen del sistema, y siendo diferente el número de moles gaseosos en reactivos y productos, el equilibrio se desplaza hacia donde menor volumen ocupe, hacia la izquierda (reactivos). Si se denomina como x ahora al número de moles de NO2 que se dimerizan para volver a formar N2O4, el cuadro de reacción queda de la siguiente forma: Llevando los valores del equilibrio a la expresión de la constante, se llega a una ecuación de segundo grado. 42 2 ON 2 NO c n n V 1 K ⋅= ; ( ) 2 x 28,0 x84,0 8 1 252,0 2 + − ⋅= ; ( )2 x84,0 2 x 28,08252,0 −=      +⋅⋅    = = =+− Válida0,054x válidaNo63,2x :0142,0x688,2x2 Conocido x se calculan los moles de cada componente en el nuevo equilibrio. ( ) mol307,0 2 054,0 28,0ONn Eq42 =+= ; ( ) mol786,0054,084,0NOn Eq2 =−= ( ) ( ) ( ) mol093,1786,0307,0NOnONnTn Eq2Eq42Eq =+=+= Conocidos los moles en el equilibrio, con la ecuación de gases ideales se calcula la nueva presión. atm4 L8 K359 Kmol Latm 0,082mol093,1 V nRT P ≈ ⋅ ⋅ ⋅ ⋅ == Septiembre 2011. Pregunta 2B.- El hidróxido de magnesio es poco soluble en agua (Ks = 1,8·10‒11 ). a) Formule el equilibrio de disolución del hidróxido de magnesio y escriba la expresión para Ks. b) Calcule la solubilidad en mol·L‒1 . c) Como afectaría a la solubilidad la adición de acido clorhídrico? d) Como afectaría a la solubilidad la adición de cloruro de magnesio? Puntuación máxima por apartado: 0,5 puntos. Solución. a. ( ) ( ) ( ) ( )aqOH2aqMgsOHMg 2 2 −+ +↔
  • 10. 10 [ ] [ ]22 s OHMgK −+ ⋅= b. Si se disuelven s moles por litro de la sal, el cuadro de reacción será: Sustituyendo en la expresión de la constante se obtiene una relación entre la constante de solubilidad y la solubilidad de la sal. [ ] [ ] ( ) 3222 s s4s2sOHMgK =⋅=⋅= −+ ; 143 11 3 s Lmol1065,1 4 108,1 4 K s −− − ⋅⋅= ⋅ == c. Al añadir un ácido fuerte, los protones del ácido reaccionan con OH‒ del hidróxido formando agua, al disminuir la concentración de OH‒ , el sistema se desplaza hacia la derecha para volver a recuperar el equilibrio, disolviéndose mas cantidad de Mg(OH)2, y aumentando su solubilidad. d. Al añadir MgCl2 se produce el efecto de ión común, aumentando la concentración de Mg2+ lo que desplaza el equilibrio hacia la izquierda y disminuye la solubilidad. Junio 2011. Pregunta 5A.- En un recipiente de 5 L se introducen 3,2 g de COCl2 a 300 K. Cuando se alcanza el equilibrio COCl2 ↔ CO + Cl2, la presión final es de 180 mm de Hg. Calcule: d) Las presiones parciales de COCl2, CO y Cl2 en el equilibrio. e) Las constantes de equilibrio Kp y Kc. Datos. R = 0,082 atm·L·mol‒1 · K‒1 ; Masas atómicas: C = 12; 0= 16; Cl = 35,5. Puntuación máxima por apartado: a) 1 punto. Solución. a. Las presiones parciales de los componentes del equilibrio se pueden calcular mediante la ecuación de gases ideales conocido el volumen, la temperatura y los moles de cada componente. V RTn P i i = Para calcular los moles en el equilibrio hay que tener en cuenta que el fosgeno (COCl2) se disocia según el siguiente cuadro de reacción. ( ) ( ) ( ) ( ) ( ) xxxnmolEquilibrioC. nmolinicialesC. gClgCOgCOCl o o 22 − −− +⇔ Siendo no el número de moles iniciales y x los moles de fosgeno (COCl2) que se disocian. El número total de moles en el equilibrio se puede calcular con los datos del equilibrio y como suma de los moles de cada uno de los componentes. ( ) ( ) ( ) xnClnCOnCOClnn oEq2EqEq2Eq +=++= mol048,0 K300 Kmol Latm 082,0 L5atm 760 180 RT VP n Eq Eq = ⋅ ⋅ ⋅ ⋅ = ⋅ = : ( ) ( ) mol032,0 mol g99 g2,3 COClM COClm n 2 2 o === Sustituyendo en la igualdad anterior: x032,0048,0 += : 016,0x = Número de moles y presiones parciales en el equilibrio ( ) ( ) ( ) atm08,0 L5 K300 Kmol Latm 082,0mol016,0 PP:016,0xClnCOn atm08,0 L5 K300 Kmol Latm 082,0mol016,0 P:016,0016.0032,0xnCOCln 2 2 ClCOEq2Eq COCloEq2 = ⋅ ⋅ ⋅ ⋅ ===== = ⋅ ⋅ ⋅ ⋅ ==−=−=
  • 11. 11 El problema también se puede resolver en función de las presiones planteando el cuadro de reacción en función de la presión. ( ) ( ) ( ) ( ) ( ) xxxPatmEquilibrioC. PatminicialesC. gClgCOgCOCl o o 22 − −− +⇔ Siendo x la presión de equilibrio del monóxido de carbono y del cloro molecular. xPPPPPP oClCOCOCliT 22 +=++== ∑ atm24,0 760 180 PT == : atm16,0 L5 K300 Kmol Latm 082,0 mol g99 g2,3 V RTn P o o = ⋅ ⋅ ⋅ ⋅ == 16,0x24,0 += : atm08,0x = atm08,016,024,0xPP oCOCl2 =−=−= : atm08,0xPP 2ClCO === b. Conocidas las presiones parciales se calcula el valor de la constante KP, y mediante la relación entre las constantes el valor de KC. 08,0 08.0 08.008,0 P PP K 2 2 COCl ClCO P = ⋅ = ⋅ = ( ) ( ) 3 12n P c 1025,3 300082,0 08,0 RT K K − −∆ ×= ⋅ == Modelo 2011. Cuestión 2A.- Diga si son ciertas o falsas las siguientes afirmaciones, razonando sus respuestas: d. La solubilidad del fluoruro de magnesio en agua es 8,2510−5 M. Dato. Ks = 6,8·10−9 . Puntuación máxima por apartado: 0,5 puntos. Solución. d. Falso. El fluoruro de magnesio se disocia según: ( ) ( ) ( )aqF2aqMgsMgF 2OH 2 2 −+ + → La solubilidad de la sal se puede obtener del producto de solubilidad a partir de su definición en función de las concentraciones y estas últimas, en función de la solubilidad (s) de la sal (por cada s moles por litro que se disuelven de la sal se forman s moles por litro de catión magnesio y 2 s moles litro de fluoruro). ( ) 322 s s4s2sFMgK =⋅=⋅= −+ M1019,1 4 108,6 4 K s 33 9 3 s − − ⋅= ⋅ == Modelo 2011. Problema 2A.- A 532 K se introducen 0,1 moles de PCl5 en un recipiente X de 1,2 L y 0,1 moles en otro recipiente Y. Se establece el equilibrio PCl5 ↔ PCl3 + Cl2, y la cantidad de PCl5 se reduce un 50% en el recipiente X y un 90% en el recipiente Y. Todas las especies se encuentran en fase gaseosa. Calcule: a) La presión en el equilibrio en el recipiente X. b) La constante de equilibrio Kc. c) El volumen del recipiente Y. d) La presión en el equilibrio en el recipiente Y. Datos. R = 0,082 atm・L・mol−1 ・K−1 . Puntuación máxima por apartado: 0,5 puntos. Solución. a. Independientemente del reactor donde se lleve a cabo la reacción y, teniendo en cuenta que se conoce el número de moles iniciales y el grado de disociación ( )9,0;5,0 YX =α=α , el cuadros de reacción es el que se muestra a continuación
  • 12. 12 La presión en el interior del reactor X se calcula mediante la ecuación de gases ideales, siendo necesario calcular el número de moles totales que hay en el equilibrio. ( ) ( ) ( ) ( ) ( ) ( ) ( )=α+α+α−=++= XoXoXoo235T nnnnClnPClnPClnXn ( ) ( ) mol15,05,011,01nnn XoXoo =+⋅=α+=α+= RTnVP XXX =⋅ : atm45,5 L1,2 K532 Kmol Latm 0,082mol15,0 V RTn P X X X = ⋅ ⋅ ⋅ ⋅ == b. La constante Kc, se obtiene por la ley de acción de masas. ( ) ( ) ( ) ( ) ( ) ( ) ( )Xo XoXo 5 23 5 23 5 23 c 1n nn V 1 PCln ClnPCln V 1 V PCln V Cln V PCln PCl ClPCl K α− α⋅α ⋅= ⋅ ⋅= ⋅ = ⋅ = 2 2 X 2 Xo X c 1017,4 5,01 5,01,0 2,1 1 1 n V 1 K − ×= − ⋅ ⋅= α− α ⋅= c. Teniendo en cuenta que el valor de constante solo depende de la temperatura, y que los dos reactores están a la misma temperatura, la constante de equilibrio aplicada al reactor Y permite calcular su volumen. Y 2 Yo Y c 1 n V 1 K α− α ⋅= : L4,19 9,01 9,01,0 1017,4 1 1 n K 1 V 2 2 Y 2 Yo c Y = − ⋅ ⋅ ⋅ = α− α ⋅= − d. Al igual que en el apartado a, la presión en el interior del reactor X se calcula mediante la ecuación de gases ideales. ( ) ( ) ( ) mol19,09,011,01nYn YoT =+⋅=α+= atm43,0 L19,4 K532 Kmol Latm 0,082mol19,0 V RTn P Y Y Y = ⋅ ⋅ ⋅ ⋅ == Modelo 2011. Cuestión 2B.- En sendos recipientes R1 y R2, de 1 L cada uno, se introduce 1 mol de los compuestos A y B, respectivamente. Se producen las reacciones cuya información se resume en la tabla: Reacción Concentración inicial Ecuación cinética reacción directa Constante cinética Constante de equilibrio R1 A ↔ C + D [A]o = 1 M v1 = k1 [A] k1 = 1 s−1 K1 = 50 R2 B ↔ E + F [B]o = 1 M v2 = k2 [B] k2 = 100 s−1 K2 = 2×10−3 Justifique las siguientes afirmaciones, todas ellas verdaderas. b. Cuando se alcance el equilibrio, la concentración de A será menor que la de B. c. Para las reacciones inversas en R1 y R2 se cumple k−1 < k−2. Puntuación máxima por apartado: 0,5 puntos. Solución. b. En este apartado debemos fijarnos en el valor de la constante de equilibrio. Teniendo en cuenta que las concentraciones iniciales son iguales y que K1 > K2, el equilibrio en la reacción 1 (R1) esta mas desplazado hacia la derecha (productos) que el de la reacción 2 (R2), por tanto, en el equilibrio [A] < [B]. c. El equilibrio químico es dinámico, las reacciones directa e inversa no se detienen al llegar al equilibrio sino que sus velocidades se igualan y por tanto las concentraciones de todas las especies presentes en el mismo permanecen constante. En el equilibrio, las concentraciones de A y B ([A], [B]) son menores a las concentraciones iniciales, teniendo en cuenta que la velocidad de reacción es directamente proporcional a la concentración de reactivo como pone de manifiesto la ecuación cinética (v1 = k1 [A]; v2 = k2 [B]), al disminuir las concentraciones, disminuye la velocidad. Septiembre 2010. FM. Problema 2A.- En un recipiente de 14 L de volumen se introducen 3,2 moles de nitrógeno y 3 moles de hidrógeno. Cuando se alcanza el equilibrio a 200°C se obtienen 1,6 moles de amoniaco. a) Formule y ajuste la reacción. b) Calcule el número de moles de H2 y de N2 en el equilibrio.
  • 13. 13 c) Calcule los valores de las presiones parciales en el equilibrio de H2, N2 y NH3 d) Calcule Kc y Kp a 200°C. Dato. R = 0,082 atm·L·mol−1 ·K−1 Puntuación máxima por apartado: 0,5 puntos Solución. a. ( ) ( ) ( )gNH2gH3gN 322 ↔+ b. Para calcular el número de moles en el equilibrio se plantea el cuadro de reacción definiendo x como el número de moles de nitrógeno que reaccionan. Sabiendo que el número de moles de NH3 en el equilibrio (2x) es 1,6, se calcula x. 6,1x2 = : mol8,0x = Conocidos los moles de nitrógeno que han reaccionado, se pueden calcular los moles de cada componente en el equilibrio. ( ) mol4,28,02,3Nn eq2 =−= ( ) mol6,08,033Hn eq2 =⋅−= ( ) mol6,1NHn eq3 = c. Aplicando la ecuación de gases ideales a cada componentes de la mezcla gaseosa, se calculan las presiones parciales. RTnVP ii =⋅ Donde Pi es la presión parcial de componente i y ni los moles de dicho componente. ( ) atm65,6 14 473082,04,2 V RTNn P 2 N2 = ⋅⋅ = ⋅ = ( ) atm66,1 14 473082,06,0 V RTHn P 2 H2 = ⋅⋅ = ⋅ = ( ) atm43,4 14 473082,06,1 V RTNHn P 3 NH3 = ⋅⋅ = ⋅ = d. Con los datos obtenidos en el apartado c se calcula el valor de KP, 65,0 66,165,6 43,4 PP P K 3 2 3 HN 2 NH P 22 3 = ⋅ = ⋅ = Conocido KP se calcula KC mediante la relación entre ellas. ( ) ( )gn CP RTKK ∆ ⋅= ( ) ( ) ( ) 5,970 473082,0 65,0 RT K K 42gn P c = ⋅ == −∆ Septiembre 2010. FM. Cuestión 2B.- La síntesis del amoniaco según la reacción en fase gaseosa, 322 NH2H3N ↔+ , es un buen ejemplo para diferenciar factores cinéticos y termodinámicos. d. Escriba la expresión para KP en función de la presión total. Dato. ( ) 0NHH 3f o <∆ Puntuación máxima por apartado: 0.5 puntos. Solución. d. Para la reacción ( ) ( ) ( )gNH2gH3gN 322 ↔+ , la constante de equilibrio en función de las presiones parciales tiene la expresión:
  • 14. 14 3 HN 2 NH P 22 3 PP P K ⋅ = Teniendo en cuenta que la presión parcial de un componente de una mezcla gaseosa es igual a la presión total por la fracción molar ( )ii PP χ⋅= , se puede expresar Kp en función de la presión total y de la fracción molar de todos los componentes de la mezcla gaseosa. ( ) ( ) 3 HN 2 NH 23 HN 2 NH HH NN NHNH 3 HN 2 NH P 22 3 22 3 22 22 33 22 3 P 1 PP P PP PP PP PP P K χ⋅χ χ ⋅= χ⋅⋅χ⋅ χ⋅ =           χ⋅= χ⋅= χ⋅= = ⋅ = Septiembre 2010. Problema lA.- A 330 K y l atm, 368 g de una mezcla al 50% en masa de N02 y N2O4 se encuentran en equilibrio. Calcule: a) La fracción molar de cada componente en dicha mezcla. b) La constante de equilibrio Kp para la reacción 2 N02 ↔ N204 c) La presión necesaria para que la cantidad de N02 en el equilibrio se reduzca a la mitad. d) El volumen que ocupa la mezcla del apartado c) en el equilibrio. Datos. R = 0,082 atm·L·K−1 ·mol−1 masas atómicas: N = 14; O = 16 Puntuación máxima por apartado: 0.5 puntos. Solución. a. La fracción molar de un componente de una mezcla gaseosa es: T i i n n =χ ; siendo ∑= iT nn ( ) ( ) ( ) ( ) ( ) ( ) ( ) ( )       === === === mol2 mol g29 g184 ONM ONm ONn mol4 mol g46 g184 NOM NOm NOn :g184 2 368 ONmNOm 42 42 42 2 2 2 422 67,0 6 4 24 4 2NO == + =χ ; 33,0 6 2 24 2 42ON == + =χ b. El equilibrio de la reacción de dimerización del dióxido de nitrógeno, viene representado por: { } ( ) 75,0 67,0 33,0 1 1 P 1 P P PP P P K 22 NO ON 2 NO ON ii2 NO ON p 2 42 2 42 2 42 =⋅= χ χ ⋅= χ⋅ χ⋅ =χ⋅=== c. Si la temperatura permanece constante, Kp permanece constante. Para que la cantidad de NO2 se reduzca a la mitad, deben reaccionar dos moles, según la estequiometria del proceso, la nueva composición de equilibrio será: Las nuevas fracciones molares son:      == + =χ == + =χ 60,0 5 3 32 3 40,0 5 2 32 2 42 2 ON NO ; Teniendo en cuenta que la constante de equilibrio no varia, se calcula la nueva presión de equilibrio. 2 NO ON 2 NO ON p 2 42 2 42 P 1 P P K χ χ ⋅== : atm5 40,0 60,0 75,0 1 K 1 P 22 NO ON p 2 42 =⋅= χ χ ⋅=
  • 15. 15 El aumento de presión que experimenta el sistema para desplazarse a la derecha y, de esta forma disminuir el número de moles de NO2, esta de acuerdo con leyes de Lechatelier. Al aumentar la presión el sistema evoluciona hacia donde menor volumen ocupa, contrarrestando de esta forma el aumento de presión. d. Conocida la presión, el número de moles y la temperatura, la ecuación de gases ideales permite calcular el volumen de la mezcla de equilibrio. nRTPV = : ( ) L27 5 330082,032 P nRT V = ⋅⋅+ == Septiembre 2010. FG. Cuestión 2B.- La siguiente descomposición: 2 NaHCO3 (s) → Na2CO3 (s) + H2O (g) + CO2 (g), es un proceso endotérmico. a) Escriba la expresión para la constante de equilibrio Kp de la reacción indicada. b) Razone cómo afecta al equilibrio un aumento de la temperatura. c) Razone cómo afecta a la cantidad de CO2 desprendido un aumento de la cantidad de NaHC03 d) Justifique cómo afecta al equilibrio la eliminación del CO2 del medio. Puntuación máxima por apartado: 0.5 puntos. Solución. a. Equilibrio heterogéneo. Las sustancias sólidas se encuentran en exceso y por tanto sus concentraciones se mantienen constantes y se introducen en el valor de la constante, quedando la constante únicamente en función de las especies en estado gaseoso. 22 COOHp PPK ⋅= b. Según Le Chatelier, al aumentar la temperatura en un sistema, el equilibrio se restablece desplazándose en el sentido en el que se consuma calor (sentido endotérmico), contrarrestando de esta forma el aumento de temperatura. Teniendo en cuenta que la reacción es endotérmica, el sistema se desplaza hacia la derecha (productos), consumiendo calor. c. Por estar en fase sólida, y por tanto no formar parte de la constante, el cociente de reacción no se ve afectado por aumentar la concentración de bicarbonato sódico (NaHCO3), por lo que no desplaza el equilibrio y por tanto no modifica la cantidad de CO2. d. Según Le Chatelier, al introducir una perturbación en un sistema en equilibrio, el sistema reaccionará de forma que se oponga a la perturbación y de esa forma restablecer el equilibrio. Si se elimina dióxido de carbono, el equilibrio se desplazará en el sentido de formación de CO2, es decir, hacia la derecha (productos). Junio 2010. FM. Problema 2A.- Se parte de 150 gramos de ácido etanoico, y se quieren obtener 176 gramos de etanoato de etilo por reacción con etanol. a) Escriba la reacción de obtención del etanoato de etilo indicando de qué tipo es. b) Sabiendo que Kc vale 5, calcule los gramos de alcohol que hay que utilizar. c) Calcule las fracciones molares de cada uno de los 4 compuestos presentes en el equilibrio. Datos. Masas atómicas: C = 12; O = 16; H = 1 Puntuación máxima por apartado: 1 punto. Solución. a. Reacción de esterificación. Adición con eliminación OHCHCHCOOCHOHCHCHCOOHCH 2323233 +−−−↔−+− Ac. Etanoico Etanol Etanoato de etilo Agua b. Se trata de un equilibrio de esterificación en el que se conocen los moles iniciales del ácido, los moles en el equilibrio del ester y la constante de equilibrio. Si denominamos por x a los moles de alcohol iniciales y por y, a los moles de ácido y alcohol que reaccionan, el cuadro de reacción queda de la siguiente forma: ( ) mol5,2 molg60 g150 Ácidon 1o = ⋅ = − ( ) mol2 molg88 g176 Estern 1o = ⋅ = −
  • 16. 16 Aplicando la ley de acción de masas al equilibrio: ( ) ( ) ( ) ( ) ( ) ( ) ( ) ( ) ( ) ( )yxy5,2 yy OHCnOHCn OHnOHCn V OHCn V OHCn V OHn V OHCn OHCOHC OHOHC K 62242 2284 62242 2284 62242 2284 c −⋅− ⋅ = ⋅ ⋅ = ⋅ ⋅ = ⋅ ⋅ = Teniendo en cuenta que los moles en el equilibrio de C4H8O2 son 2 (y = 2), y que Kc = 5: ( )2x5,0 4 5 −⋅ = Despejando se obtiene x = 3,6 mol de C2H6O. Conocidos los moles iniciales de etanol se calcula la masa. ( ) ( ) ( ) g6,165 mol g 46mol6,3OHCMOHCnOHCm 626262 =⋅=⋅= c. Conocidos los moles de todos los componentes en el equilibrio, se calculan las fracciones molares. T i i n n =χ ; 1,6226,15,0nT =+++= ( ) ( ) 0820,0 1,6 5,0 n OHCn OHC T 242 242 ===χ ( ) ( ) 2623,0 1,6 6,1 n OHCn OHC T 62 62 ===χ ( ) ( ) 3279,0 1,6 2 n OHCn OHC T 284 284 ===χ ( ) ( ) 3279,0 1,6 2 n OHn OH T 2 2 ===χ Junio 2010. FG. Problema 2A. En un reactor se introducen 5 moles de tetraóxido de di nitrógeno gaseoso, que tiene en el recipiente una densidad de 2,3 1 Lg − ⋅ . Este compuesto se descompone según la reacción NZO4 (g) ↔ 2 NO2 (g), y en el equilibrio a 325 K la presión es 1 atm. Determine en estas condiciones: a) El volumen del reactor. b) El número de moles de cada componente en el equilibrio. c) El valor de la constante de equilibrio Kp d) El valor de la constante de equilibrio Kc Datos. R = 0,082 11 KmolLatm −− ⋅⋅⋅ ; Masas atómicas: N = 14; O = 16 Puntuación máxima por 'apartado: 0,5 puntos. Solución. a. Aplicando la definición de densidad a las condiciones iniciales del reactor (solo hay N2O4), se obtiene el volumen del reactor. V m d = ; ( ) ( ) L200 Lg3,2 mol g 92mol5 d ONMONn d m V 1 4242 = ⋅ ⋅ = ⋅ == − b. Si se define x como el número de moles de N2O4 que se disocian, el cuadro de reacción queda: Las condiciones del sistema en el punto de equilibrio (V = 200 L; P = 1 atm; T = 298 K), permiten calcular el número de moles totales. mol5,7 K298 Kmol Latm 082,0 L200atm1 TR VP nT = ⋅ ⋅ = ⋅ ⋅ = ( ) ( ) 5,7x5x2x5NOnONnn 242T =+=+−=+= mol5,2x =
  • 17. 17 ( ) mol5,25,25ONn 42 =−= ( ) mol55,22NOn 2 =⋅= c. 42 2 ON 2 NO p P p K = ( ) ( ) ( ) ( ) 33,1 333,0 666,0 K: atm333,0 200 325082,05,2 V RTNOn ONP atm666,0 200 325082,05 V RTNOn NOP 2 p 2 42 2 2 ==      = ⋅⋅ = ⋅ = = ⋅⋅ = ⋅ = d. ( ) ( ) 05,0 325082,0 33,1 RT K K 12n p c −∆ ⋅ == Junio 2010. FG. Cuestión 2B. Considerando el equilibrio existente entre el oxígeno molecular y el ozono, de acuerdo a la reacción 3O2 (g) ↔ 2O3 (g), cuya entalpía de reacción ∆Hr = 284 kJ, justifique: a) El efecto que tendría sobre el equilibrio un aumento de la presión del sistema. b) El efecto que tendría sobre la cantidad de ozono en el equilibrio una disminución de la temperatura. c) El efecto que tendría sobre el equilibrio la adición de un catalizador. d) El efecto que tendría sobre la constante de equilibrio Kp añadir más ozono al sistema. Puntuación máxima por apartado: 0,5 puntos. Solución. Según Le Chàtelier, si un sistema químico en equilibrio experimenta un cambio en la concentración, temperatura, volumen, o la presión parcial, entonces el equilibrio se desplaza para contrarrestar el cambio impuesto y restablecer el equilibrio. a. Un aumento de presión tenderá a llevar al sistema hacia donde menos volumen ocupe y de esta forma contrarrestar el aumento de presión, aplicado al equilibrio oxígeno/ozono, el sistema se desplazará hacia la derecha, aumentando la concentración de ozono y disminuyendo la de oxígeno. b. Al disminuir la temperatura el sistema evoluciona en el sentido en el que produzca calor (sentido exotérmico), teniendo en cuenta que la reacción es endotérmica (∆H = 284 kJ > 0), el sentido exotérmico es hacia la izquierda, aumentando la concentración de oxígeno y disminuyendo la de ozono. c. Los Catalizadores son sustancias ajenas a una reacción cuya presencia modifica la velocidad de la misma sin que la reacción experimenten alteración permanente alguna manteniendo las condiciones iniciales y finales, por lo tanto su adición no perturba el equilibrio. d. Las constantes de equilibrio solo son función de la temperatura, un aumento de la concentración de ozono modificará el cociente de reacción obligando al sistema a evolucionar en contra de dicho aumento pero no modificara el valor de Kp. Modelo 2010. Cuestión 3A.- Dado el equilibrio C (s) + H2O (g) ↔ CO (g) + H2 (g), justifique si son verdaderas o falsas las siguientes afirmaciones: a) La expresión de la constante de equilibrio Kp es: Kp = p(CO) . p(H2) / { p(C) . p(H20) } b) Al añadir más carbono, el equilibrio se desplaza hacia la derecha. c) En esta reacción, el agua actúa como oxidante. d) El equilibrio se desplaza hacia la izquierda cuando aumenta la presión total del sistema. Puntuación máxima por apartado: 0,5 puntos. Solución. a. FALSA. En los equilibrios heterogéneos, la constante solo depende las sustancias que se encuentren en el estado de agregación de mayor libertad, en el caso de equilibrio heterogéneos sólido-gas, las constantes de equilibrio solo depende de las sustancias que estén en estado gaseoso. OH HCO p 2 2 P PP K ⋅ =
  • 18. 18 b. FALSA. En un equilibrio heterogéneo sólido-gas, el equilibrio solo depende de las sustancias que se encuentren en estado gaseoso. c. VERDADERA. El hidrógeno el agua actúa con estado de oxidación +1, capta protones y se reduce a hidrógeno molecular con estado de oxidación 0, actuando el agua como oxidante, mientras que el carbono sólido pierde electrones oxidándose a monóxido de carbono y actuando como reductor. d. VERDADERA. Según Le Chatelier, “Siempre que se modifiquen las condiciones de un sistema en equilibrio se produce un desplazamiento del mismo en el sentido que restablezca la condiciones iniciales”. Al aumentar la presión en un sistema gaseoso, el equilibrio se desplaza hacia donde menor volumen ocupe, oponiéndose de esta forma al aumento de presión e intentar reestablecer el equilibrio. Modelo 2010. Problema 1B.- Una mezcla de 2 moles de N2 y 6 moles de H2 se calienta hasta 700 ºC en un reactor de 100 L, estableciéndose el equilibrio N2 (g) + 3 H2 (g) ↔ 2NH3 (g). En estas condiciones se forman 48,28 g de amoniaco en el reactor. Calcule: a) La cantidad en gramos de N2 y de H2 en el equilibrio. b) La constante de equilibrio Kc. c) La presión total en el reactor cuando se ha alcanzado el equilibrio. Datos. Masas atómicas: N = 14, H = 1; R = 0,082 atm·L·mol−1 ·K−1 . Puntuación máxima por apartado: a) y b) 0,75 puntos; c) 0,5 puntos. Solución. a. Para resolver el problema es conveniente plantear el cuadro de reacción. Si definimos x como el número de moles de N2 que reaccionan, el cuadro queda de la siguiente forma: Conocida la masa de amoniaco en el equilibrio (48,28 gr), se calcula x. ( ) ( ) ( ) ( ) 84,2x2: x2NHn mol84,2 mol gr17 gr28,48 NHM NHm NHn eq3 3 3 eq3 =      = === : 42,1 2 84,2 x == Conocido el valor de x se calcula el número de moles en el equilibrio de N2 y de H2, y a continuación la masa en gramos de cada uno. ( ) 58,042,12x2Nn 2 =−=−= : ( ) gr24,16 mol gr28mol58,0MnNm 2 =⋅=⋅= ( ) 74,142,136x36Hn 2 =⋅−=−= : ( ) gr48,3 mol gr2mol74,1MnHm 2 =⋅=⋅= b. Conocidos los moles de todos los compuestos presentes en el equilibrio y el volumen del reactor, se calcula la constante Kc mediante su definición. ( ) ( ) ( ) 4 3 2 3 22 2 3 3 22 2 3 c 1064,2 100 74,1 100 58,0 100 84,2 V Hn V Nn V NHn HN NH K ×=       ⋅            =       ⋅            = ⋅ = c. Conocidos los moles en el equilibrio de todos los componentes de la mezcla gaseosa el volumen y la temperatura, con la ecuación de gases ideales se calcula la presión en el interior del reactor. ( ) ( ) ( ) mol16,584,274,158,0NHnHnNnn 322T =++=++= K973273700T =+= TRnVP ⋅⋅=⋅ : atm12,4 L100 K973 Kmol Latm 0,082mol16,5 V TRn P = ⋅ ⋅ ⋅ ⋅ = ⋅⋅ =
  • 19. 19 Septiembre 2009. Cuestión 3.- En las siguientes comparaciones entre magnitudes termodinámicas y cinéticas indique qué parte de la afirmación es falsa y qué parte es cierta: a) En una reacción exotérmica tanto la entalpía de reacción como la energía de activación son negativas. b) Las constantes de velocidad y de equilibrio son adimensionales. c) Un aumento de temperatura siempre aumenta los valores de las constantes de velocidad y de equilibrio. d) La presencia de catalizadores aumenta tanto la velocidad de reacción como la constante de equilibrio. Puntuación máxima por apartado: 0,5 puntos. Solución. a. Verdadero en lo referente a la entalpía de reacción, falso en lo referente a la energía de activación. La energía de activación de una reacción química siempre es positiva, no dependiendo de si la reacción es endotérmica o exotérmica, mientras que la entalpía de una reacción exotérmica es negativa. b. Verdadero en lo referente a la constante de equilibrio, falso en lo referente a la constante de velocidad. La constante de equilibrio por definición es adimensional (cociente de concentraciones o de presiones), mientras que la constante de velocidad tiene dimensiones que dependen del orden total de reacción (n). ( ) 11nn1 slmolvelocidadk −−− ⋅⋅= c. Verdadero en lo referente a la constante de velocidad, falso en lo referente a la constante de equilibrio. Según pone de manifiesto la ecuación de Arrhenius, la constante de velocidad es exponencialmente directa a la temperatura RT Ea eAk − ⋅= , si aumenta la temperatura, aumenta la constante. En el caso de la constante de equilibrio, la influencia de la temperatura sobre la constante depende el signo de la entalpía de la reacción: Si 0HR >∆ (Reacción endotérmica), si T aumenta, la constante aumenta Si 0HR <∆ (Reacción exotérmica), si T aumenta, la constante disminuye d. Verdadero en lo referente a la velocidad de reacción, falso en lo referente a la constante de equilibrio. El uso de catalizadores positivos disminuye la energía de activación aumentando la velocidad de reacción, pero no afecta al equilibrio, siendo la constante de equilibrio únicamente función de la temperatura. Septiembre 2009. Problema 2A.- En el proceso Haber-Bosch se sintetiza amoniaco haciendo pasar corrientes de nitrógeno e hidrógeno en proporciones 1:3 (estequiométricas) sobre un catalizador. Cuando dicho proceso se realiza a 500°C y 400 atm. se consume el 43 % de los reactivos, siendo el valor de la constante de equilibrio Kp=1,55×10−5 . Determine, en las condiciones anteriores: a) El volumen de hidrógeno necesario para la obtención de 1 tonelada de amoniaco puro. b) La fracción molar de amoniaco obtenido. c) La presión total necesaria para que se consuma el 60 % de los reactivos. Datos. R = 0,082 atm·L·K−1 ·mol−1 ; Masas atómicas: N = 14, H = 1. Puntuación máxima por apartado: a) y c) 0,75 puntos, b) 0,5 puntos. Solución. a. Si no es el número de moles iniciales N2 que se introducen en el reactor, y el nitrógeno e hidrógeno se introducen en proporciones estequiométricas, 3no serán los moles de hidrógeno iniciales. Si se denomina por α al tanto por uno de nitrógeno que reacciona, el cuadro de reacción queda de la siguiente forma: Conocida la cantidad de amoniaco que se quiere obtener y el porcentaje de reactivos que se consumen, se puede obtener el número de moles iniciales de nitrógeno, y de este, los moles de hidrógeno iniciales. ( ) mol108,58 mol g17 g101000 NHn 3 3 Eq3 ×= × = ( ) α= oEq3 n2NHn : ( ) 2 3 3 Eq3 o Nmol104,68 43,02 108,58 2 NHn n ×= ⋅ × = α = ( ) ( ) mol102,205104,683Nn3Hn 33 o2o2 ×=×⋅=
  • 20. 20 Aplicando la ecuación de gases ideales, se calcula el volumen de hidrógeno necesario para obtener 1 tonelada de amoniaco. TRnVP ⋅⋅=⋅ : 33 3 m32,5L105,32 atm004 K773 Kmol Latm 0,082mol102,205 P TRn V =×= ⋅ ⋅ ⋅ ⋅× = ⋅⋅ = b. ( ) T 3 NH n NHn 3 =χ ( ) ( ) ( ) ( )α−=α−=α+α−+α−=++= 24nn2n4n2n3n3nnNHnHnNnn oooooooo322T ( ) 274,0 43,02 43,0 224 2 24n n2 o o NH3 − = α− α = α− α = α− α =χ c. La presión se puede obtener a partir de Kp teniendo en cuenta que su valor no varía con la P, y que se puede expresar en función de las fracciones molares y de la presión total del sistema. 3 HN 2 NH 23 H 3 N 2 NH 2 3 HN 2 NH p 22 3 22 3 22 3 P 1 PP P PP P K χ⋅χ χ = χ⋅⋅χ⋅ χ⋅ = ⋅ = 3 HN 2 NH p 22 3 K 1 P χ⋅χ χ = ( ) ( ) ( ) ( ) 1428,0 6,024 6,01 24 1 24n 1n 24n nn n Nn o o o oo T 2 N2 = ⋅− − = α− α− = α− α− = α− α− ==χ ( ) ( ) ( ) ( ) 4286,0 6,024 6,033 24 33 24n 33n 24n n3n3 n Hn o o o oo T 2 H2 = ⋅− ⋅− = α− α− = α− α− = α− α− ==χ ( ) 4286,0 6,02 6,0 224 2 24n n2 o o NH3 − = α− α = α− α = α− α =χ atm7,1026 4286,01428,0 4286,0 1055,1 1 P 3 2 5 = ⋅× = − Valor que confirma la Ley de Le Chatelier “Si la presión aumenta, el equilibrio se desplaza hacia la derecha, debido a que como producto ocupa menor volumen” Junio 2009. Cuestión 2.- Para la reacción: a A (g) ↔ B (g) + C (g), el coeficiente estequiométrico a podría tener los valores 1, 2 ó 3. Indique de manera razonada el valor de a, los signos de las magnitudes termodinámicas ∆H0 , ∆S0 y ∆G0 , y el intervalo de temperatura en el que la reacción sería espontánea, para cada uno de los siguientes casos particulares: i) Caso A: La concentración de A en el equilibrio disminuye si aumenta la temperatura o la presión. ii) Caso B: La concentración de A en el equilibrio aumenta si aumenta la temperatura o la presión. Puntuación máxima por apartado: 1,0 punto. Solución. a A (g) ↔ B (g) + C (g) i) Que la concentración de A disminuye respecto de la de equilibrio, indica que el sistema se desplaza a la derecha (hacia productos). Si se aumenta la temperatura, según Le Chatelier, los sistemas evolucionan consumiendo calor, sentido endotérmico, por lo tanto en este caso el sentido endotérmico es hacia la derecha, la reacción es endotérmica (∆∆∆∆H > 0). Al aumentar la presión, el sistema se desplaza hacia donde menor volumen gaseoso ocupe, por lo tanto, la reacción tendrá mayor volumen gaseoso en reactivos que en productos a = 3. 3A(g) ↔ B(g) + C(g) Si a = 3, el sistema estará pasando de 3 componentes gaseosos a dos, se ordena y la entropía disminuye (∆∆∆∆S < 0).
  • 21. 21 Si ∆H > 0, ∆S < 0 y T > 0 (Temperatura absoluta), por la definición de energía libre ∆G = ∆H − T∆S > 0 a cualquier temperatura. La reacción es no espontánea a cualquier temperatura. ii) Si la concentración de A aumenta respecto a la de equilibrio, indica que el sistema se desplaza a la izquierda (hacia reactivos). Si al aumentar la temperatura el sistema se desplaza hacia la izquierda, el sentido endotérmico de la reacción es hacia la izquierda, la reacción es exotérmica (∆∆∆∆H < 0). Si al aumentar la presión es sistema se desplaza a la izquierda será porque en reactivos habrá menor número de moles gaseosos que en productos, a = 1. A(g) ↔ B(g) + C(g). Si a = 1, el sistema está pasando de un componente gaseoso a dos, se desordena, aumenta la entropía (∆∆∆∆S > 0). Si ∆H < 0, ∆S > 0 y T > 0 (Temperatura absoluta), por la definición de energía libre ∆G = ∆H − T∆S < 0 a cualquier temperatura. La reacción es espontánea a cualquier temperatura. Junio 2009. Problema 2A.- El pentacloruro de fósforo se descompone con la temperatura dando tricloruro de fósforo y cloro. Se introducen 20,85 g de pentacloruro de fósforo en un recipiente cerrado de 1 L, y se calientan a 250 °C hasta alcanzar el equilibrio. A esa temperatura todas las especies están en estado gaseoso y la constante de equilibrio Kc vale 0,044. a) Formule y ajuste la reacción química que tiene lugar b) Obtenga la concentración en mol·L−1 de cada una de las especies de la mezcla gaseosa a esa temperatura c) ¿Cuál será la presión en el interior del recipiente? d) Obtenga la presión parcial de Cl2- Datos. R = 0,082 atm-L·K−1 ·mol−l . Masas atómicas: P = 31,0; Cl = 35,5 Puntuación máxima por apartado: 0.5 puntos Solución. a. ( ) ( ) ( )gClgPClgPCl 235 +↔ b. Para resolver el ejercicio es conveniente plantear el siguiente cuadro de reacción, donde Co representa la concentración inicial del pentacloruro de fósforo y x la concentración del mismo que se disocia Aplicando la ley de acción de masas al equilibrio xC x xC xx PCl ClPCl K o 2 o5 23 c − = − ⋅ = ⋅ = ordenando se obtiene una ecuación de segundo grado. 0KCxKx ooc 2 =−+ La concentración inicial del pentacloruro (Co) se calcula a partir del número de moles iniciales y del volumen. ( ) ( ) ( ) M1,0 1 5,208 85,20 V PClM PClm V PCln C 5 5 5o o === Sustituyendo en la ecuación:    −= = =⋅−+ químicosentidoSin09,0x 048,0x :0044,01,0x044,0x2 Concentraciones en el equilibrio:     == =−= M048,0ClPCl M052,0048,01,0PCl 23 5
  • 22. 22 c. La presión en el interior del recipiente se calcula mediante la ecuación de estado de gases ideales. V TRn P ⋅⋅ = ( ) ( ) ( ) mol148,01048,01048,01052,0VPClVPClVPClClnPClnPClnn 335235 =⋅+⋅+⋅=⋅+⋅+⋅=++= atm3,6 1 523082,0148,0 P = ⋅⋅ = d. ( ) atm1,2523082,0048,0TRCl V TRCln P 2 2 =⋅⋅=⋅⋅= ⋅⋅ = Modelo 2009. Cuestión 3.- Dada la reacción endotérmica para la obtención de hidrógeno CH4 (g) ↔ C (s) + 2 H2 (g) a) Escriba la expresión de la constante de equilibrio Kp. b) Justifique cómo afecta un aumento de presión al valor de Kp. c) Justifique cómo afecta una disminución de volumen a la cantidad de H2 obtenida. d) Justifique cómo afecta un aumento de temperatura a la cantidad de H2 obtenida. Puntuación máxima por apartado: 1,0 punto. Solución. a. Equilibrio heterogéneo sólido-gas. La definición de las constantes solo es función de los componentes del equilibrio que estén en fase gaseosa.. 4 2 CH 2 H p P P K = b. Las constantes de equilibrio solo son función de la temperatura, por lo tanto un aumento de presión no afecta al valor de Kp. c. Una disminución de volumen equivale a un aumento de presión. Según Le Chatelier, al aumentar la presión el equilibrio se desplaza en el sentido en el que ocupe menos volumen, en este caso hacia la izquierda (reactivos), disminuyendo la producción de H2. d. Reacción endotérmica (∆H>0), consume calor. Según Le Chatelier, al aumentar la temperatura en un sistema en equilibrio, este se desplaza en el sentido en que consuma calor (sentido endotérmico), en este caso se desplaza hacia la derecha (productos), aumentando la producción de H2. Modelo 2009. Problema 1B.- Un recipiente de 37,5 L, que se encuentra a 343 K y 6 atm, contiene una mezcla en equilibrio con el mismo número de moles de NO2 y N2O4, según la reacción 2 NO2 (g) ↔ N2O4 (g). Determine: a) El número de moles de cada componente en el equilibrio. b) El valor de la constante de equilibrio Kp. c) La fracción molar de cada uno de los componentes de la mezcla si la presión se reduce a la mitad. Dato. R = 0,082 atm-L·K−1 ·mol−l Puntuación máxima por apartado: a) 0,5 puntos; b) y c) 0,75 puntos. Solución. a. Equilibrio homogéneo en fase gaseosa, mezcla binaria. El número de moles del sistema en equilibrio se obtienen mediante la ecuación de gases ideales a partir de las de las condiciones de equilibrio (P, V y T). TRnVP ⋅⋅=⋅ mol8 K343 Kmol Latm 0,082 L37,5atm6 TR VP n ≈ ⋅ ⋅ ⋅ ⋅ = ⋅ ⋅ = Teniendo en cuenta que la mezcla es binaria y que hay igual número de moles de cada uno de los componentes:
  • 23. 23     = =     = =+ mol4n mol4n : nn 8nn 2 2 422 422 NO NO ONNO ONNO b. La constante de equilibrio en función de las presiones viene dado por la expresión: 2 NO ON p 2 42 P P K = Usando la ley de Raoult (Pi = P · χi), las presiones parciales se pueden expresar en función de la presión total y de las fracciones molares. ( ) 2 NO ON 2 NO ON 2 NO ON p 2 42 2 42 2 42 PP P P P K χ⋅ χ = χ⋅ χ⋅ == Si el numero de moles de cada componente de la mezcla binaria es el mismo, las fracciones molares de cada componente serán 0,5. 5,0 8 4 n n χχ T i ONNO 422 ==== 33,0 5,06 5,0 P K 22 NO ON p 2 42 = ⋅ = χ⋅ χ = c. Teniendo en cuenta que Kp no varia con la presión y que las suma de las fracciones molares es la unidad, se puede plantear un sistema para calcular las nuevas fracciones molares. 2 NOON ONNO 2 NO ON ONNO 2 NO ON ONNO 2 NO ON p 242 422 2 42 422 2 42 422 2 42 : 1 1 1 3 33,0 1 P K χ=χ      =χ+χ = χ χ =      =χ+χ χ⋅ χ = =      =χ+χ χ⋅ χ =    −=χ =χ =−χ+χ=χ+χ 62,1 62,0 :01:1 2 2 2222 NO NO NO 2 NO 2 NONO La solución negativa no tiene sentido químico. 0,380,62110,62 2422 NOONNO =−=χ−=χ⇒=χ Los resultado son concordantes con las leyes de Lavoisier, las cuales predicen que al disminuir la presión el sistema se desplaza en el sentido en que aumente su volumen, en este caso a la izquierda (reactivos), aumentando la fracción molar el dióxido de nitrógeno i disminuyendo las del tetraóxido de dinitrógeno. Septiembre 2008. Problema 1B.- El valor de la constante de equilibrio a 700 K para la reacción 2HI (g) ↔ H2 (g) + I2 (g) es 0,0183. Si se introducen 3,0 moles de HI en un recipiente de 5 L que estaba vacío y se deja alcanzar el equilibrio: a) ¿Cuántos moles de I2 se forman? b) ¿Cuál es la presión total? c) ¿Cuál será la concentración de HI en el equilibrio si a la misma temperatura se aumenta el volumen al doble? Datos. R = 0,082 atm·L·K−l ·mol−1 . Puntuación máxima por apartado: a) 1 punto, b) y c) 0,5 puntos. Solución. a. Equilibrio homogéneo en fase gaseosa del que se conoce los moles iniciales de ioduro de hidrogeno y la constante de equilibrio. Teniendo en cuenta la estequiometria de reacción, si 2x son los moles de ioduro de hidrógeno disociado, x serán los moles de yodo e hidrógeno formados, el cuadro de reacción queda de la siguiente forma:
  • 24. 24 Según la ley de acción de masas: 2 22 HI IH K ⋅ = Debido a que en el equilibrio no hay variación en el número de moles gaseosos entre reactivos y productos, la constante se puede expresar en función del número de moles. ( ) ( ) ( ) ( ) ( ) ( )( ) ( ) ( ) 2 2 2 22 22 2 22 x23 x x23 x x23 xx HIn InHn V HIn V In V Hn K       − = − = − ⋅ = ⋅ =       ⋅ = Operando se despeja x 2 2 Idemoles32'0 0183,021 0183.03 K21 K3 xK x23 x x23 x K = + = + =⇒= − ⇒      − = b. La presión de equilibrio se calcula mediante la ecuación de gases ideales teniendo en cuenta que, en el proceso de disociación no hay variación del número de moles entre productos y reactivos, y por tanto, el número de moles gaseosos en el equilibrio coinciden con el número de moles iniciales. atm44,34 L5 K700 Kmol Latm 0,082mol3 V TRn V TRn P oEq Eq = ⋅⋅ = ⋅⋅ = ⋅⋅ = c. El equilibrio no se modifica por variaciones de presión o de volumen ya que ∆n(g) = 0, pero al aumentar el volumen, disminuye la concentración. ( ) 1 o Lmol236,0 52 32,023 V2 x23 V HIn HI − = ⋅ ⋅− = − == Junio 2008. Cuestión 3.- Considerando la reacción 2 SO2 (g) + O2 (g) ↔ 2 SO3 (g) razone si las siguientes afirmaciones son verdaderas o falsas. a) Un aumento de la presión conduce a una mayor producción de SO3. b) Una vez alcanzado el equilibrio, dejan de reaccionar las moléculas de SO2 y O2 entre sí. c) El valor de Kp es superior al de Kc, a temperatura ambiente. d) La expresión de la constante de equilibrio en función de las presiones parciales es: Kp = P2 (SO2)·P(O2)/P2 (SO3) Dato. R = 0,082 atm·L·K−1 ·mo1 Puntuación máxima por apartado: 0,5 puntos. Solución. a. VERDADERO. Según el principio de Le Chatelier, siempre que se modifiquen las condiciones de un sistema en equilibrio se produce un desplazamiento del mismo en el sentido en el que se restablezcan las condiciones iniciales. Sí se aumenta la presión, la reacción se desplaza en el sentido de originar aquellas sustancias que ocupen menor volumen, es decir, el sistema se desplaza hacia donde el número de moles gaseosos sea menor, en nuestra reacción hacia la derecha, aumentando la producción de SO3. b. FALSO. El equilibrio químico es un equilibrio dinámico. Un sistema llega al equilibrio cuando las velocidades de reacción directa e inversa se igual, ambas reacciones (directa e inversa) se siguen produciendo, pero las concentraciones de la especies presente en el sistema no varían en el tiempo. c. FALSO. Según la relación entre Kp y Kc: ( ) ( )gn cp RTKK ∆ ⋅= ∆n(g) = n(g)R − n(g)P = 2 − (2 + 1) = −1 ( ) RT K K:RTKK c p 1 cp =⋅= − Teniendo en cuenta que RT > 0 cp KK <
  • 25. 25 d. FALSO. Teniendo en cuenta la ley de acción de masas aplicada a equilibrios gaseosos en función de las presiones: 22 3 O 2 SO 2 SO p PP P K = Septiembre 2008. Cuestión 3.- La reacción 2H2O (1) ↔ 2H2 (g) + O2 (g) no es espontánea a 25°C. Justifique si las siguientes afirmaciones son verdaderas o falsas. b. Se cumple que Kp/Kc = RT. c. Si se duplica la presión de H2, a temperatura constante, el valor de Kp aumenta. Puntuación máxima por apartado: 0’5 puntos. Solución. b. Falso. Las constantes de equilibrio Kc y Kp están relacionadas por la expresión: ( ) ( )gn cp RTKK ∆ ⋅= Donde ∆n (g) es la diferencia del número de moles gaseosos entre productos y reactivos ( ) ( ) 3012)reactivos(n)productos(ngn gg =−+=−=∆ ∑∑ Sustituyendo en la expresión y ordenando: ( )3 cp RTKK ⋅= ⇒ ( )3 c p RT K K = c. Falso. La constante de equilibrio Kp no depende de las presiones parciales de los componentes, para cada equilibrio depende únicamente de la temperatura. Septiembre 2007. Problema 1B. En un recipiente de 25 L se introducen dos moles de hidrógeno, un mol de nitrógeno y 3,2 moles de amoniaco. Cuando se alcanza el equilibrio a 400°C, el número de moles de amoniaco se ha reducido a 1,8. Para la reacción 3H2 (g) + N2 (g) ↔ 2NH3 (g) calcule: a) El número de moles de H2 y de N2 en el equilibrio. b) Los valores de las constantes de equilibrio Kc y Kp a 400°C. Datos. R = 0,082 atm·L·mo1−1 ·K−1 . Puntuación máxima por apartado: 1’0 punto. Solución. a. Los datos del enunciado indican que la reacción se desplaza hacia la izquierda para alcanzar el equilibrio, cosa que ocurre cuando el cociente de reacción Q > KEq. Si se supone que desaparecen 2x moles de amoniaco, y teniendo en cuenta la estequiometria de la reacción, el cuadro de reacción queda de la siguiente forma: El número de moles de amoniaco en el equilibrio es un dato: ( ) 8'1x22'3NHn Eq3 =−= Expresión de la que se puede despejar x, y de esta forma calcular los moles en el equilibrio de N2 e H2. moles7'0 2 8'12'3 x = − = ( ) moles7'17'01Nn 2 =+= ( ) moles1'47'032Hn 2 =⋅+= b. Según la ley de acción de masas la constante de equilibrio en función de las concentraciones para la síntesis del amoniaco es:
  • 26. 26 ( ) ( ) ( ) 28'17 25 1'4 25 7'1 25 8'1 V Hn V Nn V NHn HN NH K 3 2 3 eq2eq2 2 eq3 3 22 2 3 c =       ⋅            =         ⋅                 = ⋅ = Kp se calcula a partir de la relación entre Kc y Kp. ( ) ( )gn cp RTKK ∆ ⋅= Donde ∆n (g) es la diferencia del número de moles gaseosos entre productos y reactivos ( ) ( ) 2312)reactivos(n)productos(ngn gg −=+−=−=∆ ∑∑ ( ) ( ) 322 cp 1067'5673082'028'17RTKK −−− ×=⋅⋅=⋅= Junio 2007. Problema 1B.- A temperatura elevada, un mol de etano se mezcla con un mol de vapor de ácido nítrico, que reaccionan para formar nitroetano (CH3CH2N02) gas y vapor de agua. A esa temperatura, la constante de equilibrio de dicha reacción es Kc = 0,050. a) Formule la reacción que tiene lugar. b) Calcule la masa de nitroetano que se forma. c) Calcule la entalpía molar estándar de la reacción. d) Determine el calor que se desprende o absorbe hasta alcanzar el equilibrio. Datos. Masas atómicas: H = 1, C = 12, N = 14, 0=16. Etano (g) Ác. nítrico (g) Nitroetano (g) Agua (g) ∆Hºf (kJmol−l ) −124,6 −164,5 −236,2 −285,8 Puntuación máxima por apartado: 0.5 puntos. Solución. a) ( ) ( ) ( ) ( )gOHgNOCHCHgHNOgCHCH 2223333 +−↔+− b) Para calcular la masa de nitroetano en el equilibrio se plantea el cuadro de reacción y se aplican las condiciones de equilibrio a la constante Kc. Sea x los moles de etano que reaccionan: 333 2223 333 2223 HNOCHCH OHNOCHCH HNOCHCH OHNOCHCH 333 2223 c nn nn V n V n V n V n HNOCHCH OHNOCHCH K ⋅ ⋅ = ⋅ ⋅ = ⋅ ⋅ = El equilibrio no depende del volumen. Sustituyendo los valores en el equilibrio, se despeja x (número de moles de etano que han reaccionado que coincide con los moles de nitroetano formados) ( ) ( ) ( )2 2 x1 x x1x1 xx 050'0 − = −⋅− ⋅ = Operando se despeja x: 18'0 05'01 05'0 x;05'0 x1 x ;050'0 x1 x 2 = + == − =      − NOTA: Solo tiene sentido químico el valor positivo de la raíz. Conocidos los moles en el equilibrio, se calcula la masa. ( ) ( ) ( ) gr5'13 mol gr75mol18'0NOCHCHMNOCHCHnNOCHCHm 223223223 =⋅=⋅=
  • 27. 27 c) Por tratarse la entalpía de una función de estado, sus variaciones solo depende de las condiciones iniciales y finales, por lo tanto ∆HR se calcula mediante la ley de Hess. ( ) ( )∑∑ ∆−∆=∆ ReactivosHProductosHH o f o f o R ( ) ( ) ( ) ( )( )3 o f33 o f2 o f223 o f o R HNOHCHCHHOHHNOCHCHHH ∆+∆−∆+∆=∆ ( ) ( )( ) 1o R molkJ9'2325'1646'1248'2852'236H − −=−−−−−+−=∆ Reacción EXOTÉRMICA. d) o RHnQ ∆⋅=∆ Donde n es el número de moles de etano que han reaccionado ( ) kJ9'41molkJ9'232mol18'0Q 1 −=−⋅=∆ − Hasta alcanzar el equilibrio, se desprende 41’9 kJ. Modelo 2007. Cuestión 3.- El cloruro de plata (I) es una sal muy insoluble en agua. a) Formule el equilibrio heterogéneo de disociación. b) Escriba la expresión de la constante del equilibrio de solubilidad (Ks) y su relación con la solubilidad molar (s). c) Dado que la solubilidad aumenta con la temperatura, justifique si el proceso de disolución es endotérmico o exotérmico. d) Razone si el cloruro de plata (I) se disuelve más o menos cuando en el agua hay cloruro de sodio en disolución. Puntuación máxima por apartado: 0,5 puntos. Solución. a) ( ) ( ) ( )aqClaqAgsAgCl −+ +↔ b) La constante de equilibrio solo depende de las concentraciones de los componentes que estén en el estado de agregación de mayor libertad. En el equilibrio sólido/líquido, la constante (Ks, constante ó producto de solubilidad) solo depende de las concentraciones de los que estén en fase líquida o disuelta (acuosa). −+ ⋅= ClAgKs Si se define s como la cantidad de moles por litro que se disuelven de la sal AgCl, por estequiometria, se formaran s moles por litro de Ag+ y Cl− . Sustituyendo las concentraciones en la expresión de la constante: 2 s sssK =⋅= c) El aumento de la solubilidad indica que el equilibrio se desplaza hacia la derecha (productos), Si al aumentar la temperatura el equilibrio se desplaza hacia la derecha es porque la reacción es endotérmica (∆H>0, absorbe calor). d) Se puede explicar de dos formas: Por leyes de equilibrio, al aumentar la concentración de un producto (Cl− ). El equilibrio se desplaza hacia la izquierda, disminuyendo la solubilidad de la sal. Efecto ión común Modelo 2007. Problema 2B.- A 400 ºC y 1 atmósfera de presión el amoniaco se encuentra disociado en un 40%, en nitrógeno e hidrógeno gaseosos, según la reacción ( ) ( ) ( )gN1/2gH3/2gNH 223 +↔ . Calcule: a) La presión parcial de cada uno de los gases en el equilibrio. b) El volumen de la mezcla si se parte de 170 g de amoniaco. c) El valor de la constante Kp. d) El valor de la constante Kc. Datos.- -1-1 KmolLatm0,082R ⋅⋅⋅= ; masas atómicas: N = 14, H = 1 Puntuación máxima por apartado: 0,5 puntos. Solución. a) Según la ley Raoult, la presión parcial de un componente de una mezcla gaseosa viene dada por la expresión: ii PP χ⋅= Donde P es la presión total, y iχ es la fracción molar del componente i.
  • 28. 28 T i i n n =χ Para conocer el número de moles de cada especie en el equilibrio, se hace el siguiente cuadro, teniendo en cuenta las relaciones estequiométricas y, definiendo no como el número de moles iniciales y α como el grado de disociación. o Disociados n n =α Según el cuadro, el número de moles totales es: ( ) ( ) ( ) ( )α+=α+=α+α+α−=++= 1nnnn 2 3 n 2 1 nnHnNnNHnn ooooooo223T Conocido el número de moles totales y los moles de cada componente, se calculan las fracciones molares. ( ) ( ) ( ) ( ) ( ) ( ) ( ) ( ) ( ) ( ) ( )α+ α = α+ α =χ α+ α = α+ α =χ α+ α− = α+ α− =χ 12 3 1n n 2 3 H: 121n n 2 1 N: 1 1 1n 1n NH o o 2 o o 2 o o 3 Para α = 0’40: ( ) ( ) ( ) ( ) ( ) ( ) ( ) 43'0 4'012 4'03 H:0'14 4'012 4'0 N:0'43 4'01 4'01 NH 223 = + ⋅ ==χ= + =χ= + − =χ Con las fracciones molares y la presión total del sistema, se calculan las presiones parciales de cada componente en la mezcla. atm43'043'0atm1PP atm14'014'0atm1PP atm43'043'0atm1PP 22 22 33 HH NN NHNH =⋅=χ⋅= =⋅=χ⋅= =⋅=χ⋅= b) El volumen de la mezcla si se parte de 170 g de amoniaco.A partir de la ecuación de gases ideales: P nRT V = Donde n es el número total de moles de la mezcla en equilibrio, que se calcula a partir del número de moles iniciales de amoniaco según la expresión:. ( ) ( ) ( ) ( ) ( ) mol1440'01 mol gr17 gr170 1 NHM NHm 1nn 3 3o oT =+=α+=α+= Aplicando la ecuación de gases ideales: ( ) L6'772 atm1 K673 Kmol Latm 082'0mol14 V = ⋅ ⋅ ⋅ ⋅ = c) Para la reacción ( ) ( ) ( )gN1/2gH3/2gNH 223 +↔ , se define la constante de equilibrio en función de las presiones (Kp) como: 245'0 43'0 14'043'0 P PP K 2 1 2 3 NH 2 1 N 2 3 H p 3 22 = ⋅ = ⋅ = d) Kc y Kp se relacionan por la siguiente expresión:
  • 29. 29 ( ) ( )gn pc RTKK ∆− ⋅= Donde ∆n(g) es la diferencia de numero de moles gaseosos entre productos y reactivos. ( ) 11 2 1 2 3 gn =−+=∆ ( ) 31 c 104'4673082'0245'0K −− ×=⋅⋅= Septiembre 2006. Cuestión 3.- El amoniaco reacciona a 298 K con oxígeno molecular y se oxida a monóxido de nitrógeno y agua, siendo su entalpía de reacción negativa. a) Formule la ecuación química correspondiente con coeficientes estequiométricos enteros. b) Escriba la expresión de la constante de equilibrio Kc. c) Razone cómo se modificará el equilibrio al aumentar la presión total a 298 K si son todos los compuestos gaseosos a excepción del H2O que se encuentra en estado líquido. d) Explique razonadamente cómo se podría aumentar el valor de la constante de equilibrio. Puntuación máxima por apartado: 0’5 puntos. Solución. a. ( ) ( ) ( ) ( )lOH6gNO4gO5gNH4 223 +↔+ b. Por tratarse de un equilibrio heterogéneo (líquido-gas), la constante de equilibrio solo es función de los componentes en estado gaseoso. 5 2 4 3 4 ONH NO Kc = c. Según Le Chatelier al aumentar la presión, la reacción se desplaza en el sentido de originar aquellas sustancias que ocupen menos volumen. En la reacción propuesta, al aumentar la presión el equilibrio se desplaza hacia PRODUCTOS (derecha), por haber nueve moles gaseosos en reactivos y cuatro moles gaseosos en productos. d. Por ser una reacción exotérmica (∆H<0, desprende calor), al disminuir la temperatura el equilibrio se desplaza en el sentido en el que se desprenda calor, hacia productos (derecha), ( ) ( ) ( ) ( ) QlOH6gNO4gO5gNH4 223 ++↔+ aumentando las concentraciones de productos y disminuyendo la de reactivos, y por lo tanto aumentado el valor de la constante de equilibrio. Junio 2006. Problema 1B.- En un recipiente de 0,4 L se introduce 1 mol de N2 y 3 mol de H2 a la temperatura de 780 K. Cuando se establece el equilibrio para la reacción N2 + 3 H2 ↔ 2 NH3, se tiene una mezcla con un 28 % en mol de NH3. Datos.- R = 0,082 atm·L·K−1 ·mol−1 . Determine: a) El número de moles de cada componente en el equilibrio. b) La presión final del sistema. c) El valor de la constante de equilibrio, Kp. Datos.- R = 0,082 atm·L·K−1 ·mol−1 . Puntuación máxima por apartado: a) y c) 0,75 puntos y b) 0,5 puntos. Solución. a. Para el equilibrio de formación del amoniaco, conocidos los moles iniciales de hidrógeno y nitrógeno y la fracción molar del amoniaco en la mezcla resultante, se pide calcular la composición en moles de la mezcla en equilibrio. Si definimos x como los moles de hidrógeno que reaccionan, el cuadro de reacción es el siguiente:
  • 30. 30 ( ) ( ) ( ) ( ) ( ) ( ) ( ) ( ) 28'0 x24 x2 x2x33x1 x2 NHnHnNn NHn n NHn 322 3 T 3 NH3 = − = +−+− = ++ ==χ Despejando: x = 0’44 ( ) ( ) ( ) 88'044'02x2NHn 68'144'033x33Hn 56'044'01x1Nn eq3 eq2 eq2 =⋅== =⋅−=−= =−=−= b. Con el número total de moles el volumen y la temperatura, mediante la ecuación de gases ideales se calcula la presión en el equilibrio. moles12'344'024x24nT =⋅−=−= atm499 L4'0 K780 Kmol Latm 082'0mol12'3 V nRT P ≈ ⋅ ⋅ ⋅ ⋅ == c. Para el equilibrio de formación el NH3 se define Kp como: 3 HN 2 NH p 22 3 PP P K ⋅ = Teniendo en cuenta la ley de Raoult ( )ii PP χ⋅= , se puede expresar Kp en función de la presión total y de las fracciones molares de los componentes gaseosos del equilibrio. 3 HN 2 NH 23 H 3 N 2 NH 2 p 22 3 22 3 P 1 PP P K χ⋅χ χ = χ⋅⋅χ⋅ χ⋅ = ( ) ( ) 54'0 12'3 68'1 n Hn 18'0 12'3 56'0 n Nn 28'0 T 2 H T 2 NNH 223 ===χ===χ=χ 5 3 2 23 HN 2 NH 2p 101'1 54'018'0 28'0 500 1 P 1 K 22 3 − ×= ⋅ ⋅= χ⋅χ χ = Modelo 2006. Cuestión 3.- Al calentar, el dióxido de nitrógeno se disocia en fase gaseosa en monóxido de nitrógeno y oxígeno: a) Formule la reacción química que tiene lugar. b) Escriba Kp para esta reacción. c) Explique el efecto que produce un aumento de presión total sobre el equilibrio. d) Explique como se verá afectada la constante de equilibrio al aumentar la temperatura. Puntuación máxima por apartado: 0,5 puntos. Solución. a. ( ) ( ) ( )gO 2 1 gNOgNO 22 +⇔ b. 2 2 1 ONO NO1 PP Kp 2 ⋅ = c. Al aumentar la presión en un sistema en equilibrio, este se desplaza hacia donde menor volumen ocupe, y de esa forma se contrarresta el aumento de presión, en este caso, el sistema se desplaza hacia los reactivos(izquierda). d. Al aumentar la temperatura se ve favorecido el proceso endotérmico y el equilibrio se desplaza hacia la derecha ya que la reacción es endotérmica ( ( )0H >∆ , aumentando el valor de Kp.
  • 31. 31 Modelo 2006. Problema 2B.- Se introduce en un recipiente de 3 L, en el que previamente se ha hecho el vacío, 0’04 moles de SO3 a 900 K. Una vez alcanzado el equilibrio, se encuentra que hay presentes 0,028 moles de SO3. a) Calcule el valor de Kc para la reacción: 2SO3 (g) ↔2SO2 (g) + O2 (g) a dicha temperatura. b) Calcule el valor de Kp para dicha disociación. Dato.- R = 0,082 atm • L • mol-1 K-1 Puntuación máxima por apartado: 1,0 punto. Solución. a. mol0'006xmol012'0028'004'0x2 =⇒=−= conocido el valor de x se conocen el número de moles de cada especie presente en el equilibrio. n(SO3) = 0’028 n(SO2) = 0’012 n(O2) = 0’006 La expresión de Kc se puede poner en función del número de moles y del volumen ( ) ( ) ( ) 2 3 2 2 2 2 2 3 2 2 2 c V SOn V On V SOn SO OSO K       ⋅      = ⋅ = Simplificando volúmenes se obtiene la siguiente expresión: ( ) ( ) ( ) ( )2 2 3 2 22 2 c 028'0 006'0012'0 3 1 SOn OnSOn V 1 K ⋅ ⋅= ⋅ ⋅= 4 c 107'3K − ⋅= b. ( ) ( ) ( ) ( ) ( )RTKcKp 1212gn RTKcKp gn ⋅=     =−+=∆ ⋅= ∆ ( ) 027'0900082'01067'3Kp 4 =⋅⋅= − Septiembre 2005. Problema 2A. Para la reacción N2 (g) + O2 (g) → 2 NO (g) el valor de la constante de equilibrio, Kc, es 8,8×10−4 a 1930 °C. Si se introducen 2 moles de N2 y 1 mol de O2 en un recipiente vacío de 2 L y se calienta hasta 1930 °C, calcule: a) La concentración de cada una de las especies en equilibrio. b) La presión parcial de cada especie y el valor de la constante de equilibrio Kp. Datos.- R = 0’082 atm·L·mo1−1 ·K−1 Puntuación máxima por apartado: 1,0 punto. Solución. a. Para la reacción de formación del monóxido de nitrógeno se plantea el cuadro de reacción. A partir de la ley de Acción de Masas se obtiene la expresión de la constante que rige el equilibrio en función del número de moles.
  • 32. 32 ( ) ( ) ( ) ( ) ( ) ( ) ( ) ( ) ( ) 4 2 22 22 2 22 2 22 2 c 108'8 2x3x x4 x1x2 x2 OnNn NOn v On v Nn v NOn ON NO K − ×= +− = −⋅− = ⋅ = ⋅       = ⋅ = Expresión que se puede transformar en una ecuación de segundo grado. ( ) 01076'1x1064'2x108'84 3324 =×−×+×− −−− despreciando 8’8×10−4 frente a 4, la ecuación queda de la forma: 01076'1x1064'2x4 332 =×−×+ −− cuyas soluciones son:    −= = 02'0x 02'0x despreciando la solución negativa por carecer de sentido químico, el número de moles en el equilibrio queda de la siguiente forma Conocidos los moles en el equilibrio y el volumen se calculan las concentraciones en el equilibrio. ( ) l mol99'0 2 98'1 v Nn N 2 2 === ( ) l mol49'0 2 98'0 v On O 2 2 === ( ) l mol02'0 2 04'0 v On NO 2 === b. Partiendo de la ecuación de gases ideales aplicada a cada componente del equilibrio, se obtienen sus presiones parciales. RTnVP ii =⋅ : RT V n P i i = : RTCP ii = ( ) atm9'178K2203 Kmol latm 082'0 l mol 99'0RTNP 2N2 =⋅      ⋅ ⋅ ⋅      =⋅= ( ) atm5'88K2203 Kmol latm 082'0 l mol 49'0RTOP 2O2 =⋅      ⋅ ⋅ ⋅      =⋅= ( ) atm6'3K2203 Kmol latm 082'0 l mol 02'0RTNOPNO =⋅      ⋅ ⋅ ⋅      =⋅= Para calcular el valor de Kp, se tiene en cuenta la relación entre las constantes Kp y Kc ( ) ( ) 4 c 0 c n cp 108'8KRTKRTKK −∆ ×==⋅=⋅= Junio 2005. Cuestión 3.- El dióxido de nitrógeno es un gas que se presenta en la forma monómera a 100 ºC. Cuando se disminuye la temperatura del reactor hasta 0 ºC se dimeriza para dar tetraóxido de dinitrógeno gaseoso. a) Formule el equilibrio químico correspondiente a la reacción de dimerización. b) ¿Es exotérmica o endotérmica la reacción de dimerización? c) Explique el efecto que produce sobre el equilibrio una disminución del volumen del reactor a temperatura constante. d) Explique cómo se verá afectado el equilibrio si disminuye la presión total, a temperatura constante. Puntuación máxima por apartado: 0,5 puntos.
  • 33. 33 Solución. a. ( ) ( )gONgNO2 422 ↔ b. Al disminuir la temperatura el equilibrio se desplaza hacia la derecha, lo cual, indica que se absorbe calor en sentido contrario. La reacción es EXOTÉRMICA. 0HR <∆ ( ) ( ) QgONgNO2 422 +↔ c. Al disminuir el volumen del reactor a temperatura constante aumenta la presión del sistema. y el equilibrio se desplaza hacia donde menor volumen ocupa, hacia la derecha, favoreciendo la dimerización. d. Si disminuye la presión, el equilibrio se desplaza hacia donde más volumen ocupa, hacia la izquierda. No favorece la dimerización Junio 2005. Problema 2B.- Se introducen 2 moles de COBr2 en un recipiente de 2 L y se calienta hasta 73 ºC. El valor de la constante Kc, a esa temperatura, para el equilibrio COBr2(g) ⇔ CO(g) + Br2(g) es 0,09. Calcule en dichas condiciones: a) El número de moles de las tres sustancias en el equilibrio. b) La presión total del sistema. c) El valor de la constante Kp Dato.- R = 0,082 atm · L · mol-1· K-1 Puntuación máxima por apartado: a) 1,0 punto; b) y c) 0,5 puntos. Solución. a. A partir de la expresión de la constante de equilibrio en función de las concentraciones, se puede encontrar una relación entre la constante y el número de moles de cada especie en equilibrio. ( ) ( ) ( ) ( ) ( ) ( )2 2 2 2 2 2 c COBrn BrnCOn V 1 V COBrn V Brn V COn COBr BrCO K ⋅ ⋅= ⋅ = ⋅ = sustituyendo por los datos del equilibrio 18'0 x2 x x2 xx 2 1 09'0K 2 c = −− ⋅ ⋅== ordenando    −= = =−+ 7'0x 52'0x :036'0x18'0x2 Se desprecia la solución negativa por no tener sentido químico, por lo que en el equilibrio queda: ( ) ( ) ( ) moles48'152'02COBrn moles52'0BrnCOn 2 2 =−= == b. Se obtiene a partir de la ecuación de gases ideales. V TRn PTRnVP T ⋅⋅ =⋅⋅=⋅ ( ) atm7'35 2 346082'052'0248'1 P = ⋅⋅⋅+ = c. Se obtiene a partir de la relación entre Kp y Kc. ( ) ( ) ( )( ) 55'2346082'009'0RTKK 111gn cp =⋅⋅=⋅= −+∆ Modelo 2005. Cuestión 3.- Para la reacción de síntesis del amoníaco, N2{g) + 3H2{g) ↔ 2NH3{g), se conocen los
  • 34. 34 valores, a temperatura ambiente, de las siguientes magnitudes: o rH∆ (valor negativo), o rG∆ (valor negativo), Kp (valor muy alto) y Ea (valor muy alto). Conteste a las siguientes preguntas, indicando cuál o cuáles de dichas magnitudes están directamente relacionadas con los conceptos que se enumeran a continuación: Puntuación máxima por apartado:0,5 puntos. d. Efecto de la presión. ¿Qué efecto tiene para esta reacción un aumento de presión? Solución. d. Relacionada con la constante del equilibrio a través del cociente de reacción y por tanto de la estequiometria. Si aumenta la presión el cociente de reacción disminuye y por tanto el sistema se desplaza hacia la derecha para restablecer el valor de la constante de equilibrio. 3 HN 2 NH 23 HN 2 NH p 22 3 22 3 χχ χ P 1 PP P K ⋅ ⋅= ⋅ = Modelo 2005. Problema 1B.- En un recipiente cerrado, a la temperatura de 490 K, se introduce 1 mol de PCl5(g) que se descompone parcialmente según la reacción PCl5(g) ↔ PCl3 (g) + Cl2 (g). Cuando se alcanza el equilibrio, la presión es de 1 atm y la mezcla es equimolecular (igual número de moles de PCl5, PCl3 y Cl2). a) Determine el valor de la constante de equilibrio, Kp, a dicha temperatura. b) Si la mezcla se comprime hasta 10 atm, calcule la nueva composición de equilibrio. Puntuación máxima por apartado: 1,0 punto. Solución. a. Teniendo en cuenta que la mezcla es equimolecular: ( ) ( ) ( )235 ClnPClnPCln == moles0'5xxx1 ==− Si el número de moles de los tres componentes del equilibrio son iguales, sus fracciones molares también lo serán. Teniendo en cuenta además que la suma de todas las fracciones molares de una mezcla gaseosa es 1, se puede concluir que en este equilibrio la fracción molar de cada componente es de un tercio 3 1 235 ClPClPCl =χ=χ=χ 033' 3 1 P P PP P PP K 5 23 5 23 5 23 PCl ClPCl PCl ClPCl PCl ClPCl p == χ χ⋅χ = χ⋅ χ⋅⋅χ⋅ = ⋅ = Solución. b. La expresión de la constante Kp para la reacción de disociación del pentacloruro de fósforo es: 5 23 5 23 5 23 PCl ClPCl PCl ClPCl PCl ClPCl p P P PP P PP K χ χχ ⋅= χ⋅ χ⋅⋅χ⋅ = ⋅ = El número total de moles en el equilibrio es: ( )α−⋅=α+=α+α+α−= 1nn:nnnnnnn oTooooooT Conocido el número de moles totales se calculan las fracciones molares en función de α.
  • 35. 35 ( ) ( ) ( ) ( ) ( ) ( )      α+ α = α+ α ===χ=χ α+ α− = α+ α− ==χ =χ 11n n n Cln n PCln 1 1 1n 1n n PCln : n n o o T 2 T 3 ClPCl o o T 5 PCl T i i 23 5 Sustituyendo las fracciones molares en Kp, se obtiene la expresión buscada: 2 2 PCl ClPCl p 1 P 1 1 11PPK 5 23 α− α ⋅= α+ α− α+ α ⋅ α+ α ⋅= χ χχ ⋅= expresión que relaciona Kp α y P. Teniendo en cuenta que Kp no depende de la presión y que por tanto permanece invariable si se aumenta la presión, se puede calcular el nuevo α. 18'0: 1 1033'0 2 2 =α α− α ⋅= Conocido α se calculan las fracciones molares en el nuevo equilibrio 15'0 1 7'0 18'01 18'01 1 1 23 5 ClPCl PCl = α+ α =χ=χ = + − = α+ α− =χ Septiembre 2004. Cuestión 1.- La reacción de obtención del polietileno a partir de eteno, [ ] )s(CHCH)g(CHCHn n2222 −−−⇔= es exotérmica: a) Escriba la expresión de la constante de equilibrio KP. b) ¿Qué tipo de polimerización se produce? c) ¿Cómo afecta un aumento de la temperatura a la obtención de polietileno? d) ¿Cómo afecta un aumento de la presión total del sistema a la obtención de polietileno? Puntuación máxima por apartado: 0’5 puntos. Solución. a. Teniendo en cuenta que es un equilibrio heterogéneo (sólido /gas) la constante es función únicamente de los componentes de equilibrio que estén en fase gas, y por tanto su expresión es: n 22 CHCH 1 Kp = = b. Reacción de polimerización por adición. c. Teniendo en cuenta que la reacción es exotérmica, desprende calor, al aumentar la temperatura el equilibrio se desplazará en el sentido endotérmico, consumiendo calor y contrarrestando el aumento de temperatura, como la reacción es exotérmica, se desplazará hacia la izquierda (reactivos) disminuyendo la obtención de polietileno. d. Favorece la obtención de acetileno ya que al aumentar la presión el sistema se desplaza hacia donde menor volumen ocupa, contrarrestando el aumento de presión, en este caso hacia la derecha (productos). Septiembre 2004. Problema 2A. En un reactor de 1 L, a temperatura constante, se establece el equilibrio NO2 + SO2 ⇔ NO + SO3 siendo las concentraciones molares en l equilibrio: 2'0NO2 = , 6'0SO2 = , 0'4NO = y 2'1SO3 = . a) Calcular el valor de KC a esa temperatura. b) Si se añaden 0’4 moles de NO2 ¿Cuál será la nueva concentración de reactivos y productos cuando sé reestablezca de nuevo el equilibrio? Puntuación máxima por apartado: 1,0 punto. Solución. a. Según la ley de acción de masas, la constante del equilibrio tiene la siguiente expresión.
  • 36. 36 22 3 c SO·NO SO·NO K = Sustituyendo los valores de las concentraciones en equilibrio: 40 6'02'0 2'14 Kc = ⋅ ⋅ = b. Sustituyendo en la expresión de la constante: ( )( ) ( )( ) 40 x6'0·x6'0 x2'1·x4 Ke = −− ++ = ; 40 36'0x2'1x 8'4x2'5x 2 2 = +− ++ ordenando como una ecuación de segundo grado y resolviendo:    = = =++ 21'0x 15'1x 06'9x2'53x39 2 despreciando la solución x = 1’15 por ser mayor que las concentraciones de partida, las concentraciones en equilibrio en (ml/l) son: Junio 2004. Problema 1B. El yoduro de hidrógeno se descompone a 400 ºC de acuerdo con la ecuación ( ) )g(IgH)g(HI2 22 +↔ , siendo el valor de Kc = 0’0156. Una muestra de 0’6 moles de HI se introduce en un matraz de 1 L y parte del HI se descompone hasta que el sistema alcanza el equilibrio. a) ¿Cuál es la concentración de cada especie en el equilibrio? b) Calcule KP. c) Calcule la presión total en el equilibrio. Puntuación máxima por apartado: a) y c) 0’75 puntos; b) 0’5 puntos. Solución. a. Se pide estudiar el equilibrio de disociación del yoduro de hidrógeno conocida la constante de equilibrio, las variables de estado(P, V, T) y la concentración inicial. La constante de equilibrio, según la Ley de Acción de Masas tiene la expresión: 2 22 c HI IH K ⋅ = por no haber variación en el numero de moles, la constante se puede expresar en función del número de moles en el equilibrio. 2 HI IH 2 HI IH 2 22 c n nn V n V n V n HI IH K 22 22 ⋅ =       ⋅ = ⋅ = sustituyendo los datos de la tabla ( ) ( ) 2 2 2 22 HI IH c x26'0 x x26'0 x x26'0 xx n nn K 22       − = − = − ⋅ = ⋅ = expresión de la que se puede despejar x en función de Kc
  • 37. 37 06'0 0156'021 0156'06'0 K21 K6'0 x x26'0 x K c c c = + ⋅ = + ⋅ =⇒ − = conocido el valor de x, se puede calcular el número de moles en el equilibrio, y conocido el volumen, la concentración de cada especie en el equilibrio. l mol06'0 1 06'0 V n IH06'0nn l mol48'0 1 48'0 V n HI48'006'026'0n 22IH HI HI 22 ====⇒== ===⇒=⋅−= b. Teniendo en cuenta que no hay variación en el número de moles de reactivos a productos(∆n=0), y que: ( ) 0156'0KKRTKK cp n cp ==⇒⋅= ∆ c. La presión total se calcula mediante la ecuación de estado de gases ideales utilizando el número total de moles presentes en el equilibrio 6'006'006'048'0nnnn 22 IHHIT =++=++= TRnVP ⋅⋅=⋅ ( )atm1'33 1 673082'06'0 V TRn P = ⋅⋅ = ⋅⋅ = Septiembre 2003. Problema 2B. En un recipiente cerrado de volumen constante igual a 22 L y a la temperatura de 305 K se introduce 1 mol de N2O4(g). Este gas se descompone parcialmente según la reacción N2O4 (g) ↔ 2NO2 (g) cuya constante de equilibrio KP vale 0,249 a dicha temperatura. a) Calcule el valor de la constante de equilibrio, Kc. b) Determine las fracciones molares de los componentes de la mezcla en equilibrio. c) ¿Cuál es la presión total cuando se ha alcanzado el equilibrio? Dato: R = 0,082 atm·mol-1 ·K-1 Puntuación máxima por apartado: b)1,0 punto; a) y c) 0,5 puntos. Solución. a. Según la ley de acción de masas 42 2 2 c ON NO K = Aplicando la ecuación de gases ideales a cada componente de la mezcla RT P V n :RTnVP ii ii ==⋅ siendo V ni la concentración de componente i. RT P ON: RT P NO 422 ON 42 NO 2 == sustituyendo estas expresiones en Kc y ordenando RT 1 K RT 1 P P RT P RT P K p K ON 2 NO ON 2 NO c P 42 2 42 2 ⋅=⋅=             = 321 expresión que permite calcular Kc conocida KP. Sustituyendo los datos 2 c 10 305082'0 1 249'0K − = ⋅ ⋅= b. Partiendo de la estequiometria de la reacción se plantean las condiciones de equilibrio en función del número inicial de moles de tetraóxido de nitrógeno y del grado de disociación
  • 38. 38 sustituyendo en Kc y ordenando V 1 1 n4 V nn V n2 K 2 o oo 2 o c ⋅ α− α = α−       α = Sustituyendo por los datos α− α =⋅ α− α⋅⋅ =− 1 4 22'0: 22 1 1 14 10 22 2 ordenando se obtiene una ecuación de segundo grado. Al resolver la ecuación se obtienen dos valores de los que uno de ellos no tendrá sentido químico    −=α =α =−α+α químicosentidotieneNo26'0 21'0 :022'022'04 2 Las fracciones molares se pueden expresar en función del grado de disociación teniendo en cuenta que, el número total de moles presentes en el equilibrio es la suma de los moles de NO2 y de N2O4. ( ) ( )α+=α+=α−+α=+= 1nnnnnn2nnn ooooooONNOT 422 ( ) ( ) ( )       = + − = α+ α− = α+ α− ==χ = + ⋅ = α+ α = α+ α ==χ 65'0 21'01 21'01 1 1 1n 1n n n 35'0 21'01 21'02 1 2 1n n2 n n o o T ON NO o o T NO NO 42 2 2 2 c. La presión de equilibrio se puede calcular de dos formas, mediante KP ó mediante la ecuación de gases ideales. Empleando esta segunda forma: atm38'1 22 305082'021'1 V RTn P:RTnVP T T = ⋅⋅ = ⋅ =⋅=⋅ Junio 2003. Cuestión 3. Justifique si las siguientes afirmaciones son ciertas o falsas: a) Un valor negativo de una constante de equilibrio significa que la reacción inversa es espontánea. b) Para una reacción exotérica, se produce un desplazamiento hacia la formación de productos al aumentar la temperatura. c) Para una reacción a temperatura constante con igual número de moles gaseosos de reactivos y productos, no se produce desplazamiento del equilibrio si se modifica la presión. d) Para una reacción a temperatura constante donde únicamente son gases los productos, el valor de la constante de equilibrio disminuye cuando disminuimos el volumen del recipiente. Puntuación máxima por apartado: 0,5 puntos. Solución. a) Falso. La constante de equilibrio es la relación entre el producto de las concentraciones de los productos y el producto de las concentraciones de los relativos. Al ser las concentraciones nº reales positivos, las constantes de equilibrio son positivas. b) Falso. Una reacción exotérmica desprende calor, si se aumenta la temperatura, se introduce calor en el sistema y el sistema se desplaza hacia los reactivos para volver a recuperar el equilibrio. QBA +⇔ c) Verdadero. Las variaciones de presión solo modifican el equilibrio en sistemas gaseosos en los que el número de moles gaseosos de reactivos es distinto al número de moles gaseosos de productos. d) Falso. Las constantes de equilibrio solo son funciones de la temperatura y de la naturaleza de los reactivos, no siendo función de la concentración, volumen ó presión.
  • 39. 39 Junio 2003. Problema 2B. El equilibrio PCl5 (g) ↔ PCl3 (g) + Cl2 (g) se alcanza calentando 3 g de pentacloruro de fósforo hasta 300ºC en un recipiente de medio litro, siendo la presión final de 2 atm. Calcule: a) El grado de disociación del pentacloruro de fósforo. b) El valor de Kp a dicha temperatura. Datos.- .0,31P;5,355CI:atómicasMasas;K·atm·L·mol082,0R 1-1 === − Puntuación máxima por apartados: 1 punto. Solución. a. Se pide estudiar el equilibrio de disociación del pentacloruro de fósforo en tricloruro de fósforo y cloro, conocida la cantidad inicial de pentacloruro de fósforo y las variables presión, volumen y temperatura de equilibrio. Las condiciones iniciales y de equilibrio se plantean en el siguiente cuadro donde α representa el grado de disociación del pentacloruro de fósforo El número total de moles en el equilibrio se puede expresar en función del número de moles iniciales y del grado de disociación: nT = n (PCl5) + n (PCl3) + n (Cl2) = no − noα + noα + noα = no + noα = no· (1 + α) siendo ( ) ( ) moles0144'0 mol gr208'5 gr3 PClM PClm n 5 5 o === El número total de moles se puede calcular a partir de las variables de estado que definen el equilibrio mediante la ecuación de gases ideales: TR VP nT ⋅ ⋅ = igualando las dos expresiones de nT ( ) TR VP 1no ⋅ ⋅ =α+⋅ se puede despejar el grado de disociación del pentacloruro de fósforo 1 TRn VP o − ⋅⋅ ⋅ =α sustituyendo por los datos del problema ( ) ( ) ( ) ( )( ) 478'01 K273300 Kmol Latm 082'0moles0'0144 L5'0atm2 =− +⋅      ⋅ ⋅ ⋅ ⋅ =α El grado de disociación del pentacloruro de fósforo es del 47’8 %. b. La constante de equilibrio en función de las presiones viene dada por la expresión: 5 23 PCl ClPCl P P PP K ⋅ = que mediante la ley de Raoult “Pi = P · χi” se puede expresar en función de la fracciones molares de los componentes gaseosos de la mezcla 5 23 5 23 PCl ClPCl PCl ClPCl P P ·P ·P·P K χ χ⋅χ ⋅= χ χ⋅χ = Las fracciones molares se pueden expresar en función del grado de disociación ( ) ( ) ( ) α+ α− = α+ α− ==χ 1 1 1n 1n n PCln o o T 5 PCl5 ( ) ( ) α+ α = α+ α ==χ=χ 11n n n PCln o o T 3 ClPCl 23